You are on page 1of 141

Generated by Foxit PDF Creator Foxit Software http://www.foxitsoftware.com For evaluation only.

NP1 NP2 NP3 NP4 NP5

PAGE 1 PAGE 30 PAGE 60 PAGE 88 PAGE 112

Generated by Foxit PDF Creator Foxit Software http://www.foxitsoftware.com For evaluation only.

NURSING PRACTICE I 1. A nurse calls the physician of a client scheduled for a cardiac catheterization because the client has numerous questions regarding the procedure and has requested to speak to the physician. The physician is very upset and arrives at the unit to visit the client after prompting by the nurse. The nurse is outside of the clients room and hears the physician tell the client in a derogatory manner that the nurse doesnt know anything. Which legal tort has the physician violates? a. Libel b. Slander c. Assault d. Negligence Answer: B Defamation takes place when something untrue is said (slander) or written (libel) about a person, resulting in injury to that persons good name and reputation. An assault occurs when a person puts another person in fear of a harmful or an offensive contact. Negligence involves the actions of professionals that fall below the standard of care for a specific professional group. Source: Saunders Q&A Review for NCLEX-RN by Linda Anne Silvestri, 2nd edition, page 62. 2. A nurse is assessing a client who has just been measured and fitted for crutches. The nurse determines that the clients crutches are fitted correctly if: a. The elbow is at a 30 degrees angle when the hand is on the handgrip b. The elbow is straight when the hand is on the handgrip c. The clients axilla is resting on the crutches pad during ambulation d. The top of the crutch is even with the axilla Answer: A For optional upper extremity leverage, the elbow should be at approximately 30 degrees of flexion when the hand is resting on the handgrip. The top of the crutch need to be two to three fingerwidths lower than the axilla. When crutch walking, all weight needs to be on the hands to prevent nerve palsy from pressure on the axilla. Source: Saunders Q&A Review for NCLEX-RN by Linda Anne Silvestri, 2 edition, page 73. 3. The first attempt to elevate nursing as a profession by enriching and broadening the preparation of nurses and by educating them in University setting is an idea conceived by: a. b. c. d. Rosario Delgado Julita V. Sotejo Florence Nightingale Faye Abdellah
nd

Answer: B Julita V. Sotejo is a nurse and lawyer who became the first dean of the University of the Philippines, College of Nursing Source: Fundamentals in Nursing by Tungpalan page 37-38
For more nursing reviewers, contact ebookwholeseller@yahoo.com 09394837323

Generated by Foxit PDF Creator Foxit Software http://www.foxitsoftware.com For evaluation only.

4. A nurse is instructing a client how to safely use crutches for ambulating at home. Which measure would the nurse recommend to minimize the risk of falls while ambulating with the crutches? a. Use grab bars in the bathtub or shower b. Remove scatter rugs in the home c. Keep all pets out of the house d. Use soft-soled slippers when walking with the crutches Answer: B To reduce the risk of falls, all obstacles should be removed from the home. Not all pets are trip hazards (fish, birds, guinea pigs). Grab bars in the bathtub or shower will not necessarily assist the client while walking with crutches. Shoes with non-slip soles should be worn. Source: Saunders Q&A Review for NCLEX-RN by Linda Anne Silvestri, 2 edition, page 75. 5. A client is being discharged and will receive oxygen therapy at home. The nurse is teaching the client and family about oxygen safety measures. Which of the following statements by the client indicates the need for further teaching? a. b. c. d. I realize that I should check the oxygen level of the portable tank on a consistent basis. I will keep my scented candles within 5 feet of my oxygen tank. I will not sit in front of my wood-burning fireplace with my oxygen on. I will call the physician if I experience any shortness of breath.
nd

Answer: B Oxygen is a highly combustible gas, although it will not spontaneously burn or cause an explosion. It can easily cause fire to ignite in a clients room if it contacts a spark from a cigarette, burning candle or electrical equipment. Options A, C, and D are appropriate oxygen safety measures. Source: Saunders Q&A Review for NCLEX-RN by Linda Anne Silvestri, 2nd edition, page 110. 6. The four main concepts common to nursing that appear in each of the current conceptual models are: a. b. c. d. Person, Nursing , Environment, Medicine Person, Health, Nursing, Support System Person, Health, Psychology, Nursing Person, Environment, Health, Nursing

Answer: D The four concepts that have been accepted by all theorists as the focus of nursing practice from the time of Florence Nightingale include the PERSON, receiving the nursing care, his ENVIRONMENT, his HEALTH on the health-illness continuum, and the NURSING, actions necessary to meet his needs. Source: Nurse Test Review Series (Fundamentals) page 51 7. A nurse is taking care of a client on contact isolation. After the nursing care has been performed, on leaving the room, which protective item during client care, would the nurse remove first? a. b. c. d. Gloves Mask Eye wear(goggles) Gown

Answer: C The nurse removes the goggles first. The nurse unties the gown at the waist and then removes the goggles and discards them. The nurse then removes and discards the mask, unties the neck strings of the gown and allows the gown to fall from the shoulders. The gown is removed without touching the outside of the gown and discarded. The hands are then washed.
For more nursing reviewers, contact ebookwholeseller@yahoo.com 09394837323

Generated by Foxit PDF Creator Foxit Software http://www.foxitsoftware.com For evaluation only.

Source: Saunders Q&A Review for NCLEX-RN by Linda Anne Silvestri, 2nd edition, page 93. 8. An older adult woman client with a fractured left tibia has a long leg cast and is using crutches to ambulate. In caring for the client, the nurse assesses for which of the following signs and symptoms that indicate a complication associated with crutch walking? a. Forearm muscle weakness b. Left leg discomfort. c. Triceps muscle spasm d. Weak biceps brachii Answer: A Forearm muscle weakness is a sign of radial nerve injury caused by crutch pressure on the axillae. When clients lack upper body strength, especially in the extensor and flexor muscle of the arms, they frequently allow their weight to rest on their axillae instead of their arms while ambulating with crutches. Leg discomfort is expected as a result of the injury. Triceps muscle spasm may occur as a result of increase muscle use but is not a complication of crutch walking. Weak biceps brachii is a common physical assessment finding in older adults and is not a complication of crutch walking. Source: Potter, P., & Perry, A. (2001). Fundamentals of nursing (5 th ed.). St. Louis: Mosby, p.1008. 9. A client requests pain medication and the nurse administers an intramuscular (IM) injection. After administration of the injection, the nurse does which of the following first? a. Recaps the needle b. Removes the gloves c. Washes the hands d. Places the syringe in the puncture-resistant needle box container Answer: D Following administration of an IM injection, the nurse would massage the site to assist in medication absorption. Then the nurse assists the client to a comfortable position. The uncapped needle is discarded in a puncture-resistant container, gloves are removed, and the hands are washed. A needle is never recapped. Of the options provided, the nurse would perform option D first. Source: Saunders Q&A Review for NCLEX-RN by Linda Anne Silvestri, 2 edition, page 93. 10. A nursing manager is reviewing the purpose for applying restraints with the nursing staff. The nurse manager tells the staff that which of the following is not an indication for the use of a restraint? a. To prevent falls b. To restrict movement of a limb c. To prevent the client from pulling out IV lines and catheters d. To prevent the violent client from injuring self and others Answer: A Restraints do not necessarily prevent falls. Restraints are devices used to restrict the clients movement in situations when it is necessary to immobilize a limb or other body part. They are applied to prevent selfinflicted injury or from injuring others; from pulling out intravenous lines, catheters, or tubes; or from removing dressings. Restraints also may be used to keep children still and from injuring themselves during treatments and diagnostic procedures. Restraints should not be used as a form of punishment. Source: Saunders Q&A Review for NCLEX-RN by Linda Anne Silvestri, 2nd edition, page 94. 11. A client who is scheduled for gallbladder surgery is mentally impaired and is unable to communicate. In regard to obtaining permission for the surgical procedure, which nursing intervention would be most appropriate? a. Ensure that the family has signed the informed consent b. Ensure that the client has signed the informed consent
For more nursing reviewers, contact ebookwholeseller@yahoo.com 09394837323
nd

Generated by Foxit PDF Creator Foxit Software http://www.foxitsoftware.com For evaluation only.

c. Inform the family about the advance directive process d. Inform the family about the process of a living will Answer: A A client must be alert, able to communicate, and competent to sign the informed consent. If the client is unable to, then the family can sign the consent. A living will lists the medical treatment a person chooses to omit or refuse if the person becomes unable to make decisions and is terminally ill. Advanced directives are forms of communication in which persons can give direction on how they would like to be treated when they cannot speak for themselves. Source: Saunders Q&A Review for NCLEX-RN by Linda Anne Silvestri, 2 edition, page 92. 12. A client diagnosed with tuberculosis (TB) is scheduled to go to the radiology department for a chest xray evaluation. Which nursing intervention would be appropriate when preparing to transport the client? a. Apply a mask to the client b. Apply a mask and gown to the client c. Apply a mask, gown, and gloves to the client d. Notify the x-ray department that the personnel can be sure to wear a mask when the client arrives. Answer: A Clients known or suspected of having TB should wear a mask when out of the room to prevent the spread of the infection to others. A gown or gloves are not necessary. Source: Saunders Q&A Review for NCLEX-RN by Linda Anne Silvestri, 2 edition, page 92. 13. A nurse is observing a client using a walker. The nurse determines that the client is using the walker correctly if the client: a. Puts all four points of the walker flat on the floor, puts weight on the hand pieces, and then walks into it b. Puts weight on the hand pieces, moves the walker forward, and the walks into it. c. Puts weight on the hand pieces, slides the walker forward, and then walks into it. d. Walks into the walker, puts weight on the hand pieces, and then puts all four points of the walker flat on the floor. Answer: A When the client uses a walker, the nurse stands adjacent to the affected side. The client is instructed to put all four points of the walker two feet forward flat on the floor before putting weight on the hand pieces. This will ensure client safety and prevent stress cracks in the walker. The client is then instructed to move the walker forward and walk into it. Source: Saunders Q&A Review for NCLEX-RN by Linda Anne Silvestri, 2nd edition, page 77. 14. A nurse has an order to obtain a 24-hour urine collection of a client with renal disorder. The nurse avoids which of the following to ensure proper collection of the 24-hour specimen? a. b. c. d. Have the client void at the start time, and place this specimen in the container. Discard the first voiding; save all subsequent voiding during the 24-hour time period. Place the container on ice, or in a refrigerator Have the client void at the end time and place this specimen in the container.
nd nd

Answer: A The nurse asks the client to void at the beginning of the collection period and discards the urine sample. All subsequent voided urine is saved in a container, which is placed on ice or refrigerated. The client is asked to void at the finish time, and this sample is added to the collection. The container is labeled, placed on fresh ice, and sent to the laboratory immediately. Source: Potter, P., & Perry, A. (2001). Fundamentals of nursing (5 th ed.). St. Louis: Mosby, p.1398.
For more nursing reviewers, contact ebookwholeseller@yahoo.com 09394837323

Generated by Foxit PDF Creator Foxit Software http://www.foxitsoftware.com For evaluation only.

15. A client is receiving total parenteral nutrition (TPN) via central intravenous (IV) line is scheduled to receive an antibiotic by the IV route. Which action by the nurse is appropriate before hanging the antibiotic solution? a. b. c. d. Ensure a separate IV access for the antibiotic. Turn off the TPN for 30 minutes before administering the antibiotic. Check with the pharmacy to be sure the antibiotic can be hung through the TPN line. Flush the central line with 60 mL of normal saline solution before hanging the antibiotic.

Answer: A The TPN line is used only for the administration of the TPN solution. Any other intravenous medication must be administered through a separate IV site. Source: Potter, P., & Perry, A. (2001). Fundamentals of nursing (5 ed.). St. Louis: Mosby, p.1218 16. A nurse has inserted a nasogastric (NG) tube to the level of the oropharynx and has repositioned the clients head in a flexed forward position. The client has been asked to begin swallowing. The client begins to cough, gag, and choke. Which of the following nursing actions would least likely result in proper tube insertion and promote client relaxation? a. Continue to advance the tube to the desired distance. b. Pulling the tube back slightly. c. Checking the back of the pharynx using a tongue blade and flashlight. d. Instructing the client to breath slowly. Answer: A As the NG tube is passed through the oropharynx, the gag reflex is stimulated, which may cause coughing, gagging, and choking. Instead of passing through the esophagus, the NG tube may coil around itself in the oropharynx, or it may enter the larynx and obstruct the airway. Since the tube may enter the larynx, advancing the tube may position it in the trachea. Slow breathing help the client relax to reduce the gag response. The tube maybe advance after the client relaxes. Source: Potter, P., & Perry, A. (2001). Fundamentals of nursing (5 ed.). St. Louis: Mosby, p.1467. 17. A nurse has an order to obtain a urinalysis from a client with an indwelling urinary catheter. The nurse avoids which of the following, which could contaminate the specimen? a. Obtaining the specimen from the urinary drainage bag b. Clamping the tubing of the drainage bag c. Aspirating a sample from the port on the drainage bag d. Wiping the port with an alcohol swab before inserting the syringe Answer: A A urine specimen is not taken from the urinary drainage bag. Urine undergoes chemical changes while sitting in the bag and does not necessarily reflect the current client status. In addition, it may become contaminated with bacteria from opening the system. Source: Saunders Q&A Review for NCLEX-RN by Linda Anne Silvestri, 2 edition, page 96 18. A nursing assistant is caring for an elderly client with cystitis who has an indwelling urinary catheter. The registered nurse provides directions regarding care and ensures that the nursing assistant: a. b. c. d. Uses soap and water to cleanse the perineal area Keeps the drainage bag above the level of the bladder Loops the tubing under the clients leg Lets the drainage tubing rest under the leg
nd th th

Answer: A Proper care of an indwelling urinary catheter is especially important to prevent prolonged infection or reinfection in the client with cystitis. The perineal area is cleansed thoroughly using mild soap and water at least twice a day and following a bowel movement. The drainage bag is kept below the level of the
For more nursing reviewers, contact ebookwholeseller@yahoo.com 09394837323

Generated by Foxit PDF Creator Foxit Software http://www.foxitsoftware.com For evaluation only.

bladder to prevent urine from being trapped in the bladder, and for the same reason, the drainage tubing is not placed or looped under the clients leg. The tubing must drain freely at all times. Source: Saunders Q&A Review for NCLEX-RN by Linda Anne Silvestri, 2 edition, page 96. 19. A nurse is inserting an indwelling urinary catheter into a male client. As the catheter is inserted into the urethra, urine begins to flow into the tubing. At this point, the nurse: a. Immediately inflates the balloon b. Withdraws the catheter approximately 1 inch and inflates the balloon c. Inserts the catheter until resistance is met and inflates the balloon d. Inserts the catheter 2.5 to 5 cm and inflates the balloon Answer: D The catheters balloon is behind the opening at the insertion tip. The catheter is inserted 2.5 to 5 cm after urine begins to flow in order to provide sufficient space to inflate the balloon. Inserting the catheter the extra distance will ensure that the balloon is inflated inside the bladder and not in the urethra. Inflating the balloon in the urethra could produce trauma. Source: Saunders Q&A Review for NCLEX-RN by Linda Anne Silvestri, 2nd edition, page 82. 20. A nurse is caring for a client with cancer. The client tells the nurse that a lawyer will be arriving today to prepare a living will. The client asks the nurse to act as one of the witnesses for the will. The most appropriate nursing action is to: a. b. c. d. Agree to act as a witness. Refuse to help the client. Inform the client that a nurse caring for the client cannot serve as a witness to a living will. Call the physician.
nd

Answer: C Living wills address the withdrawal or withholding of life sustaining interventions that unnaturally prolong life. It identifies the person who will make care decisions if the client is unable to take action. It is witnessed and signed by two people who unrelated to the client. Nurses or employees of a facility in which the client is receiving care, and beneficiaries of the client, must not serve as a witness. There is no reason to call the physician. Source: Potter, P., & Perry, A. (2001). Fundamentals of nursing (5th ed.). St. Louis: Mosby, p.436 21. Which of the following signs and symptoms would the nurse expect to find when assessing an Asian patient for postoperative pain following abdominal surgery? a. b. c. d. Decreased blood pressure and heart rate and shallow respirations Quiet crying Immobility, diaphoresis, and avoidance of deep breathing or coughing Changing position q 2 hours

ANSWER: C An Asian patient is likely to hide his pain. Consequently the nurse must observe for objective signs. In an abdominal surgery patient, these might include immobility, diaphoresis and avoidance of deep breathing or coughing, as well as increased heart rate, shallow respirations (stemming from pain upon moving the diaphragm and respiratory muscles), and guarding or rigidity of the abdominal wall. Such a patient is unlikely to display emotion such as crying. Source: Nurse Test: a review series, Fundamentals of Nursing. Page 80 22. A patient with signs and symptoms of congestive heart failure and leg edema has been placed on diuretic therapy. Which of the following data would best gauge his progress? a. Fluid intake and output b. Vital signs
For more nursing reviewers, contact ebookwholeseller@yahoo.com 09394837323

Generated by Foxit PDF Creator Foxit Software http://www.foxitsoftware.com For evaluation only.

c. Weight d. Urine specific gravity ANSWER: C A patient with congestive heart failure and leg edema has fluid overload, which typically results in weight gain. Thus, monitoring his weight is the most accurate way to measure his response to therapy. Intake and output measurements are helpful in evaluating fluid status but are not the best indicator of the patients progress. Vital signs particularly blood pressure, usually are used to monitor the progress of patients on antihypertensive or diuretic therapy. Vital signs can also help indicate other variables in a patients condition for example increased BP can be a reaction to stress, exercise or medication use. Urine specific gravity can indicate over hydration or dehydration. Source: Nurse Test: a review series, Fundamentals of Nursing. Page 81 23. The correct sequence for assessing the abdomen is: a. b. c. d. Tympanic percussion, measurement of the abdominal girth and inspection Assessment for distention, tenderness and discoloration around the umbilicus Percussion, palpation and auscultation Auscultation, percussion and palpation

ANSWER: D Because percussion and palpation can affect bowel motility and, thus, bowel sounds, they should follow auscultation in abdominal assessment. Tympanic percussion, measurement of abdominal girth and inspection are methods of assessing the abdomen. Assessing for distention, tenderness and discoloration around the umbilicus can indicate various bowel-related conditions, such as cholecystitis, appendicitis and peritonitis. Source: Nurse Test: a review series, Fundamentals of Nursing. Page 81 24. Penicillin is classified as an antibiotic with bactericidal action. The term bactericidal indicates that this antibiotic will: a. b. c. d. Inhibit the growth of a specific bacterium Destroy a specific bacterium Decrease the number of bacteria Increase the number of bacteria

ANSWER: B A bactericidal agent kills or destroys bacteria; a bacteriostatic agent inhibits the growth of bacteria. Source: Nurse Test: a review series, Fundamentals of Nursing. Page 240 25. A physician asks a nurse to discontinue the feeding tube in a client who is in a chronic vegetative state. The physician tells the nurse that the request was made by the clients spouse and children. The nurse understands the legal basis for carrying out the order and first checks the clients record for documentation of: a. A court approval to discontinue the treatment. b. A written order by the physician to remove the tube. c. Authorization by the family to discontinue the treatment. d. Approval by the institutional Ethics Committee. ANSWER: C The family or a legal guardian can make treatment decisions for the client who is unable to do so. Once the decision is made, the physician writes the order. Generally, the family makes decisions in collaboration with the physicians, other health care workers, and other trusted advisors. Source: Potter, P., & Perry, A. (2001). Fundamentals of nursing (5 th ed.). St. Louis: Mosby, p.436.
For more nursing reviewers, contact ebookwholeseller@yahoo.com 09394837323

Generated by Foxit PDF Creator Foxit Software http://www.foxitsoftware.com For evaluation only.

26. A nurse provides medication instructions to a home health care client. To ensure safe administration of medication in the home, the nurse: a. Demonstrate the proper procedure for taking prescribed medications. b. Allows the client to verbalize and demonstrate correct administration procedure. c. Instruct the client that it is OK to double up on medications if a dose has been missed. d. Conducts pill counts on each home visit. Answer: B To ensure safe administration of medication, the nurse allows the client to verbalize and demonstrate correct procedure and administration of medication. Demonstrating the proper procedure for the client does not ensure that the client safely perform this procedure. It is not acceptable to double up on medication, and conducting a pill count on each visit is not realistic or appropriate. Source: Potter, P., & Perry, A. (2001). Fundamentals of nursing (5 ed.). St. Louis: Mosby, p.492 27. A client is admitted to the hospital for a bowel resection following a diagnosis of a bowel tumor. During the admission assessment, the client tells the nurse that a living will was prepared three years ago. The client asks the nurse if this document is still effective. The most appropriate nursing response is which of the following? a. Yes it is. b. You will have to ask your lawyer. c. It should be reviewed yearly with your physician. d. I have no idea. Answer: C The client should discuss the living will with the physician and it should be reviewed annually to ensure that it contains the clients present wishes and desires. Option A is incorrect. Option D is not at all helpful to the client and is in fact a communication block. Although a lawyer would need to be consulted if the living will needed to be changed, the most appropriate and accurate nursing response would be to inform the client that the living will should be reviewed annually. Source: Saunders Q&A Review for NCLEX-RN by Linda Anne Silvestri, 2nd edition, page 51. 28. A nurses note that a postoperative client has not been obtaining relief of pain with prescribed narcotics, but only while a particular licensed practical nurse (LPN) is assigned to the client. The nurse: a. Reviews the clients medication administration record and immediately discuss the situation with the nursing supervisor b. Notifies the physician that the client needs an increase in narcotic dosage c. Decides to avoid assigning the LPN to the care of clients receiving narcotics d. Confronts the LPN with the information about the client having pain control problems and asks if the LPN is using the narcotics personally Answer: A In the situation, the nurse has noted an unusual occurrence, but before deciding what action to take next, the nurse needs more data than just suspicion. This can be obtained by reviewing the clients record. State and federal labor and narcotic regulations, as well as institutional policies and procedures, must be followed. It is therefore most appropriate that the nurse discuss the situation with the nursing supervisor before taking further action. The client does not need an increase in narcotics. To avoid assigning the LPN to clients receiving narcotics only ignores the issue. A confrontation is not the most advisable action, because the appropriate administrative authorities need to be consulted first. Source: Saunders Q&A Review for NCLEX-RN by Linda Anne Silvestri, 2nd edition, page 59. 29. A clients vital signs have noticeably deteriorated over the past four hours following surgery. A nurse does not recognize the significance of these changes in vital signs and take no action. The client later requires emergency surgery. The nurse could be prosecuted for which of these?
For more nursing reviewers, contact ebookwholeseller@yahoo.com 09394837323
th

Generated by Foxit PDF Creator Foxit Software http://www.foxitsoftware.com For evaluation only.

a. b. c. d.

Tort Misdemeanor Common law Statutory law

Answer: A A tort is a wrongful act intentionally or unintentionally committed against a person or his or her property. The nurses inaction in the situation described is consistent with the definition of a tort offense. Option B is an offense under criminal law. Option C describes case law that has evolved over time via precedents. Option D describes laws that are enacted by State, Federal, or local governments. Source: Saunders Q&A Review for NCLEX-RN by Linda Anne Silvestri, 2nd edition, page 60. 30. A nurse plans to carry out a multidisciplinary research project on the effects of immobility on clients stress levels. The nurse understands that which principle is most important when planning this project? a. Collaboration with other disciplines is essential to the successful practice of nursing. b. The corporate Nurse Executive should be consulted, because the project will take nursing time. c. All clients have the right to refuse to participate in research using human subjects. d. The cooperation of the physicians on staff must be ensured for the project to succeed. Answer: C The proposed project is research and includes human subjects. Although options a, b and d need to be considered, they are all secondary to the overriding principle of legal and ethical practice of nursing that any client has the right to refuse to participate in research using human subjects. Source: Potter, P., & Perry, A. (2001). Fundamentals of nursing (5 ed.). St. Louis: Mosby, p.436 31. A multidisciplinary health care team is planning care for client with hyperparathyroidism. The health care team develops which most important outcome for the client? a. b. c. d. Describes the administration of aluminum hydroxide gel. Restricts fluids to 1000 mL per day. Walk down the hall for 15 minutes, three times per day. Describes the use of loperamide (Imodium)
th

Answer: C Mobility of the client with hyperparathyroidism should be encouraged as much as possible because of the calcium imbalance that occurs in this disorder and predisposition to the formation of renal calculi. Fluids should not be restricted. Discussing the use of this medication is not the priority in this client. Source: Potter, P., & Perry, A. (2001). Fundamentals of nursing (5 edition) St. Louis: Mosby, p.1052 32. Stressors cause the release of the mineralocorticoid aldosterone, which regulates sodium absorption and potassium excretion in the renal tubules, resulting in: a. b. c. d. The need for supplemental potassium The need for a low sodium (500-mg) diet The conservation of water and maintenance of blood volume Increased diuresis
th

ANSWER: C Because aldosterone regulates the bodys sodium and potassium levels, it acts as an adaptive mechanism in maintaining blood volume and conserving water. Supplemental potassium usually is given to a patient with a low serum potassium level or one who is receiving a diuretic or other medication (such as digoxin) that has a mild diuretic effect. A low sodium diet is usually prescribed for a patient with a high serum sodium level, as in CHF, HPN or prolonged episodes of edema. Diuresis is increased naturally when a healthy patient increases his intake of fluids, especially those containing caffeine. Patients receiving diuretics also experience increase diuresis.
For more nursing reviewers, contact ebookwholeseller@yahoo.com 09394837323

Generated by Foxit PDF Creator Foxit Software http://www.foxitsoftware.com For evaluation only.

Source: Nurse Test: a review series, Fundamentals of Nursing. Page 125 33. The therapeutic effect of incentive spirometry depends on the: a. b. c. d. Maximum amount of air exhaled Sustained maximum deflation Maximum volume of air remaining after exhaling Sustained maximum inflation

ANSWER: D Incentive spirometry measures respiratory flow or volume. The patient is instructed to inhale slowly and deeply. At the point of maximum inspiration, he is asked to hold his breath for 3 to 5 seconds; this provides sustained maximum inflation. The other answers do not discuss maximum inflation. Source: Nurse Test: a review series, Fundamentals of Nursing. Page 136 34. a. b. c. d. The natural sedative in meat and milk products (especially warm milk) that can help induce sleep is:

Flurazepam Temazepam Tryptophan Methotrimeprazine

ANSWER: C Tryptophan is a natural sedative; flurazepam (Dalmane), temazepam (Restoril), and methotrimeprazine (Levoprome) are hypnotic sedatives. Source: Nurse Test: a review series, Fundamentals of Nursing. Page 165 35. One of the main principles of hospice program is that: a. b. c. d. The familys needs continue after the death of a loved one All persons need palliative care Hospice care must be provided by professional caregivers only Holistic care should not include medical care

ANSWER: A The national hospice organization developed the Standards of Hospice Programs in 1981, which includes the principle that the family a central part of palliative care - has needs that continue after the patients death. The other answers are incorrect for the following reasons: not all persons need or desire palliative care, hospice care consists of a blending of professional and nonprofessional services, and medical care is a necessary element of holistic care. Source: Nurse Test: a review series, Fundamentals of Nursing. Page 185 36. In the acceptance stage, the terminally ill patient reaches a point where he: a. b. c. d. Is happy Is neither depressed nor angry about his fate Has many mixed feelings Increased verbal communication with others

ANSWER: B In the acceptance stage, the patient is neither depressed nor angry about his fate; he is almost devoid of feelings. This state of mind should not be mistaken for happiness. In this final stage, the patient communicates more nonverbally than verbally: he may want to silently or just hold someones hand. Source: Nurse Test: a review series, Fundamentals of Nursing. Page 184

For more nursing reviewers, contact ebookwholeseller@yahoo.com

09394837323

10

Generated by Foxit PDF Creator Foxit Software http://www.foxitsoftware.com For evaluation only.

37. A nurse administers the morning dose of digoxin (Lanoxin) to the client. When the nurse charts the medication, the nurse discovers that a dose of 0.25 mg was administered rather than the prescribed dose of 0.125 mg. Which nursing action is most appropriate? a. b. c. d. Administer the additional 0.125 mg Tell the client that the dose administered was not the total amount and administer the additional dose Tell the client that too much medication was administered and an error was made Complete an incident report

Answer: D In accord with the agencys policy, nurses are required to file incident reports when a situation arises that could or did cause client harm. The nurse also contacts the physician. If a dose of 0.125 mg was prescribed, and a dose of 0.25 mg was administered, then the client received too much medication. Additional medication is not required and in fact should be detrimental. The client should be informed when an error has occurred, but in a professional manner so as to cause great fear and concern. In many situations, the physician will discuss this with the client. Source: Saunders Q&A Review for NCLEX-RN by Linda Anne Silvestri, 2 edition, page 115. 38. A registered nurse (RN) is orienting a nursing assistant to the clinical nursing unit. The RN would intervene if the nursing assistant did which of the following during a routine handwashing procedure? a. b. c. d. Kept hands lower than elbows Used 3 to 5 ml of soap from the dispenser Washed continuously for 10 to 15 seconds Dried from forearm down to fingers
nd

Answer: D Proper handwashing procedure involves wetting hands and wrist, keeping hands lower than forearms so water flows toward the fingertips. The nurse uses 3 to 5 ml of soap and scrubs for 10 to 15 seconds using rubbing and circular motions. The hands are rinsed and then dried, moving from the fingers to the forearms. The paper towel is then discarded, and a second one is used to turn off the faucet to avoid hand contamination. Source: Saunders Q&A Review for NCLEX-RN by Linda Anne Silvestri, 2nd edition, page 121 39. A client who is immunosuppressed is being admitted to the hospital and will be placed on neutropenic precautions. The nurse plans to ensure that which of the following does not occur in the care of the client? a. b. c. d. Placing a mask on the client if the client leaves the room Removing a vase with fresh flowers left by a previous client Admitting the client to a semi private room. Placing a precaution sign on the door to the room.

Answer: C The client who is on neutropenic precautions is immunosuppressed, and is admitted to a single (private) room on the nursing unit. A precaution sign should be placed on the door to the clients room. Removal of standing water and fresh flowers is done to decrease the microorganism count. The client should wear a mask whenever leaving the room to be protected from exposure to microorganisms. Source: Saunders Q&A Review for NCLEX-RN by Linda Anne Silvestri, 2nd edition, page 122 40. A client has an order for enemas until clear before major bowel surgery. After preparing the equipment and solution, the nurse assists the client into which of the following positions to administer the enema? a) Left-lateral Sims position b) Right-lateral Sims position c) Left side-lying with the head of the bed elevated 45 degrees
For more nursing reviewers, contact ebookwholeseller@yahoo.com 09394837323

11

Generated by Foxit PDF Creator Foxit Software http://www.foxitsoftware.com For evaluation only.

d) Right side-lying with the head of the bed elevated 45 degrees Answer: A For administration of an enema, the client is placed in a left-lateral Sims positions so that the enema solution can flow by gravity in the natural direction of the colon. The head of the bed is not elevated in the Sims position. Source: Potter, P., & Perry, A. (2001). Fundamentals of nursing (5 th ed.). St. Louis: Mosby, p.1463. 41. The nurse has complete tracheostomy care for a client whose tracheostomy tube has a nondisposable inner cannula. The nurse reinserts the inner cannula into the tracheostomy immediately after: a. b. c. d. Suctioning the clients airway. Rinsing it with sterile water. Tapping it against a sterile surface to dry it Drying it thoroughly with sterile gauze

Answer: C After washing and rinsing the inner cannula, the nurse dries it by tapping it against a sterile surface. The nurse then reinserts the cannula into the tracheostomy and turns it clockwise to lock it into place. Options A, B and D are inaccurate actions. Source: DeLaune, S., & Ladner, P., (1998). Fundamentals of nursing: Standards and practice, Albany, NY: Delmar, p.803 41. A nurse is caring for a client who has an order for dextroamphetamine (Dextrine) 25mg PO daily. The nurse collaborates with the dietician to limit the amount of which of the following items on the clients dietary trays? a. b. c. d. Starch Caffeine Protein Fat

Answer: B Dextroamphetamine is a central nervous system (CNS) stimulant. Caffeine is a stimulant also, and should be limited in client taking this medication. The client should be taught to limit their caffeine intake as well. Option A, C and D are acceptable dietary items. Source: Saunders Q&A Review for NCLEX-RN by Linda Anne Silvestri, 2nd edition, page 126. 43. Before performing a venipuncture to initiate continuous intravenous (IV) therapy, a nurse would: a. b. c. d. Apply a tourniquet below the chosen vein site. Inspect the IV solution for particles or contamination. Secure a arm board to the joint located above the IV site. Place a cool compress over the vein.

Answer: B All IV solution should be free of particles or precipitates. A tourniquet is to be above the chosen vein site. Cool compresses will cause vasoconstriction, making the vein less visible. Arm boards are applied after the IV is started. Source: Potter, P., & Perry, A. (2001). Fundamentals of nursing (5 th ed.). St. Louis: Mosby, p.1220

44. Which assessment is most important for the nurse to make before advancing a client from liquid to solid?
For more nursing reviewers, contact ebookwholeseller@yahoo.com 09394837323

12

Generated by Foxit PDF Creator Foxit Software http://www.foxitsoftware.com For evaluation only.

a. b. c. d.

Food preferences. Appetite. Presence of bowel sounds. Chewing ability.

Answer: D It may be necessary to modify a clients diet to a soft or mechanically chopped diet if the client has difficulty chewing. Food preferences should be ascertain on admission assessment. Appetite will affect the amount of food eaten, but not the type of diet ordered. Bowel sounds should be present before introducing any diet, including liquids. Source: Potter, P., & Perry, A. (2001). Fundamentals of nursing (5 th ed.). St. Louis: Mosby, p.1711. 45. a. b. c. d. A nurse is preparing to access an implanted vascular port to administer chemotherapy. The nurse: Anchors the port with the dominant hand. Palpates the port to locate the center of the septum. Places a warm pack over the area for several minutes to alleviate possible discomfort. Cleans the area with alcohol working from the outside ward.

Answer: B Before accessing an implanted port, the nurse must palpate the port to locate the center of the septum. The port should then be anchored with the non-dominant hand. Cool compresses over the site can help to alleviate pain upon entry. The site should be cleansed with alcohol working from the inside out to prevent introducing germs into the access site. Source: Potter, P., & Perry, A. (2001). Fundamentals of nursing (5 th ed.). St. Louis: Mosby, p.1219. 46. An elderly woman is brought to the emergency room. On physical assessment, the nurse notes old and new ecchymotic areas on both arms and buttocks. The nurse asks the client how the bruises were sustained. The client, although reluctant, tells the nurse in confidence that her daughter frequently hits her if she gets in the way. Which of the following is the moist appropriate nursing response? a. b. c. d. I promise I will not tell anyone but lets see what we can do about this. I have a legal obligation to report this type of abuse. Lets talk about ways that will prevent your daughter from hitting you. This should not be happening, and if it happens again you must call the emergency department.

Answer: B Confidential issues are not to be discussed with non-medical personnel or the persons family or friends without the persons permission. Clients should be assured that information is kept confidential, unless it places the nurse under a legal obligation. The nurse must report situations related to child or elderly abuse, gunshot wounds, and certain infectious disease. Source: Saunders Q&A Review for NCLEX-RN by Linda Anne Silvestri, 2nd edition , page 133. 47. A client tells the home health care nurse of the decision to refuse external cardiac massage. Which of the following is the most appropriate initial nursing actions? a. b. c. d. Notify the physician of the clients request Document the clients request in the home health nursing care plan Conduct a client conference with the home health care staff to share the clients request Discuss the clients request with the family

Answer: A External cardiac massage is one type of treatment that a client can refuse. The most appropriate nursing action is to notify a physician because a written Do not resuscitate (DNR) order from the physician must be present. The DNR order must be renewed on a regular basis per agency policy. Source: Saunders Q&A Review for NCLEX-RN by Linda Anne Silvestri, 2nd edition, page 134.
For more nursing reviewers, contact ebookwholeseller@yahoo.com 09394837323

13

Generated by Foxit PDF Creator Foxit Software http://www.foxitsoftware.com For evaluation only.

48. A nurse manager employs a leadership style in which decisions regarding the management of the nursing unit are made without input from the staff. Type of leadership style that is implemented by this nurse manager is: a. b. c. d. Autocratic Situational Democratic Laissez-faire

Answer: A The autocratic style of leadership is task oriented and directive. The leader uses his or her power and position in an authoritarian manner to set and implement organizational goals. Decisions are made without inputs from the staff. Democratic styles best empower staff toward excellence because this style of leadership allows nurses an opportunity to grow professionally. Situational leadership style utilizes a style depending on the situation and events. Laissez-faire allows staff to work without assistance, direction, or supervision. Source: Saunders Q&A Review for NCLEX-RN by Linda Anne Silvestri, 2 edition , page 137. 49. A registered nurse (RN) in charge is preparing the assignments for the day. The RN assigns a nursing assistant to make beds and bathe one of the clients on the unit and assigns another nursing assistant to fill the water pitchers and to serve juice to all the clients. Another RN is assigned to administer all medications. Based on the assignments designed by the RN in charge, which type of nursing care is being implemented? a. Functional nursing b. team nursing c. Exemplary model of nursing d. Primary nursing Answer: A. The functional model of care involves an assembly line approach to client care, with major tasks being delegated by the charge nurse to individual staff members. Team nursing is characterized by a high degree of communication and collaboration between members. The team is generally led by a registered nurse who is responsible for assessing, developing nursing diagnoses, planning and evaluating each clients plan of care. In an exemplary model of nursing, each staff member works fully within the realm of his or her educational and clinical experience in an effort to provide comprehensive individualized client care. Each staff member is accountable for client care and outcomes of care. In primary nursing, the concern is with keeping the nurse at the bedside actively involved in care, providing goal-directed and individualized client care. Source: Saunders Q&A Review for NCLEX-RN by Linda Anne Silvestri, 2nd edition, page 138 50. Visual acuity may be assessed by using a Snellen chart. If a patient has acuity of 20/40 in both eyes, this means: a. The patient can see twice as well as normal b. The patient has double vision c. The patient has less than normal vision d. the patient has normal vision Answer: C. Normal vision is 20/20. A finding of 20/40 would mean that a patient has les than normal vision. Source: Fundamentals of Nursing by Taylor, Lillis and Lemone, 5th Ed., p.610
nd

51. The nurse in a well baby clinic is providing safety instructions to a mother of a 1-month-old infant. Which of the following safety instructions is most appropriate at this age?
For more nursing reviewers, contact ebookwholeseller@yahoo.com 09394837323

14

Generated by Foxit PDF Creator Foxit Software http://www.foxitsoftware.com For evaluation only.

a. Cover electrical outlets b. Remove hazardous objects from low places c. Lock all poisons d. Never shake the infants head. Answer: D. The age-appropriate instruction that is most important is to instruct the mother not to shake or vigorously jiggle the babys head. Options A,. B & C are most important instructions to provide to the mother as the child reaches the age of 6 months and begins to explore the environment. Source: Saunders Q&A Review for NCLEX-RN by Linda Anne Silvestri, 2 edition, page 144 52. A nurse is receiving a client in transfer from the post anesthesia care unit following an above-theknee amputation. The nurse should take which of the following most important actions when positioning the client at this time? a. Put the bed in reverse Trendelenburgs position b. Keep the stump flat with the client lying on operative side c. Position the stump flat on the bed d. Elevate the foot of the bed. Answer: D. Edema of the stump is controlled by elevating the foot of the bed for the first 24 hours after surgery. Following the first 24 hours, the stump is placed flat on the bed to prevent hip contracture. Edema is also controlled by stump wrapping techniques. Source: Saunders Q&A Review for NCLEX-RN by Linda Anne Silvestri, 2nd edition, page 139 53. A nurse manager is planning to implement a change in the method of the documentation system in the nursing unit. Many problems have occurred as a result of the present documentation system and the nurse manager determines that a change is required. The initial step in the process of change for the nurse manager is which of the following? a. Plan strategies to implement the change b. Identify potential solutions and strategies for the change process. c. Set goals and priorities regarding the change process. D. Identify the inefficiency that needs improvement or correction. Answer: D. When beginning the change process, the nurse should identify and define the problem that needs improvement or correction. This important first step can prevent many future problems, because if the problem is not correctly identified, a plan for change may be aimed at the wrong problem. This is followed by goal setting, prioritizing and identifying potential solutions and strategies to implement the change. Source: Saunders Q&A Review for NCLEX-RN by Linda Anne Silvestri, 2nd edition, page 140 54. A nurse has received the client assignment for the day and is organizing the required tasks. Which of the following will not be a component of the plan for time management? a. b. c. d. Prioritizing client needs and daily tasks Providing time for unexpected tasks Gathering supplies before beginning a tasks Documenting task completion at the end of the day.
nd

Answer: D The nurse should document task completion continuously throughout the day. Option A, B, and C identify accurate component of time management. Source: Saunders Q&A Review for NCLEX-RN by Linda Anne Silvestri, 2nd edition, page 136.
For more nursing reviewers, contact ebookwholeseller@yahoo.com 09394837323

15

Generated by Foxit PDF Creator Foxit Software http://www.foxitsoftware.com For evaluation only.

55. A nurse enters the clients room and finds the client lying on the floor. Following assessment of the client, the nurse calls the nursing supervisor and the physician to inform them of the occurrence. The nursing supervisor instructs the nurse to complete an incident report. The nurse understands that incident reports allow the analysis of adverse client events by: a. b. c. d. Evaluating quality care and the client Determining the effectiveness of nursing intervention in relation to the client Providing a method of reporting injuries to local, state, and federal agencies Providing clients with necessary stabilizing treatments

Answer: A Proper documentation of unusual occurrences, incidents, and accidents, and the nursing actions taken as a result of the occurrence, are internal to the institution or agency and allow the nurse and administration to review the quality of care and determine any potential risks present. Incident reports are not routinely filled out for interventions nor are they used to report occurrences to other agencies. Source: Saunders Q&A Review for NCLEX-RN by Linda Anne Silvestri, 2nd edition , page 130. 56. A nurse observes that the client received pain medication 1 hour ago from another nurse, but that the client still has severe pain. The nurse has previously observed this same occurrence. The nurse practice act requires the observing nurse to do which of the following? a. b. c. d. Talk with the nurse who gave the medication Report the information to a nursing supervisor Call the impaired nurse organization Report the information to the police

Answer: B Nurse practice acts require reporting the suspicion of impaired nurses. The board of nursing has jurisdiction over the practice of nursing and may develop plans for treatment and supervision. This suspicion needs to be reported to the nursing supervisor, who will then report to the board of nursing. Source: Saunders Q&A Review for NCLEX-RN by Linda Anne Silvestri, 2 edition, page 131. 57. a patient has intravenous fluids infusing in the right arm. When taking a blood pressure on this patient, the nurse would: a. Take the blood pressure in the right arm. b. Take the blood pressure in the left arm. c. Use the smallest possible cuff d. report inability to take the blood pressure Answer: B. The blood pressure should be taken in the arm opposite the one with the infusion. Blood pressure should not be taken in the arm with an IV infusion because the pressure of inflating the cuff may allow the artery to clot. Source: Fundamentals of Nursing by Taylor, Lillis and Lemone, 5th Ed., p.558 58. A client is 2 days post operative. The vital signs are: BP - 120/70, HR - 110, RR - 26, and Temperature - 100.4 degrees Fahrenheit (38 degrees Celsius). The client suddenly becomes profoundly short of breath, skin color is gray. Which assessment would have alerted the nurse first to the client's change in condition? a. b. c. d. Heart rate Respiratory rate Blood pressure Temperature
nd

Answer B:
For more nursing reviewers, contact ebookwholeseller@yahoo.com 09394837323

16

Generated by Foxit PDF Creator Foxit Software http://www.foxitsoftware.com For evaluation only.

Tachypnea is one of the first clues that the client is not oxygenating appropriately. The compensatory mechanism for decreased oxygenation is increased respiratory rate. Lewis, S.M., Heitkemper, M.M., & Dirksen, S. R. (2004). Medical-Surgical Nursing: Assessment & management of clinical problems. St. Louis: Mosby. 59. Constipation is one of the most frequent complaints of elders. When assessing this problem, which action should be the nurse's priority? a. b. c. d. Add a thickening agent to the fluids Obtain a health and dietary history Refer to a provider for a physical examination Measure height and weight

Answer: B Initially, the nurse should obtain information about the chronicity of and details about constipation, recent changes in bowel habits, physical and emotional health, edications, activity pattern, and food and fluid history. This information may suggest causes as well as an appropriate, safe treatment plan. Source: Edelman, C.L. and Mandle, C.M.(2002). Health promotion throughout the lifespan. 60. While caring for a client, the nurse notes a pulsating mass in the client's periumbilical area. Which of the following assessments is appropriate for the nurse to perform? a. b. c. d. Measure the length of the mass Auscultate the mass Percuss the mass Palpate the mass

Answer: B Auscultation of the abdomen and finding a bruit will confirm the presence of an abdominal aneurysm and will form the basis of information given to the health care provider. The mass should not be palpated because of the risk of rupture. Smeltzer, S.C. and Bare, B.G. (2004). Medical surgical nursing. (10th edition). Philadelphia, PA. Lippincott Williams & Wilkins. Weber, J., and Kelley, J. (2003). Health Assessment in Nursing. (2nd edition). Philadelphia, PA: Lippincott Williams & Wilkins. 61. A client being treated for hypertension returns to the community clinic for follow up. The client says, "I know these pills are important, but I just can't take these water pills anymore. I drive a truck for a living, and I can't be stopping every 20 minutes to go to the bathroom." Which of these is the best nursing diagnosis? a. Noncompliance related to medication side effects b. Knowledge deficit related to misunderstanding of disease state c. Defensive coping related to chronic illness d. Altered health maintenance related to occupation Answer: A The client kept his appointment, and stated he knew the pills were important. He is unable to comply with the regimen from side effects, not a lack of knowledge about the disease process. Source: Key, J.L. and Hayes, E.R. (2003). Pharmacology, a nursing process approach. (4th edition). Philadelphia: Saunders

For more nursing reviewers, contact ebookwholeseller@yahoo.com

09394837323

17

Generated by Foxit PDF Creator Foxit Software http://www.foxitsoftware.com For evaluation only.

62. A client with congestive heart failure is newly admitted to home health care. The nurse discovers that the client has not been following the prescribed diet. What would be the most appropriate nursing action? a. b. c. d. Discharge the client from home health care related to noncompliance Notify the health care provider of the client's failure to follow prescribed diet Discuss diet with the client to learn the reasons for not following the diet Make a referral to Meals-on-Wheels

Answer: C When new problems are identified, it is important for the nurse to collect accurate assessment data. Before reporting findings to the health care provider, it is best to have a complete understanding of the clients behavior and feelings as a basis for future teaching and intervention. Source: Edelman, C.L. and Mandle, C.M.(2002). Health promotion throughout the lifespan. (5th edition). St. Louis, Missouri: Mosby. Smeltzer, S.C. and Bare, B.G. (2004). Medical surgical nursing. (10th edition). Philadelphia, PA. Lippincott Williams & Wilkins. 63. A client is admitted to the rehabilitation unit following a CVA and mild dysphagia. The most appropriate intervention for this client is: a. b. c. d. Position client in upright position while eating Place client on a clear liquid diet Tilt head back to facilitate swallowing reflex Offer finger foods such as crackers or pretzels

Answer: A An upright position facilitates proper chewing and swallowing. Source: Beare, P. and Myers, J. (1998) Adult Health Nursing. (3rd Edition). St. Louis, Missouri: Mosby. 64. A client has altered renal function and is being treated at home. The nurse recognizes that the most accurate indicator of fluid balance during the weekly visits is a. b. c. d. difference in the intake and output changes in the mucous membranes skin turgor weekly weight

Answer: D The most accurate indicator of fluid balance in an acutely ill individual is the daily weight. A one-kilogram or 2.2 pounds of weight gain is equal to approximately 1,000 mls of retained fluid. Other options are considered as part of data collection, but they are not the most accurate indicator for fluid balance. Source: Altman, G. (2004). Delmars Fundamental and Advanced Nursing Skills, 2nd ed. Albany, NY: Delmar. 65. One of the ethical obligations of nursing is accountability. Accountability means that the staff nurse is responsible for: a. The behavior of clients who are noncompliant b. The consequences of his or her actions, even mistakes in judgment
For more nursing reviewers, contact ebookwholeseller@yahoo.com 09394837323

18

Generated by Foxit PDF Creator Foxit Software http://www.foxitsoftware.com For evaluation only.

c. The behavior of other staff members who are negligent in their nursing care d. The consequences of an administrative decision to decrease nursing staff Answer: B Accountability means responsibility for nursing actions and the consequences of those actions, even if an honest mistake in judgment is made. Source: Tutor- Daviss NCLEX-RN Success, 2nd edition 66. An RN has been assigned for six clients for the 12-hour shift. The RN is responsible for every aspect of planning, giving, and evaluating their care during the shift. When leaving at 7:00 am, the nurse will pass this same responsibility to the incoming nurse. This illustrates nursing care delivered via the: a. b. c. d. Case method Functional method Team method Primary nursing method

Answer: A In case management, the nurse assumes total responsibility for meeting the needs of the client during his or her time o duty. Source: Tutor- Daviss NCLEX-RN Success, 2 edition 67. The nurse has been asked to witness an informed consent for surgery. The nurse understands that he or she is witnessing is that the: a. b. c. d. Informed consent took place Client signed the consent form Client was fully informed about the procedure. Family consented to the procedure.
nd

Answer: B The legal obligation of the witness is to verify only that the signature took place. Source: Tutor- Daviss NCLEX-RN Success, 2nd edition 68. A 7-week-old is admitted with a 2-week history of vomiting and weight loss. Tentative diagnosis is pyloric stenosis. While doing the admission assessment, in what order should the nurse assess the infants abdomen? a. b. c. d. Auscultate, inspect, palpate, percuss. Palpate, percuss, inspect, auscultate. Inspect, auscultate, percuss, palpate. Percuss, palpate, auscultate, inspect.

Answer: C The first step is to inspect or visually observe the abdomen. The second step is to auscultate or listen to all four quadrants of the abdomen. The third step would be to percuss and palpate the abdomen, or to feel the abdomen. Source: Tutor- Daviss NCLEX-RN Success, 2nd edition
For more nursing reviewers, contact ebookwholeseller@yahoo.com 09394837323

19

Generated by Foxit PDF Creator Foxit Software http://www.foxitsoftware.com For evaluation only.

69. A client is scheduled for cardioversion to treat sustained atrial fibrillation. The nursing priority before the procedure would be to: a. b. c. d. Auscultate the heart sounds. Administer medication for sedation. Give the prescribed analgesic. Start an antibiotic IV per order.

Answer: B During cardioversion the client is awake, but sedated. The anticipation of the procedure may be anxiety producing. Source: Tutor- Daviss NCLEX-RN Success, 2 edition 70. To monitor a clients fluid volume more closely, a central venous pressure (CVP) line has been inserted via the right subclavian vein. The nurse needs to know that CVP assesses the pressure in: a. b. c. d. The left atrium The right atrium The left ventricle The right ventricle.
nd

Answer: B CVP is a reflection of pressures in the right atrium and systemic veins. Although CVP is the least sensitive indicator of left ventricular end-diastolic pressure (increased with decreased ventricular compliance because of MI and left ventricular failure), the CVP line is a safer one than pulmonary artery (PA) line. In addition, it can be used to estimate blood volumes, obtain venous blood samples, and administer fluids. Source: Tutor- Daviss NCLEX-RN Success, 2 edition 71. The nursing priority to look for in assessing a client with right ventricular failure is the presence of: a. b. c. d. Fluid retention and distended neck veins. Weight gain and bradycardia. Confusion and apathy. Chest pain and elevated temperature.
nd

Answer: A Fluid retention and distended neck veins are direct effects of right-sided heart failure. Signs are manifested in the venous system. Source: Tutor- Daviss NCLEX-RN Success, 2nd edition 72. A client is to have a breast biopsy and possible mastectomy. Before going to see this client the morning of surgery, the nurse who is assigned to assist her in the final preparation for surgery should first: a. b. c. d. Prepare the preoperative medication. Check to be sure the operative permit has been assigned. Check to see if the operative laboratory reports have been placed in the chart. Check the diet orders to be sure the clients has been placed on NPO list.

Answer: B
For more nursing reviewers, contact ebookwholeseller@yahoo.com 09394837323

20

Generated by Foxit PDF Creator Foxit Software http://www.foxitsoftware.com For evaluation only.

Before any operative procedure can proceed, however minor, a voluntary, informed consent must be given. Source: Tutor- Daviss NCLEX-RN Success, 2nd edition 73. Which is not true about informed consent? a. Obtaining consent is the responsibility of the physician. b. A nurse may accept responsibility for witnessing a consent form. c. A physician subjects himself or herself to liability of the physician withholds any facts that are necessary to for the basis of an intelligent consent. d. If a nurse witnesses a consent for surgery, the nurse is, in effect, indicating that the client is informed. Answer: D The nurse who witnesses a consent for surgery or other procedure is witnessing only that the signature is that of the purported person and that the persons condition is as indicated at the time of signing. The nurse is not witnessing that the client is informed. Source: Tutor- Daviss NCLEX-RN Success, 2nd edition 74. In preparing preop injections for a 3 year old, which size needle would the nurse be most correct in selecting to administer IM injection? a. b. c. d. 25 G 5/8 inch 21G, 1 inch 18 G, 1 inch 18 G, 1 inch

Answer: B. In selecting the correct needle to administer an IM injection to a preschool child, the nurse should always ook at the child and use judgment in evaluating muscle mass and amount of subcutaneous fat. In this case, in the absence of further data, the nurse would be most correct in selecting a needle gauge and length appropriate for the average preschool child. A medium gauge needle 21G that is 1 inch long would be asppropriate. Source: Tutor Davis NCLEX RN, Success, 2nd Edition 75. Mr. L. is homeless and has gangrene on his foot. The physician has recommended hospitalization and surgery. Mr. L. has refused. The nurse knows which of the following is true? The client a. b. c. d. Cannot be hospitalized against his will. Can be restrained if one physician declares him incompetent Cannot choose which treatment to refuse. May sign against medical advice (AMA).

Answer: D. Against Medical Advice, or AMA is a term used with a patient who checks him or herself out of a hospital against the advice of his or her doctor. While it may not be medically wise for the person to leave early, in most cases the wishes of the patient are considered first. The patient is usually asked to sign a form stating that he or she is aware that he or she is leaving the facility against medical advice, and the AMA term is used on reports concerning the patient. This is for legal reasons in case there are complications to limit liability on the part of the medical facility. In a mental hospital setting, a patient is typically allowed to check out of the hospital by giving at least a day's notice (though in some jurisdictions the time may vary depending on whether the patient is under
For more nursing reviewers, contact ebookwholeseller@yahoo.com 09394837323

21

Generated by Foxit PDF Creator Foxit Software http://www.foxitsoftware.com For evaluation only.

"informal" or "formal" voluntary commitment). This is so that if the doctor feels that the patient would be a danger to self or others, the doctor has time to begin commitment proceedings against the patient to compel the patient to remain in the hospital for treatment.

Source: http://en.wikipedia.org/wiki/Against_medical_advice 76. Ms. R. has been medicated for her surgery. The operating room (OR) nurse, when going through the client's chart, realizes that the consent form has not been signed. Which of the following is the best action for the nurse to take? a. b. c. d. Tell the physician that the consent form is not signed. Assume it is emergency surgery and the consent is implied. Get the consent form and have the client sign it. Have a family member sign the consent form.

Answer: A. Informed consent is an agreement by a client to accept a course of treatment or a procedure after complete information, including the risks of treatment and facts relating to it, has been provided by the physician. It is therefore, the exchange between a client and a physician. Obtaining informed consent for specific medical and surgical treatments is the responsibility of the physician. Often, the nurses responsibility is too witness the giving of informed consent. This involves the ff: 1. Witnessing the exchange between the client and the physician 2. Establishing that the client really did understand 3. Witnessing the clients signature Source: Fundamentals of Nursing by Kozier, Erb, Blais and Wilkinson, 5 Ed., pp.228229
th

77. Mr. T. is a client on your medical-surgical unit. His cousin is a physician and wants to see the chart. Which of the following is the best response for the nurse to take? a. b. c. d. Tell the cousin that the request cannot be granted. Hand the cousin the client's chart to review. Call the attending physician and have the doctor speak with the cousin. Ask Mr. T. to sign an authorization, and have someone review the chart with the cousin.

Answer: D. Rationale: The clients record is protected legally as a private record of the clients care. Thus, access to the record is restricted to health professionals involved in giving care to the client. Insurance companies, for example, have no legal right to demand access to medical records, eventhough they may be determining compensation to the client. However a client who is making acclaim for compensation may ask to have the medical history used as evidence. In this instance, the client must sign an authorization for review, copying or release of information form the record. This form clearly indicates what information is to be released and to whom. In no instance may a nurse allow access to the clients record by significant others or any person other than a caregiver. Source: Fundamentals of Nursing by Kozier, Erb, Blais and Wilkinson, 5th Ed., p. 176 78. Ms. L. is admitted to the floor. She is in the terminal stages of AIDS. During the admission assessment, the nurse would ask her if she had which of the following except? a. An organ donation card. b. Healthcare proxy. c. Living will
For more nursing reviewers, contact ebookwholeseller@yahoo.com 09394837323

22

Generated by Foxit PDF Creator Foxit Software http://www.foxitsoftware.com For evaluation only.

d. Durable power of attorney for health care Answer : A Rationale: An advanced medical directive is a statement the client makes prior to receiving heath care, specifyingthe clients wishes regarding heath care decisions. There are three types of advance medical directives,the living will, the health care proxy and the Durable power of attorney for health care. The living will states what medical treatment the client chooses to omit or refuse in the event that the client is unable to make those decisions and is terminally ill. With a health care proxy, the client appoints a proxy, usually a relative or a trusted friend, to make medical decisions on the clients behalf, in the event that the client is unable to do so. A durable power of attorney is a notarized statement appointing someone else to manage health care treatment decisions when the client is unable to do so. Source: Fundamentals of Nursing by Kozier, Erb, Blais and Wilkinson, 5th Ed., p. 230 79. The nurse enters a room and finds a fire. Which is the best initial action? a. Activate the fire alarm or call the operator, depending on the institution's system. b. Get a fire extinguisher and put out the fire. c. Evacuate any people in the room, beginning with the most ambulatory and ending with the least mobile. d. Close all the windows and doors, and turn off any oxygen or electrical appliances. Answer : C. Rationale: Upon the detection of smoke and/or fire, follow the R-A-C-E plan described below. Rescue - Rescue/Remove person(s) from the immediate fire scene/room. Alert - Alert personnel by activating the nearest fire alarm pull station then call the Control Center to report the exact location of the fire. Confine - Confine fire and smoke by closing all doors in the area. Extinguish - Extinguish a small fire by using a portable fire extinguisher or use to escape from a large fire. Evacuate the building immediately and, once outside, report to your supervisor. Source: http://www.bu.edu/ehsmc/flipchart/firepro.htm 80. Ms. R. has had both wrists restrained because she is agitated and pulls out her IV lines. Which of the following would the nurse observe if Ms. R. is not suffering any ill effects from the restraints? That a. b. c. d. Ms. R.'s capillary refill is less than two seconds. She has difficulty moving her fingers and making a fist. Her skin is reddened where the mitts were tied around her wrist. The client complains of numbness and tingling in her hand.

Answer: A. Rationale: The client (in restraints) must periodically be evaluated for integrity of distal circulation, motor movement, and sensory level of the restrained extremities. (p.2263) Capillary refill time is an evaluation of peripheral perfusion and cardiac output. Capillaries usually refill in a fraction of a second but normal times range up to 3 seconds for color to return. With diminished blood flow, the return to the baseline color is delayed and a refill time of 3 seconds is sometimes called sluggish. (p. 1371) Source: Medical- Surgical Nursing Black, Hawks, Keene 81. When a patient you are admitting to the unit asks you why you are doing a history
For more nursing reviewers, contact ebookwholeseller@yahoo.com 09394837323

23

Generated by Foxit PDF Creator Foxit Software http://www.foxitsoftware.com For evaluation only.

and exam since the doctor just did one, your best reply is: a. In addition to providing us with valuable information about your health status, the nursing assessment will allow us to plan and deliver individualized, holistic nursing care that draws on your strengths. b. Its hospital policy. I know it must be tiresome, but I will try to make this quick! c. I am a student nurse and need to develop the skill of assessing your health status and need for nursing care. This information will help me develop a plan of care individualized to your unique needs. d. We want to make sure that your responses are consistent and that all our data are accurate. Answer: A. Though it may be true that you need to develop assessment skills (c) , the chief reason you are doing a nursing history and exam is because there needs to be a documented nursing admission assessment to serve as a basis for nursing care. Source: Fundamentals of Nursing by Taylor, Lillis and Lemone, 5 Ed., p.250 82. Mr. I. is supine. Which of the following can the nurse do to prevent external rotation of the legs? a. b. c. d. Put a pillow under the client's lower legs. Lower the client's legs so that they are below the hips. Use a trochanter roll alongside Mr. I.'s upper thighs. Place a pillow directly under the client's knees.
th

Answer: C. Rationale: When the legs are externally rotated, a trochanter roll is placed lateral to the femur. This would also prevent the external rotation of the hips. Source: Medical- Surgical Nursing Black, Hawks, Keene p. 912; Table 34-9 83. Mr. T. is a C4 quadriplegic. He has slid down in the bed. Which of the following is the best method for the nurse to use to reposition him? a. b. c. d. One nurse lifting under his buttocks while he uses the trapeze. One nurse lifting him under his shoulders from behind. Two people lifting him up in bed with a draw sheet. Two people log rolling the client from one side to the other.

Answer: C. Rationale: Two people are required to move clients who are unable to assist because of their condition or weight. Two nurses can use a turn sheet to move a client up in bed. A turn sheet distributes the clients weight more evenly, decreases friction, and exerts a more even force on the client during the move. In addition, it prevents injury on the clients skin. Source: Medical- Surgical Nursing Black, Hawks, Keene p.917 84. You are surprised to detect an elevated temperature (102 F) in a patient scheduled for surgery. The patient has been afebrile and shows no other signs of being febrile.. The first thing you do is to: a. inform the charge nurse. b. Inform the surgeon c. Validate your finding d. Document your finding
For more nursing reviewers, contact ebookwholeseller@yahoo.com 09394837323

24

Generated by Foxit PDF Creator Foxit Software http://www.foxitsoftware.com For evaluation only.

Answer: C. You should first validate your finding if it is unusual, deviates from normal and is unsupported by other data. Should your initial recoding prove to be in error, it would have been prematurity to notify the charge nurse. Source: Fundamentals of Nursing by Taylor, Lillis and Lemone, 5 Ed., p.250 85. The nurse knows the difference between the left lateral and the Sims position is that the a. Lateral position places the client's weight on the anterior upper chest and the left shoulder. b. Sims position is semiprone, halfway between lateral and prone. c. Lateral position places the weight on the right hip and shoulder. d. Sims position places the weight on the right shoulder and hip. Answer: B. Rationale: In (left) lateral position, the person lies on one side of the body (left). The top hip and knee are flexed and placed in front of the body to create a wider, triangular base of support. In Sims position, the patient assumes a posture halfway between the lateral and prone positions. The patient assumes a side- lying position with lowermost arm behind the body and uppermost leg flexed. Source: Medical- Surgical Nursing Black, Hawks, Keene (p. 468, 914) 86. a professional nurse committed to the principle of autonomy would be careful to: a. Provide the information and support a patient needed to make decisions to advance her own interests. b. Treat each patient fairly, trying to give everyone his or her due. c. Keep any promises made to a patient or another professional caregiver. d.Avoid causing harm to a patient. Answer: A. The principle of autonomy obligates us to provide the information and support patients and their surrogates need to make decisions that advance their interests. Source: Fundamentals of Nursing by Taylor, Lillis and Lemone, 5 Ed., p.110 87. Ms. S. is brought in after a motor vehicle accident. She has suffered a head injury and possible spinal injury. When moving her from the stretcher to the bed, the nurse should a. b. c. d. have the client move segmentally. log roll the client. move Ms. S. with a draw sheet. sit Ms. S. up and transfer her to the bed.
th th

Answer: B. Rationale: Logrolling is a technique used to turn a client whose body must at all times be kept in straight alignment. An example is a client with spinal injury. This technique requires two nurses, or if the client is large, three nurses. Source: Medical- Surgical Nursing Black, Hawks, Keene p.919 88. Ms. F. suffered a stroke and has right-sided hemiparesis. The nurse is going to transfer her from bed to wheelchair. Which of the following is the best method?
For more nursing reviewers, contact ebookwholeseller@yahoo.com 09394837323

25

Generated by Foxit PDF Creator Foxit Software http://www.foxitsoftware.com For evaluation only.

a. b. c. d.

Place the wheelchair about a foot away from the bed. Position the wheelchair closer to the weaker foot. Have the client put her arms around the nurse's neck. Put the wheelchair at a 45 angle to the bed.

Answer: D. Rationale: In transferring a client between a bed and a wheelchair, lower the bed to its lowest position so the clients feet will rest flat on the floor. Lock the wheels of the bed. Place the wheelchair parallel to the bed as close to the bed as possible. For clients who have difficulty walking, place the wheelchair at a 45 degree angle to the bed. This enables the client to pivot into the chair and lessens the amount of body rotation required. Source: Medical- Surgical Nursing Black, Hawks, Keene p.924- 925 89. The nurse knows which of the following is the proper technique for medical asepsis? a. b. c. d. Gloving for all client contact. Gowning to care for a one-year-old child with infectious diarrhea. Using your hands to turn off the faucet after handwashing. Changing hospital linen weekly.

Answer: B. Rationale: Medical asepsis includes all practices intended to confine a specific microorganism to a specific area, limiting the number, growth, and transmission of microorganisms. Objects are often referred to as clean or dirty. Clean or disposable gowns are worn during procedures when the nurses uniform is likely to become soiled. Source: Medical- Surgical Nursing Black, Hawks, Keene p. 678, 687 90. The nurse is conducting a class on aseptic technique and universal precautions. Which of the following statements is correct and should be included in the discussion? a. The term universal precautions is synonymous with disease or categoryspecific isolation precautions. b. Medical asepsis is designed to decrease exposure to bloodborne pathogens. c. Universal precautions are designed to reduce the number of potentially infectious agents. d. Medical asepsis is designed to confine microorganisms to a specific area, limiting the number, growth, and transmission of microorganisms. Answer: D. Rationale: The CDC did not recommend that universal precautions replace diseasespecific or category- specific precautions, but that they be used in conjunction with them. Universal precautions (not medical asepsis) decrease exposure to bloodborne pathogens. Isolation precautions (not universal precautions) are designed to prevent the spread of infections or potentially infectious microorganisms. Letter D provides a correct definition of medical asepsis. Source: Medical- Surgical Nursing Black, Hawks, Keene p.678- 680 91. The nurse is to open a sterile package from central supply. Which is the correct direction to open the first flap? a. b. c. d. Away from the nurse. To the nurse's left or right. It does not matter as long as the nurse only touches the outside edge. Toward the nurse.

Answer: A.
For more nursing reviewers, contact ebookwholeseller@yahoo.com 09394837323

26

Generated by Foxit PDF Creator Foxit Software http://www.foxitsoftware.com For evaluation only.

Rationale: To open a (sterile) wrapped package on a surface, place the package in the center of the work area so that the top flap of the wrapper opens away from you. This position prevents the nurse from subsequently reaching over the exposed sterile contents, which could contaminate them. Source: Fundamentals of Nursing by Kozier, Erb, Blais and Wilkinson, p.694 92. Which of the following statements or questions would be appropriate in establishing a discharge plan for a patient who has had major abdominal surgery? a. I will bet you will be so glad to be home in your own bed. b. What are your expectations for recovery from your surgery? c. Be sure and take your pain medications and change your dressing. d. You will just be fine! Please stop worrying. Answer: B. It is important to assess the expectations of the patient when assessing healthcare needs for discharge planning. Source: Fundamentals of Nursing by Taylor, Lillis and Lemone, 5 Ed., p.175 93. A patient who decides to leave the hospital against medical advice (AMA) must sign a form. What is the purpose of this form? a. To indicate the patients wishes b. To use in the event of readmission c. To release the physician and hospital from legal responsibility for the patients health status. d. To ethically illustrate that the patient has control of his or her own care and treatment. Answer: C. Patients who leave the hospital AMA sign a form releasing the physician and hospital from legal responsibility for their health status. This signed form becomes part of the medical record. Source: Fundamentals of Nursing by Taylor, Lillis and Lemone, 5th Ed., p.175
th

94. Ms. P. is transferred to a skilled nursing facility from the hospital because she is unable to ambulate due to a left femoral fracture. The nurse knows Ms. P.'s greatest risk factor for developing a pressure ulcer is that she a. b. c. d. Is apathetic but oriented to person, place, and time. Has slightly limited mobility and needs assistance to move from bed to chair. Has good skin turgor, no edema, and her capillary refill is less than three seconds. Is 5 ft 4 in tall, 130 lb, and eats more than half of most meals.

Answer : B. Rationale: Although pressure is the major cause of pressure ulcers, immobility and inactivity are also important risk factors. Immobility refers to an alteration in the amount and control of movement a person has. Inactivity refers to an alteration in a persons ability to ambulate independently. Source: Medical- Surgical Nursing Black, Hawks, Keene p. 786-787

95. An elderly male client is transferred to a skilled nursing facility from the hospital because he is unable to ambulate due to a left femoral fracture. When doing a skin assessment, the nurse notices a 3-cm, round area partial thickness skin loss that looks like a blister on the client's sacrum. The nurse knows this is a
For more nursing reviewers, contact ebookwholeseller@yahoo.com 09394837323

27

Generated by Foxit PDF Creator Foxit Software http://www.foxitsoftware.com For evaluation only.

a. b. c. d.

Stage II pressure ulcer. Stage I pressure ulcer. Stage III pressure ulcer. Stage IV pressure ulcer.

Answer: A. Rationale: Stages of pressure ulcer formation: Stage I- Non- blanchable erythema of intact skin. Stage II- Partial- thickness skin loss involving epidermis and/ or dermis. The ulcer is superficial and presents clinically as an abrasion, blister, or shallow crater. Stage III- Full- thickness skin loss involving damage or necrosis of subcutaneous tissue that may extend down to, but not through, underlying fascia. The ulcer presents clinically as a deep crater with or without undermining of adjacent tissue. Stage IV- Full- thickness skin loss with extensive destruction, tissue necrosis, or damage to muscle, bone, or supporting structures such as tendon or joint capsule. Source: Medical- Surgical Nursing Black, Hawks, Keene p.787-788 96. You are to administer a medication to Mr. B. In addition to checking his identification bracelet, you can correctly identify his identity by: a. Asking the patient his name. b. Reading the patients name on the sign over the head. c. Asking the patients roommate to verify his name. d. Asking, Are you Mr. B.? Answer: A. A sign over the patients bed may not be always current.. The roommate is an unsafe source of information.The patient may not hear his name but may reply in the affirmative way. Source: Fundamentals of Nursing by Taylor, Lillis and Lemone, 5 Ed., p.774-775 97. The nurse takes an 8am medication to the patient and properly identifies her. The patient asks the nurse to leave the medication on the bedside table and stats that she will take it when with breakfast when it comes. What is the best response to this request? a. Leave the medication and return later to make sure that it was taken. b. Tell her that it is against the rules, and take the medication with you. c. Tell her that you cannot leave the medication but will return with it when breakfast arrives. d. Take the drug from the room and record it as refused. Answer: C. Safe nursing practice requires that a medication never be left at the patients bedside. It is not correct to say that the patient has refused medication in this situation. Source: Fundamentals of Nursing by Taylor, Lillis and Lemone, 5th Ed., p.775 98. Why is the intravenous method of medication administration is called the most dangerous route of administration? a. The vein can take only a small amount of fluid at a time. b. The vein may harden and become nonfunctional. c. Blood clots may become a serious problem. Ds. The drug is placed directly into the bloodstream and its action is immediate. Answer: D. The intravenous route is a direct access to the bloodstream, and medications act quickly when given intravenously. The condition of the veins is not a s important as the
For more nursing reviewers, contact ebookwholeseller@yahoo.com 09394837323
th

28

Generated by Foxit PDF Creator Foxit Software http://www.foxitsoftware.com For evaluation only.

rapid effect of the medication administered intravenously. Source: Fundamentals of Nursing by Taylor, Lillis and Lemone, 5 Ed., p.775 99. Mr. A. is going home from the emergency room with directions to apply a cold pack to his ankle sprain. He asks how he will know if the cold pack has worked. The nurse tells him a. b. c. d. there should be less pain after applying the cold pack. that the skin will be blanched and numb afterward. he will notice the red-blue bruises will turn purple. after the first application, the swelling will be decreased.
th

Answer: A. Rationale: Cold compresses should be applied for 20 minutes at a temperature of 15C to relieve inflammation and swelling. When using cold compresses, the nurse observes for adverse reactions such as burning or numbness, mottling of the skin, redness, extreme paleness, and a bluish skin discoloration. Source: Fundamentals of Nursing by Potter and Perry 3rd Ed., p.1692 100. A nurse discovers that she has made a medication error. Which of the following should be her first response? a. Record the error on the medication sheet b. Notify the physician regarding course of action. c. Check the patients condition to note any possible effect of the error d. Complete an incident report, explaining how the mistake was made. Answer: C. The nurses first responsibility is the patient and careful observation is necessary to assess for any effect of the medication error. The other nursing actions are pertinent but only after checking the patient. Source: Fundamentals of Nursing by Taylor, Lillis and Lemone, 5 Ed., p.774
th

For more nursing reviewers, contact ebookwholeseller@yahoo.com

09394837323

29

Generated by Foxit PDF Creator Foxit Software http://www.foxitsoftware.com For evaluation only.

NURSING PRACTICE II 1. The dynamic care of this nursing tool provides measurement of progress. What is the scientific process for quality care? a. b. c. d. Nursing policies Nursing standard Nursing procedures Nursing process

Answer: D The nursing process is a systematic, scientific, dynamic, on going interpersonal process in which the nurses and the clients are viewed as a system with each affecting the other and both being affected by the factors within the behavior. The process is a series of actions that lead toward a particular result. This process of decision making results in optimal health care for the clients to whom the nurse applies the process Source: DOH (Green and Yellow) pp. 43 2. Which of the following serves as basis for evaluating nursing care plan for the patient and or family? a. b. c. d. Activities undertaken Nursing diagnosis Baseline information Set objectives of the plan

Answer: D Objectives refer to more specific statements of the desired results or outcomes of care. They specify the criteria by which the degree of effectiveness of care is to be measured. Source: Nursing Practice in the Community 4th Ed, pp. 98 3. The Dental Health Program of the DOH has committed to contribute to the improvement of the quality of life of Filipinos through its project Sang Milyong Sepilyo for which strategy? a. b. c. d. Operation research study Social mobilization Partnership with other sector Capability building and value formation

Answer: B The Dental Health Program conceptualizes a strategy through Sang Milyong Sepilyo project for Social Mobilization. Source: DOH (Green and Yellow) pp. 123

4. The setting under which health assessment will be made is best decided by:

For more nursing reviewers, contact ebookwholeseller@yahoo.com

09394837323

30

Generated by Foxit PDF Creator Foxit Software http://www.foxitsoftware.com For evaluation only.

a. b. c. d.

What is practical and effective The public health supervisor What is the agencies standard operating procedure Both the nurse and the client

Answer: D The nursing care plan is prepared jointly with the family. This is consistent with the principle that the nurse works with and not for the family. She involves the family in determining health needs and problems in establishing priorities, in selecting appropriate courses of actions, implementing them and evaluating outcomes. Through participatory planning, the nurse makes the family feel that the health of its members is a family responsibility and commitment. Source: Nursing Practice in the Community 4th Ed, pp. 84 Situation 1: Being a Public health Nurse, there are different Roles to play in the community. 5. A nurse who motivates changes in health behavior of individuals, families, group and community including lifestyle in order to promote and maintain health: a. b. c. d. Role model Trainer Community organizer Change agent

Answer: D A change agent is the one who motivates changes in health behavior of individuals, families, group and community including lifestyle in order to promote and maintain health. Source: Community Health Nursing Services in the Philippines, p. 24. 6. A nurse that develops the familys capability to take care of the sick, disabled, or dependent members: a. b. c. d. Programmer Community organizer Health educator Provider of Nursing Care

Answer: D A provider of Nursing Care also provides direct nursing to the sick, disabled in the home, clinic, school or place of work; and provides continuity of patient care. Source: Community Health Nursing Services in the Philippines, p. 21-22. 7. A nurse that is responsible for motivating and enhancing community participation in terms of planning, implementing and evaluating health programs and/or services a. b. c. d. Provider of nursing care Community organizer Counselor/trainer Supervisor/manager

Answer: B A community organizer is responsible for motivating and enhancing community participation in terms of planning, implementing and evaluating health programs and/or services; and initiates and participates in community development activities
For more nursing reviewers, contact ebookwholeseller@yahoo.com 09394837323

31

Generated by Foxit PDF Creator Foxit Software http://www.foxitsoftware.com For evaluation only.

Source: Community Health Nursing Services in the Philippines, p. 22.

8. A nurse that identifies the needs, priorities and problems of individuals, families and community: a. b. c. d. Health Educator Coordinator of Services Manager programmer

Answer: D A programmer also formulates nursing component of health plans; interprets and implements the nursing plan, program policies, memoranda and circulars for the concerned personnel/staff; and provides technical assistance to rural health midwives in health matters like target setting. Source: Community Health Nursing Services in the Philippines, p. 21. 9. A nurse who coordinates with the government and non-government organization in the implementation of the studies. a. b. c. d. Researcher Statistician Change agent Community organizer

Answer: A A researcher is the one who coordinates with the government and non-government organization in the implementation of the studies; and participates and/ or assist in the conduct of surveys studies and researches on nursing and health related subjects. Source: Community Health Nursing Services in the Philippines, p. 25. 10. Which of the following is the health concern in the primary level of prevention? a. b. c. d. Development of health habits and practices Poverty alleviation Early and prompt treatment Case finding

Answer: A Primary prevention is directed to the healthy population, focusing on prevention of emergence of risk factors and removal of the risk factors or reduction of their levels. Secondary prevention aims to identify and treat existing health problems at the earliest problems. The interventions at this stage can still lead to the control or eradication of the health problem. Such interventions include screening, casefinding, disease surveillance, prompt and appropriate treatment. Tertiary prevention limits disability progression. The nurse attempts to reduce the magnitude or severity of the residual effects of both infectious diseases and non communicable ones. Source: Nursing Practice in the Community 4th Ed, pp.180 11. Which one of the following is not a pillar of PHC? a. b. c. d. Multi sectoral approach Community involvement Appropriate technology Qualification of health providers
09394837323

For more nursing reviewers, contact ebookwholeseller@yahoo.com

32

Generated by Foxit PDF Creator Foxit Software http://www.foxitsoftware.com For evaluation only.

Answer: D The four cornerstones or pillars of Primary Health Care are active community participation, multisectoral linkages, use of appropriate technology and support mechanisms made available. Source: DOH ( Green and Yellow ) pp. 69 12. A guide or scheme used by the nurse in providing care for individuals and families is: a. b. c. d. nursing diagnosis Nursing assessment List of health problems Nursing care plans

Answer: D A family nursing care plan is the blueprint of care that the nurse designs to systematically minimize or eliminate the identified health and family nursing problems through explicitly formulated outcomes of care and deliberately chosen set of interventions, resources and evaluation criteria, standards, methods and tools. Source: Nursing Practice in the Community 4 Ed, pp. 83 13. Infant mortality rate means death under one year of age per 1000 live births. Which formula below is correct? X 100 a. Deaths under one year Live births of the same year b. Deaths under one year X 1000 Live births of the same year c. Live births of the same year X 100 Deaths under one year d. Live births of the same year X 1000 Deaths under one year Answer: B Infant mortality rate measures the risk of dying during the 1st year of life. It is a good index of the general health condition of a community since it reflects the changes in the environmental and medical conditions of a community. Source: DOH ( Green and Yellow ) pp. 330 14. These are essential characteristics you must consider most in providing primary health care except: a. b. c. d. Accessibility of health service Health Programs financial assistance Multisectoral approach to health care Appropriate technology
th

Answer: B A, C and D together with the support mechanism made available characterized the primary health care. Source: DOH ( Green and Yellow ) pp. 69

Situation 2: The following 2006 data are available in your health center. You analyze these for planning purposes.
For more nursing reviewers, contact ebookwholeseller@yahoo.com 09394837323

33

Generated by Foxit PDF Creator Foxit Software http://www.foxitsoftware.com For evaluation only.

No. of population Population under 1 year No. of infant deaths No. of live births No. of maternal deaths Total deaths from all causes No. of deaths from Pneumonia No. of Pneumonia cases 15. The crude birth rate for 2006 is: a. b. c. d. 24/1000 25/1000 30/1000 32/1000

600 600 6 900 100 -

24,000 3

450

Answer: B Crude Birth Rate=Total # of live births registered in a given calendar year X 1000 Mid Year Population = 600 24000 = 600000 24000 = 25/1000 Source: DOH ( Green and Yellow ) pp. 330 X 1000

16. The maternal mortality rate is: a. b. c. d. 10/1000 20/1000 12/1000 15/1000

Answer: A MMR =Total # of deaths from maternal causes registered for a given year X 1000 Total # of livebirths registered of same year = 6_ 600 = 6000 600 = 10/1000 Source: DOH ( Green and Yellow ) pp. 330 X 1000

17. The infant mortality rate is: a. 5/1000 b. 4/1000 c. 3/1000


For more nursing reviewers, contact ebookwholeseller@yahoo.com 09394837323

34

Generated by Foxit PDF Creator Foxit Software http://www.foxitsoftware.com For evaluation only.

d. 2/1000 Answer: A IMR=Total # of deaths under 1yr of age registered in a given calendar year X 1000 Total # of registered live births of same calendar year =3 600 = 3000 600 =5 Source: DOH ( Green and Yellow ) pp. 330 18. The case fatality rate for Pneumonia is: a. b. c. d. 40% 30% 22% 35% X 1000

Answer: C CFR=No. of registered deaths from a specific disease for a given year X 100 No of registered cases from same specific disease in same year = 100 450 X 100

= 10000 450 = 22% Source: DOH ( Green and Yellow ) pp. 333 19. The crude death rate is: a. b. c. d. 41/1000 38/1000 31/1000 25/1000

Answer. A Crude Death Rate= Total # of deaths registered in a given calendar year X 1000 Mid Year Population = 900 X 1000 24000 = 900000 24000 = 37.5/1000
For more nursing reviewers, contact ebookwholeseller@yahoo.com 09394837323

35

Generated by Foxit PDF Creator Foxit Software http://www.foxitsoftware.com For evaluation only.

= 38/1000 Source: DOH ( Green and Yellow ) pp. 333 20. What is believed to be a guarantee of effective delivery of health services? a. b. c. d. Integration Reorganization Devolution Promotion

Answer: C One of the most significant laws that radically changed the landscape of health care delivery system in the country is RA 7160 or more commonly known as the Local Government Code. The Code aims to transform local government units into self reliant communities and active partners in the attainment of national goals through a more responsive and accountable local government structure instituted through a system of decentralization. Source: Nursing Practice in the Community 4 Ed, pp. 25 21. Why was there devolution? a. b. c. d. To bring services nearer to the people To empower local politicians To become self reliant To stop funding from national funds
th

Answer: C One of the most significant laws that radically changed the landscape of health care delivery system in the country is RA 7160 or more commonly known as the Local Government Code. The Code aims to transform local government units into self reliant communities and active partners in the attainment of national goals through a more responsive and accountable local government structure instituted through a system of decentralization. Source: Nursing Practice in the Community 4th Ed, pp. 25 22. For a group of children where interaction with causative agents of disease has not taken place, the nurse concern is to provide: a. b. c. d. Diagnostic and curative type All this type of care Promotive and preventive care Rehabilitative care

Answer: C Primary prevention is directed to the healthy population, focusing on prevention of emergence of risk factors and removal of the risk factors or reduction of their levels. In communicable disease prevention, activities on primary prevention are targeted at intervening before the agent enters the host and cause pathological changes. Secondary prevention aims to identify and treat existing health problems at the earliest problems. The interventions at this stage can still lead to the control or eradication of the health problem. Such interventions include screening, case-finding, disease surveillance, prompt and appropriate treatment. Tertiary prevention limits disability progression. The nurse attempts to reduce the magnitude or severity of the residual effects of both infectious diseases and non communicable ones. Source: Nursing Practice in the Community 4th Ed, pp.180
For more nursing reviewers, contact ebookwholeseller@yahoo.com 09394837323

36

Generated by Foxit PDF Creator Foxit Software http://www.foxitsoftware.com For evaluation only.

23. When modifying harmful health practices, among minority families, the most important factor to remember is: a. b. c. d. Ensure that the change blends with their culture Increase health knowledge of the family Involve the family in bringing about change Persuade family to change practices.

Answer: C Involve the family in bringing about change. Active participation of individuals, families, and the health community, in planning and making decisions for their health care needs, determine to a large extent, the success of community health nursing program. Organized community groups are encouraged to participate in activities thnat will meet community needs and interest. Source: CHN by DOH page 31. 24. The following are approved types of toilet facilities under level I except: a. b. c. d. pit latrines reed odorless earth closet pour flush toilet water sealed and flush type with septic vault

Answer: D. Level I Non water carriage toilet facility no water is necessary to wash the waste into the receiving space. Ex: pit latrines, reed odorless wart closet Toilet facilities requiring small amount of water to wash the waste into the receiving space. Ex: pour flush toilet and aqua privies. Level II on site toilet facilities of the water carriage type with water sealed and flush type with septic vault/tank disposal facilities. Level III -water carriage types of toilet facilities connected to septic tanks and/ or to sewerage system to treatment plant. Source: Community Health Nursing Services in the Philippines, p. 317 25. An approved type of water supply facility which is composed of a source, a reservoir, a piped distribution network and communal faucets, located at not more than 25 meters from the farthest house is level: a. b. c. d. II I IV III

Answer: A. Level II (Communal faucet system or Stand Posts) a system composed of a source, a reservoir, a piped distribution network and communal faucets, located at not more than 25 meters from the farthest house. The system is designed to deliver 40-80 liters of water per capital per day to an average of 100 households. Level I (Point source) a protected well or developed spring with an outlet but without a distribution system, generally adaptable for rural areas where the house are thinly scattered. It serves around 15-25 households and its outreach must not be more than 250 meters from the farthest user. The yield or discharge is generally from 40-140 liters per minute.
For more nursing reviewers, contact ebookwholeseller@yahoo.com 09394837323

37

Generated by Foxit PDF Creator Foxit Software http://www.foxitsoftware.com For evaluation only.

Level III (Waterworks system or Individual House Connections) A system with a source, a reservoir, a piped distributor network and household taps. It is generally suited for densely populated urban areas; this type of facility requires a minimum treatment of disinfection. Source: Community Health Nursing Services in the Philippines, p. 315 26. Nurse Jessica volunteered to work with a cultural minority for a three month period. To reach to the place, they have to walk for 2 hours. Upon arrival, he noticed a toddler with thin, light colored hair, thin upper arm with swollen hands and feet, moonfaced, with dark spots around skin folds. This condition is described as : a. b. c. d. Failure to thrive Marasmus Kwashiorkor Avitaminosis

Answer: C Kwashiorkor is a malnutrition disease primarily of children caused by a severe protein deficiency that usually occurs when the child is weaned from the breast. Symptoms are retarded growth, changes in skin and hair pigmentation, diarrhea, loss of appetite, nervous irritability, lethargy, edema, anemia, fatty degeneration of the liver, necrosis, dermatoses and fibrosis, often accompanied by infection and multivitamin deficiencies. A- Is an abnormal retardation of growth and development of an infant resulting form condition that interfere with normal metabolism, appetite and activity. B- Is a condition of extreme malnutrition and emaciation, occurring chiefly in young children. It is characterized by progressive wasting of subcutaneous tissue and muscle. Marasmus results from lack of adequate calories and proteins and is seen in children with failure to thrive and individuals in a state of starvation. C- Is a condition resulting from a deficiency of or lack of absorption or use of one or more dietary vitamins. Source: Mosbys Pocket Dictionary of Medicine, Nursing and Allied Health 4th Edition, pp. 706, 478, 764, 478 Situation 3: These are laws related to devolution. 27. Which one of the following is the local government code? a. b. c. d. RA 4073 RA 3573 EO 119 RA 7160

Answer: D RA 7160 or commonly known as Local Governement Code. RA 4073 liberalizes the treatment of leprosy RA 3573 declares that all communicable diseases should be reported to the nearest health station. 28. Which laws cover Ethical Conduct of Public Officials? a. b. c. d. RA 7305 LOI 949 RA 6713 RA 6675
09394837323

For more nursing reviewers, contact ebookwholeseller@yahoo.com

38

Generated by Foxit PDF Creator Foxit Software http://www.foxitsoftware.com For evaluation only.

Answer: A RA 7305 is known as the Magna Carta for Public Health Workers. LOI 949 Legal basis for PHC RA 6713- Code of Conduct and Ethical Standards for Public Officials and Employees RA 6675- Generics Act of 1988 29. RA No. 7277 is otherwise known as: a. Magna Carta for Public Health workers. b. Magna carta for Disabled persons c. National Immunization Days d. Traditional and Alternative Health Care Answer: B. RA 7305 is Magna Carta for Public Health workers RA 8423 is Traditional and Alternative Health Care Source: Community Health Nursing Services in the Philippines, p. 354 30. It is an act requiring compulsory immunization against hepatitis B for infants and children below eight (8) years old. a. b. c. d. RA7846 RA 6365 RA 6758 RA 8749

Answer: A. RA 7846 is an act requiring compulsory immunization against hepatitis B for infants and children below eight (8) years old. Source: Community Health Nursing Services in the Philippines, p.100 RA 6365 established a National Policy on Population and created the Commission of Population. RA 6758 standardized the salaries of government employess which included the nursing personnel. RA 8749 is the Clean air Act. Approved in year 2000 but took effect on January of 2001. Source: Community Health Nursing Services in the Philippines, pp. 352-353 Situation 4: Luzviminda is a commercial sex worker in Hong Kong. She came home due to maculopapular rashes. Her diagnosis is HIV/AIDS 31. What is the causative agent in HIV/AIDS? a. b. c. d. Trichomonas vaginalis Human T cell Lymphotropic virus Treponema Pallidum Chlamydia trachomatis

Answer: B A- Trichomoniasis C- Syphilis D- Chlamydia Source: DOH ( Green and Yellow ) pp. 300

For more nursing reviewers, contact ebookwholeseller@yahoo.com

09394837323

39

Generated by Foxit PDF Creator Foxit Software http://www.foxitsoftware.com For evaluation only.

32. What is the mode of transmission in the case of Luzviminda with HIV/AIDS? a. b. c. d. Contaminated syringes Direct contact with contaminated fluids Blood transfusion Sexual contact

Answer: D Luzviminda is a commercial sex worker so she must have acquired it through sexual contact. Source: DOH ( Green and Yellow ) pp. 300 33. What is the confirmatory test for AIDS/HIV? a. b. c. d. Western Blot Sputum exam ELISA (+) DEXA

Answer: A B- confirmatory test for TB C- presumptive test D- diagnostic test for Osteoporosis Source: DOH ( Green and Yellow ) pp. 294

34. It is a chronic parasitic infection which greatly reduces human productivity and quality of life. It is frequently encountered in communities where eating of fresh or inadequately cooked crabs is a practice. a. b. c. d. STH Paragonimiasis PSP Hepa A

Answer: B. Paragonimiasis is a chronic parasitic infection which greatly reduces human productivity and quality of life. It is frequently encountered in communities where eating of fresh or inadequately cooked crabs is a practice. Source: Community Health Nursing Services in the Philippines, p. 277

35. The following are qualified for home delivery, except: a. b. c. d. full term previous cesarean section imminent deliveries adequate pelvis

Answer: B. The following are qualified for home delivery: a. full term b. less than 5 pregnancies c. cephalic presentation d. without existing diseases such as diabetes, bronchial asthma, heart diseases, hypertension, goiter, tuberculosis, severe anemia e. no history of complications like hemorrhage during previous deliveries f. no history of difficult delivery and prolonged labor
For more nursing reviewers, contact ebookwholeseller@yahoo.com 09394837323

40

Generated by Foxit PDF Creator Foxit Software http://www.foxitsoftware.com For evaluation only.

g. no previous cesarean section h. imminent deliveries i. in case of imminent deliveries by risk mothers, they should still be referred to the appropriate level of health facility if the risk remains after delivery but if the risk condition has disappeared then no referral is needed. j. No premature rupture of membranes k. Adequate pelvis l. Abdominal enlargement is appropriate for age of gestation Source: Community Health Nursing Services in the Philippines, pp.96-97

36. Mrs. Santos gave birth to a healthy baby boy via home delivery. Instruct member of the family to watch Mrs. Santos for hemorrhage for atleast how many hours just after the nurse or midwife has left the house after delivery? a. b. c. d. 2 3 4 1

Answer: A. Instruct member of the family to watch mother for hemorrhage for at least two hours just after the nurse or midwife has left the house after delivery. The first two hors after delivery are dangerous due to atony of the uterus. Source: Community Health Nursing Services in the Philippines, p.100 37. Which of the following is given to the pregnant woman? a. Chloroquine b. Iron c. iodized oil capsule d. all of the above Answer: D. Chloroquine (150 mg. base/ tablet), 2 tabs/week for the whole duration of pregnancy are given to all pregnant women in malaria infested areas. Iron, given from the 5th month of pregnancy up to 2 months post partum (100-200 mg. orally per day p.o for 210 days. Iodized oil, given once a year in goiter endemic areas. Source: CHN Services in the Phil. Dept of Health, 9 th ed., pp. 95-96

38. In order to increase survival of neonate tetanus patient, which of the following should you cover in your health education sessions with the mother? a. Go back to health center if infection develops at the site of tetanus toxoid immunization b. Need for prenatal visits c. To bring previously healthy babies for immediate consultation if they develop difficulty or inability to suck within the first 3 to 28 days of life d. Tetanus toxoid immunizations for pregnant mothers Answer: C Sign and symptom which can be used to suspect tetanus 1. History of normal suck and cry for the first 2 days of life 2. History of onset of illness between 3 and 28 days of life 3. History of inability to suck followed by stiffness and convulsions
For more nursing reviewers, contact ebookwholeseller@yahoo.com 09394837323

41

Generated by Foxit PDF Creator Foxit Software http://www.foxitsoftware.com For evaluation only.

4. Typical findings on physical examination by a qualified health worker: inability to suck (trimus) and / or stiffness, generalized muscle rigidity and / or convulsion (muscle spasm). Source: CHN by DOH page 299 39. How much Vitamin A should be given to the 6-11 months old infants who is experiencing Vit. A deficiency? a. 200 000 IU b. 400 000 IU c. 100 000 IU d. 50 000 IU Answer: C. For Vit. A deficiency, 100, 000 IU of Vit. A is given. Dosing is give today, give tomorrow, give after 2 weeks. Source: CHN Services in the Phil. Dept of Health, 9 th ed., p. 139

40. Micronutrient supplementation is included in what program of the DOH? a. Expanded Program on Immunization b. Reproductive health c. Araw ng Sangkap Pinoy d. Sentrong Sigla Answer: C. Araw ng Sangkap Pinoy. Source: CHN Services in the Phil. Dept of Health, 9 ed., p. 139 41. Expected results of Sentrong Sigla Movement for the individuals includes all of the following except: a. b. c. d. Adopt healthy lifestyle Demand for quality health services Develop systems for surveillance/ merits Promote well-being
th

Answer: C Expected Results of Sentrong Sigla Movement

For more nursing reviewers, contact ebookwholeseller@yahoo.com

09394837323

42

Generated by Foxit PDF Creator Foxit Software http://www.foxitsoftware.com For evaluation only.

Individuals will be empowered to: 1. Adopt healthy lifestyle 2. Demand for quality health services 3. Promote well-being 4. Improve healthy seeking behavior Institutions 1. Develop policies 2. Develop quality services for healthy providers 3. Develop system for surveillance/ merits 4. Advocate for laws

For more nursing reviewers, contact ebookwholeseller@yahoo.com

09394837323

43

Generated by Foxit PDF Creator Foxit Software http://www.foxitsoftware.com For evaluation only.

Source: Community Health Nursing Service in the Philippines page 127.

42. An expected result of Sentrong Sigla Movement for the institution includes which of the following? a. b. c. d. Adopt healthy lifestyle Demand for quality health services Promote well-being Develop quality services for healthy providers

Answer: D Expected Results of Sentrong Sigla Movement

For more nursing reviewers, contact ebookwholeseller@yahoo.com

09394837323

44

Generated by Foxit PDF Creator Foxit Software http://www.foxitsoftware.com For evaluation only.

Individuals will be empowered to: 1. Adopt healthy lifestyle 2. Demand for quality health services 3. Promote well-being 4. Improve healthy seeking behavior Institutions 1. develop policies 2. develop quality services for healthy providers 3. develop system for surveillance/ merits 4. advocate for laws Source: Community Health Nursing Service in the Philippines page 127. 43. EPI is based on 3 principles. Which one is pro-people? a. b. c. d. Based on epidemiological situation A basic health service Case finding and treatment on sites Mass approach rather than individuals

Answer: D Three principles: 1. The whole community rather than just the individual is to be protected thus mass approach is utilized 2. The program is based on epidemiological situation; schedules are drawn on the basis of occurrence and characteristic epidemiological features of the disease. 3. Immunization is a basic health service and such it is integrated into the health services being provided for by the RHU. Source: Community Health Nursing Services in the Philippines, p. 108 44. Which of the following is a primary element of EPI? a. b. c. d. Logistic management Target setting Information campaign Surveillance and Research

Answer: B Elements of EPI: 1. Target Setting 2. Cold chain logistic management 3. Information, education, and communication 4. Assessment and evaluation of the programs overall performance. 5. Surveillance, studies and research Source: Community Health Nursing Services in the Philippines, p. 109 45. Isolation techniques in the home are difficult to do but fundamental principles must be followed. Soiled articles with discharges should be boiled in water before laundry. How long should these articles be boiled? a. b. c. d. One hour Two hours One half day Thirty minutes
09394837323

For more nursing reviewers, contact ebookwholeseller@yahoo.com

45

Generated by Foxit PDF Creator Foxit Software http://www.foxitsoftware.com For evaluation only.

Answer: D Articles soiled with discharges should first be boiled in water 30 minutes before laundering. Those which could be burned should be burned. Source: DOH ( Blue and Yellow) pp. 62 46. Disinfection of water supply sources are required on the following except: a. newly constructed water supply facilities b. water supply facility that has been repaired/improved c. water supply sources found to be negative bacteriologically by laboratory analysis. d. Container disinfection of drinking water collected from a water facility that is subject to recontamination like open dug wells, unimproved springs and surface water. Answer: C. Disinfection of water supply sources is required on the following: a. newly constructed water supply facilities b. water supply facility that has been repaired/improved c. water supply sources found to be positive bacteriologically by laboratory analysis. d. Container disinfection of drinking water collected from a water facility that is subject to recontamination like open dug wells, unimproved springs and surface water. Source: Community Health Nursing Services in the Philippines, p. 316 47. The following are principles of Community Health Nursing except: a. Health teaching is the primary responsibility of the community health nurse b. The community health nurse utilizes the already existing active organized groups in the community c. The community health nurse participate in the development of an over-all health plan for the communities and in its implementation and evaluation d. The community health nurse must understand fully the objectives and policies of the agency she represents Answer: C To participate in the development of an over-all health plan for the community and in its implementation and evaluation, is one the objectives of Community Health Nursing. Option A, B, and D are principles of Community Health Nursing Source: Community Health Nursing Services in the Philippines, p. 18-19. 48. All of the following are complications of gonorrhea except one: a. Pelvic inflammatory disease b. sterility in both sexes c. arthritis d. meningocele Answer: D. Complications of gonorrhea: PID, sterility in both sexes, arthritis, blindness, menin gitis, heart damage, kidney damage, skin rash, ectopic pregnancy and eye damage in newborns (acquired from mothers vagina during childbirth). Source: Community Health Nursing Services in the Philippines, p. 299 49. The ten elements of reproductive health includes all of the following except:
For more nursing reviewers, contact ebookwholeseller@yahoo.com 09394837323

46

Generated by Foxit PDF Creator Foxit Software http://www.foxitsoftware.com For evaluation only.

a. b. c. d.

Maternal and child health and nutrition Mens reproductive health Violence against women Rehabilitation on reproductive health disorders

Answer: D Ten Elements of Reproductive Health a. Maternal and Child Health and Nutrition b. Family Planning c. Prevention and Management of Abortion Complications d. Prevention and Treatment of Reproductive Tract Infection e. Education and Counseling on Sexuality and Sexual Health f. Breast and Reproductive Tract Cancers and Other Gynecological conditions g. Mens Reproductive Health h. Adolescent Reproductive Health i. Violence Against Women j. Prevention and Treatment of Infertility and Sexual Disorder Source: Community Health Nursing Services in the Philippines, p. 86

50. Which of the following vitamins helps prevent arteriosclerosis; protect neuro-muscular system; important for normal immune function and is a strong anti-oxidant a. b. c. d. Vitamin K Vitamin C Vitamin D Vitamin E

Answer: D Vitamin C is for the formation of protein, collagen, bone, teeth, cartilage, skin and scar tissues. Vitamin D help in the mineralization of bones by enhancing absorption of calcium Vitamin K involves in the synthesis of blood clotting proteins and a bone protein that regulates calcium level. Source: Community Health Nursing Services in the Philippines, p. 135. 51. Which of the following is a mineral that is necessary for absorption and use of iron in the formation of hemoglobin a. b. c. d. Chromium Copper Calcium Chloride

Answer: B Chromium works with insulin and is required for release of energy from glucose Calcium helps in mineralization of bones and teeth, regulator of many of the bodys biochemical processes, involve in blood clotting, muscle contraction and relaxation, nerve functioning, blood pressure and immune defenses. Chloride maintains the normal fluid and electrolyte balance Source: Community Health Nursing Services in the Philippines, p. 136. 52. The community health nursing diagnosis is an important input to the formulation of a community health nursing program. In order to assure a successful implementation of the program, the diagnosis must be carried out in a manner where:
For more nursing reviewers, contact ebookwholeseller@yahoo.com 09394837323

47

Generated by Foxit PDF Creator Foxit Software http://www.foxitsoftware.com For evaluation only.

a. b. c. d.

An outside consultant determines what data to collect The nurse delegates the collection of data to the barangay health workers The community is directly involved in data collection and analysis. All the members of the Rural Health Unit participate in data collection.

Answer: C Community involvement is a vital component in the CHN principles: Considerations: 1. Alternatives or option in attaining the objective 2. Resources needed 3. Time table of activities 4. Night control 5. Evaluation scheme Source: CHN in the Phil. By Dizon page 166-167. 53. Preventive and Promotive health as a policy statement of the Department of Health (DOH) refers to the following except: a. The hospitals and other center for curative care are not required to integrate Promotive/ preventive health programs and health care delivery. b. Preventive and Promotive care will be the priority of the DOH and its partners in health c. Hospitals will also become centers of wellness d. The DOH will promote health and prevent disease and disability in work-sites, schools, industrial areas and commercial centers. Answer: A It is on the DOH mandate. The DOHs primary function is the promotion, protection, preservation restoration of the health of the people through the provision and delivery of health services and through the regulation and encouragement of providers of health goods and services. Source: CHN by DOH (Blue book) page 1 54. Following are initial steps to gain entry in Organizing a Community for Health Action 1. Gather initial information about the community from other members of the RHU or from records and reports 2. Make your courtesy calls 3. Prepare agenda for the first meeting 4. Arrange meeting with identified leaders, request barangay officials to sign for a meeting a. b. c. d. 1, 2, 3, 4 2, 4, 1, 3 3, 1, 4, 2 1, 2, 4, 3

Answer: D The following are initial steps to be done: 1. Gather initial information about the community from other members of the RHU or from records and reports. 2. List down names of persons to contact for a courtesy call 3. Arrange first meeting with identified key leaders, request barangay officials to sign invitation for a meeting 4. Prepare agenda for the first meeting. Source: CHN Services in the Phil. Dept of Health, 9th ed., p. 312.
For more nursing reviewers, contact ebookwholeseller@yahoo.com 09394837323

48

Generated by Foxit PDF Creator Foxit Software http://www.foxitsoftware.com For evaluation only.

55. Which of the following herb is used to treat ringworm? a. Guava leaves b. lagundi c. acapulco d. garlic Answer: C Acapulco is used to treat tinea flava, ringworm, athletes foot, and scabies. Lagundi is used to treat/ manage asthma, dysentery, dermatitis, eczema, etc. Guava could be utilized for washing wounds, diarrhea, and relief of toothache. Garlic is useful in lowering blood cholesterol and in managing toothache. Source: CHN by DOH, 9 Ed page 76-78 56. The DOH recommends 10 backyard plants to be used as medicines. Which is not included? a. niyug-niyogan b. bitter gourd c. ginger d. garlic Answer: C The 10 recommended plants are: lagundi, yerba Buena, sambong, tsaang gubat, niyug-niyogan, Acapulco, ulasimang bato, garlic, ampalaya (bitter gourd), and guava. Source: CHN by DOH 9th Ed 57. We consider a 2-year-old a fully immunized child when he was able to receive: a. 1 BCG, 2 DPT, 2 OPV, 3 Hepatitis, 1 measles b. 2 BCG, 3 DPT, 3 OPV, 3 Hepatitis, 1 measles c. 1 BCG, 3 DPT, 3 OPV, 3 Hepatitis, 1 measles d. 1 BCG, 3 DPT, 3 OPV, 3 Hepatitis, 2 measles Answer: C Source: CHN by DOH 9th Ed page 110-111 58. Which is a primary element of EPI? a. Target setting b. Assesment c. Surveillance d. Education Answer: A Elements of EPI includes target setting, cold chain logistic management, information, education and communication, Assessment and evaluation of programs overall performance; and surveillance, studies and research. Choices B, C, and D are incomplete Source: CHN by DOH 9 Ed 59. Which one is not a function of calcium? a. Bone and teeth mineralization b. Absorption of iron in the formation of Hgb c. Blood clotting d. muscle relaxation and contraction Answer: B
For more nursing reviewers, contact ebookwholeseller@yahoo.com 09394837323
th th

49

Generated by Foxit PDF Creator Foxit Software http://www.foxitsoftware.com For evaluation only.

Choice describes function of copper and /or Vitamin C Source: DOH ( Green and Yellow ) , pp. 136 60. A deficiency in protein leads to: a. Kwashiorkor b. Ricketts c. Beri-beri d. Hemorrhage Answer: A Ricketts results from Vitamin D deficiency. Beri-beri from Viamin B deficiency. Hemorrhage may result from Vitamin k deficiency. 61. What is the characteristic rash/ eruptions in chicken pox? a. Macular b. Petechial spots c. Maculopapular d. Vesiculopapular Answer: C Maculopapular rashes appear during the 1st 3 days follwed by vesicular rashes (not vesiculopapular). Petechial spots are minute hemorrhages. Chickenpox is an acute disease of sudden onset with slight fever, mild constitutional symptoms and eruption which are maculopapular for a few hours, vesicular for 3-4 days and leaves granular scabs. Not more than one day before and more than 6 days after appearance of first crop of vesicles. Source: DOH ( Green and Yellow ) , pp. 243 62. What is the period of communicability of chicken pox? a. 2 days before to 7 days after the appearance of vesicle b. 1 day before to 6 days after the appearance of vesicles c. 1 day before to 7 days after the appearance of vesicles d. 2 days before to 6 days after the appearance of vesicle Answer: B Source: DOH ( Green and Yellow ) , pp. 243

63. What is the primary source of infection of chicken pox? a. respiratory secretions of infected persons b. skin lesions c. scabs d. vesicular skin eruptions Answer: A Secretions from respiratory tract of the persons is the source of infection; lesions of skin are of little consequence. Scabs themselves are not infective. Source: CHN by DOH 9th Ed page 242 64. What are the 2 most common complications of chicken pox? a. Meningitis and TB
For more nursing reviewers, contact ebookwholeseller@yahoo.com 09394837323

50

Generated by Foxit PDF Creator Foxit Software http://www.foxitsoftware.com For evaluation only.

b. Pneumonia and encephalitis c. Pneumonia and endocarditis d. Encephalitis and endocarditis Answer: B Pneumonia and encephalitis are the most common although they occur only in severe cases of chicken pox. Source: CD by Bower, Pilant, and Craft page 107 65. Who are the persons in category 1 of TB? a. Previously treated patients in relapse b. New pulmonary TB cases whose sputum smear is (-) for three times c. New pulmonary TB paients whose sputum smear is (+) d. Extra-pulmonary but not acutely ill patients Answer: C Category I TB patients are those new TB cases with (+) sputum smear; and seriously ill pulmonary patients e.g. meningitis, TB carditis. Choice A describes Category II. Choice B and D describes Category III. Source: DOH ( Green and Yellow ) , pp. 208 66. How long is the intensive phase of the treatment regimen for category I patients? a. 2 months b. 3 months c. 4 months d. 6 months Answer: A 2 months. Choice D is the total length of treatment for Category I patients. 2 months intensive and 4 months maintenance. Source: DOH ( Green and Yellow ) , pp. 208 67. We have three levels of assessment. In the first level assessment, which among these problems is not a health threat? a. broken stairs b. strained marital relationship c. self medication d. illegitimacy Answer: D Illegitimacy is listed under foreseeable crisis. A, B and C are all health threats. Source: Nursing Practice in the Community 4 Ed., by Maglaya, pp. 68-70 68. In the Family service and Progress record, one item that should have an answer regarding our informant is marital status. What is the appropriate term given to a couple living together as husband and wife without the benefit of legal marriage? a. married b. live-in partner c. single d. common law
For more nursing reviewers, contact ebookwholeseller@yahoo.com 09394837323
th

51

Generated by Foxit PDF Creator Foxit Software http://www.foxitsoftware.com For evaluation only.

Answer: D Common law is the cohabitation of a couple even when it does not constitute a legal marriage. Married if legally married. Single if those who have never been married. Live- in partner is not an appropriate term. Source: Merriam Websters Collegiate Dictionary 10th Ed, pp. 232 69. What refers to the familys evaluation of the problem in terms of seriousness and urgency? a. Nature of the problem b. Modifiability of the problem c. Salience d. Preventive potential Answer: C A is categorized into health threat, health deficit and foreseeable crisis. B refers to the probability of minimizing or totally eradicating the problem. D refers to the nature or magnitude of the future problems that can be minimized or totally prevented if intervention is done on the problem. Source: Nursing Practice in the Community 4th Ed., by Maglaya, pp. 86 70. What is the minimum interval between doses of hepatitis vaccine? a. 3 weeks b. 2 weeks c. 6 weeks d. 4 weeks ANSWER: D Source: CHN by DOH, 9 th Ed. 71. At what age and route of administration is measles given? a. 9 months, IM b. 10 months, IM c. 9 months, ID d. 9 months, SQ ANSWER: D th Source: CHN by DOH, 9 Ed 72. When is the 2nd dose of BCG vaccine given? a. 1 month after the first dose b. Upon school entry c. 3 months after the first dose d. 12 years old ANSWER: B BCG immunization is given to school entrants both in public and private schools regardless of the presence or absence of a BCG scar. Source: CHN by DOH, 9 th Ed SITUATION 2: The 2000 Nutritional guidelines is formulated to improve the nutritional status of Filipinos. The following questions are concerned with nutrition.

73. Xeropthalmia is characterized by: a. Tunnel vision


For more nursing reviewers, contact ebookwholeseller@yahoo.com 09394837323

52

Generated by Foxit PDF Creator Foxit Software http://www.foxitsoftware.com For evaluation only.

b. Floaters c. Night blindness d. Window Vision ANSWER: C Xerpthalmia or night blindness results due to destruction of rods and cones. Tunnel vision is related to open angle glaucoma. Floaters occur in retinal detachment because of intraocular hemorrhage. Situation: The public health nurse participate in activities aimed towards the achievement of the goals of each and every program. 74. Hospital waste management program is a new requirement before construction of a facility. The hospital personnel required to train in waste management to prevent which of the following? a. Communicable diseases b. Nosocomial infection c. Cross infection d. Transmission of diseases Answer: B Policies have been set to prevent the risk of contracting nosocomial and other diseases ( diseases or illnesses that are acquired from staying in the hospital. Source: DOH ( Green and Yellow ) , pp. 319 75. Approved type of toilet facilities may need water or not depending on receiving space. What type of toilet is without need of water? a. Pit latrines b. Water sealed c. Flush toilet d. Aqua privies Answer: A Non water carriage toilet facility needs no water to wash into the receiving space. Examples are pit latrines and reed odorless earth closet. Source: DOH ( Green and Yellow ) , pp. 317 76. Disinfection of water supply sources is required on a newly constructed well, required water pipes, contaminated water supply and container disinfections collected from all except: a. Open wall b. Surface water c. River dam d. Unimproved spring Answer: C Disinfections of water supply sources are required on the following: 1. Container disinfection of drinking water collected from a water facility that is subject to recontamination like open dug wells, unimproved springs and surface water. 2. Newly constructed water supply 3. Water supply facility that has been repaired or improved 4. Water supply sources found to be positive bacteriologically by laboratory analysis. Source: DOH ( Green and Yellow ) , pp. 316 77. The nurse should know that the examination of drinking water by the government of non-government must be coordinated by the municipality through RHU. Certification of potability of an existing water source is issued by the: a. Sanitary engineer
For more nursing reviewers, contact ebookwholeseller@yahoo.com 09394837323

53

Generated by Foxit PDF Creator Foxit Software http://www.foxitsoftware.com For evaluation only.

b. Municipality c. Secretary of health or his representative d. DOH Answer: Certification of potability of an existing water source is issued by the Secretary of Health or his duly authorized representative . Source: DOH ( Green and Yellow) , pp. 316 78. Every municipality through its RHU must formulate an operational for quality monitoring and surveillance of their water supply every year using the areaprogram based approach. Assistance may be solicited from the internal planning Service in the collaboration with the: a. DOH b. Environmental Health Service c. Secretary of health d. Mayor Answer: B Every municipality through its RHU must formulate an operational for quality monitoring and surveillance of their water supply every year using the areaprogram based approach. Assistance may be solicited from the internal planning Service in the collaboration with the Environmental Health Service. Source: DOH ( Green and Yellow) , pp. 316 79. The Sentrong Sigla Movement (SSM) is a joint program of the Department of Health and the Local Government Units. What is the aim of this movement? a. Promote availability of quality health services in health centers and hospitals and make these accessible to every Filipino b. Certification and recognition program c. Benefits for local executions and health workers d. Foster better and more effective collaboration between DOH and LGU. Answer: A SSM aims to promote availability of quality health services in health centers and hospitals and to make these services accessible to every Filipino. B- this is the main component of the program D- objectives of SSM Source : DOH ( Green and Yellow), pp. 125 80. All of the following are drugs given to patients with Malaria except: a. Chloroquine b. Quinidine c. Sulfalene d. Biltricide Answer: D A, B and C are all recommended drugs for Maria. Source: DOH ( Green and Yellow ) , pp. 232 81. Guidelines no.2 in the Nutritional Guidelines for Filipinos is intended to promote exclusive breastfeeding: a. From birth to 4-6 months b. from birth to 2 year or longer c. from birth up to one year only d. From birth to 5 years
For more nursing reviewers, contact ebookwholeseller@yahoo.com 09394837323

54

Generated by Foxit PDF Creator Foxit Software http://www.foxitsoftware.com For evaluation only.

Answer: A Nutritional Guideline 2 states that breastfeed infants exclusively from birth to 4-6 months and then give appropriate food while continuing breasfeeding. Source: Source: DOH ( Green and Yellow ) , pp. 129 Situation: A home visit is a professional face to face contact made by a nurse to the client or his family. 82. Which of the following is the first step a nurse must do when conducting a home visit? a. Place PHN bag in convenient place before doing bag technique b. Greet client or household member and introduce yourself c. Explain purpose of visit d. Look into detailed aspects of the household Answer: B Source: DOH ( Green and Yellow) , pp. 53 83. A public health Nurse (PHN) bag is essential and indispensable when a nurse conducts a home visit. Which of the following is the vital principle in the use of the bag techniques? a. Bag when in communicable cases should be thoroughly cleaned and disinfected before keeping and using. b. Should minimize if not totally prevent spread of infection from individuals to families to the community. c. Arrangements of the contents is convenient to the user d. Should contain all necessary articles supplies and Equipment. Answer: B One of the principles of Bag Technique is that it should should minimize if not totally prevent spread of infection from individuals to families to the community. Source: DOH ( Green and Yellow) , pp. 54 84. Considering the steps and procedures in bag technique which side of the linen or paper lining of the PHN bag is clean to make a non-contaminated work field or areas? a. Beneath b. Outside c. Cover d. Inside Answer: D The paper lining must be cleaned side out or folded part out to make a non contaminated work field or area. Source: DOH ( Green and Yellow) , pp. 56 85. The following are the basic prenatal delivery service at BHS except: a. History taking b. Psychological examination c. Tetanus toxoid immunization d. Oral dental examination Answer: D The basic prenatal service delivery at the hospitals, RHU and BHS should include the following: History taking, physical examination, treatment of diseases, tetanus toxoid immunization, iron supplementation, health education, laboratory examination. Oral Dental Examination is not done in the RHU.
For more nursing reviewers, contact ebookwholeseller@yahoo.com 09394837323

55

Generated by Foxit PDF Creator Foxit Software http://www.foxitsoftware.com For evaluation only.

Source: DOH ( Green and Yellow) , pp. 95 Situation: Quality service is the aim of the DOH. Numerous programs have been devise to promote such goal. 86. It is the certification recognition program that develops and promotes standard for healthy facilities: a. Sentrong Sigla Movement b. Sang Milyong Sipilyo c. Reproductive health d. Expanded Program of Immunization (EPI) Answer: A SSMs main component is the certification recognition program that develops and promotes standard for healthy facilities. Source: DOH ( Green and Yellow) , pp. 125 87. Among are the pillars of Sentrong Sigla Movement except: a. Quality assurance b. Award c. International recognition d. All of the above Answer: C The 4 Pillars of SSM are the following: 1. Quality Assurance Pillar 2. Grants and Technical Assistance 3. Health Promotion 4. Award Pillar Source: DOH ( Green and Yellow) , pp. 126 88. All are the priorities of Sentrong Sigla Movement except: a. EPI b. Disease surveillance c. Family Planning d. Voluntary Blood Donation Answer: D A, B and C are the priorities of SSM. Source: DOH ( Green and Yellow) , pp. 128 89. All of the following are the standard requirements of Sentrong Sigla Movement except: a. Infrastructure b. Equipment c. Pharmaceuticals d. Herbal Medicine Answer: D The focus of SSMs standards and requirements will be inputs like basic infrastructure, equipment, pharmaceuticals, supplies and training that demonstrates preparedness or readiness of facilities to deliver quality services. Source: DOH ( Green and Yellow) , pp. 128 90. An expected result of SSM in every individual is to: a. Adapt healthy lifestyles
For more nursing reviewers, contact ebookwholeseller@yahoo.com 09394837323

56

Generated by Foxit PDF Creator Foxit Software http://www.foxitsoftware.com For evaluation only.

b. Develop policies c. Develop a system for surveillance d. Advocate law Answer: A B, C and D are expected results of SSM to institutions. Source: DOH ( Green and Yellow) , pp. 126 Situation: Reproductive Health (RH) is the exercise of reproduction right with responsibility. One of the goals of the reproductive health is to prevent illness/injuries related to sexuality and reproduction. 91. The following are goals of RH except: a. Every pregnancy should be intended b. Every birth should be healthy c. All married couple should use artificial contraceptive d. Achieve a desired family size Answer: C A, B and D are goals of RH. Source: DOH ( Green and Yellow) , pp. 84 92. In the international framework of RH, the focus is on: a. Past 40 years group age b. Womens health c. Displaced people with RH problems d. Barren couple Answer: B In the international framework, the focus is on womens health not only as a mother during her child bearing, but throuout life, from infancy to post reproductive health with full exercise of her reproductive life. Source: DOH ( Green and Yellow) , pp. 85 93. Which of the following is not an element of RH? a. Prevention and management of diseases b. Violence against women c. Self-employed d. Mens reproductive health Answer: C A, B and D are among the ten elements of RH. Source: DOH ( Green and Yellow) , pp. 86 94. In the Philippines, the focus of RH is on: a. Men and women b. Men only c. Women only d. Women and children Answer: B The focus of Philippine framework is the Reproductive Health Status in terms of its elements. It doesnt only address women but men and women. Source: DOH ( Green and Yellow) , pp. 86
For more nursing reviewers, contact ebookwholeseller@yahoo.com 09394837323

57

Generated by Foxit PDF Creator Foxit Software http://www.foxitsoftware.com For evaluation only.

95. What factor generally affects reproductive health in the international framework: a. Poverty b. Underemployment c. Environment d. Gender discrimination Answer: C Factors that may affect womens health are the general environment like poverty and under employment which could deter them from availing of socials services to the maximum, powerlessness or gender discrimination which could deprive women achieve full self development, because they cannot decide for themselves, so they are left behind. Source: DOH ( Green and Yellow) , pp. 85 96. In the immunization schedule, when is the BCG vaccine given? a. At birth or anytime after birth b. Anytime when mother requests c. At nine months together with measles d. At 6 weeks together with DPT Answer: A Source: DOH ( Green and Yellow) , pp. 110 97. An alternative for Mefenamic Acid for toothache is: a. Tsaang gubat b. Lagundi c. Sambong d. Bawang Answer: D For toothache: Pound a small piece and apply to affected part. Source: DOH ( Green and Yellow) , pp. 78 98. During home delivery, which concepts must be emphasized: a. Clean cord b. Clean hands c. Clean delivery surface d. All of the above Answer: D The 3 cleans namely: Clean hands, clean surface and clean cord should be strictly followed to prevent infection. Source: DOH ( Green and Yellow) , pp. 98 99. Preventive Measures done to malarial cases are the following except: a. Planting of Neem trees b. Using mosquito repellants c. Avoiding outdoor activities d. Treat snail breeding sites Answer: D Source: DOH ( Green and Yellow ), pp. 233 100. All of these are health deficits except: a. Blindness form measles b. Lameness from polio
For more nursing reviewers, contact ebookwholeseller@yahoo.com 09394837323

58

Generated by Foxit PDF Creator Foxit Software http://www.foxitsoftware.com For evaluation only.

c. Resettlement in a new community d. Aphasia after a CVA Answer: C Resettlement in a new community belongs to your foreseeable crisis. Source: Nursing Practice in the Community 4 Ed. By Maglaya, pp. 70
th

For more nursing reviewers, contact ebookwholeseller@yahoo.com

09394837323

59

Generated by Foxit PDF Creator Foxit Software http://www.foxitsoftware.com For evaluation only.

NURSING PRACTICE III 1. Which of the following would the nurse identify as an advantage to using a cervical cap for contraception? a. b. c. d. Provides continuous protection for 48 hours Is disposable and available over the counter Allows Spermicide application 2 hours before intercourse Minimizes risk for allergic reactions to plastic

Answer. A The cervical cap is a small rubber or plastic dome that fits snugly over the cervix. It provides continuous protection for 48 hours, no matter how many times intercourse occurs. Additional Spermicide is not necessary for repeated acts of intercourse. The cervical cap is not disposable or available over the counter, as is the female condom. A cervical cap must be fitted to the individual by a health care provider. There is risk for allergic reaction if the woman develops allergies to plastic, rubber or Spermicide. Source: Lippincotts Review Series Maternal and Newborn Nursing by Stright,3 edition p.80. 2. Which of the following statements by a male client would indicate that he understands the instructions for use of a condom? a. I should lubricate the condom with an oil-based product to avoid friction that could rupture the condom. b. I should unroll the condom and check it for holes before applying it. c. I should hold the rim of the condom while withdrawing my penis from the vagina to avoid leakage. d. I should begin sexual intercourse without the condom and don the condom just before ejaculation. Answer. C Oil-based lubricants can break down latex condoms. The condom should be unrolled onto the penis, starting at the tip of the penis. Holding the rim keeps the condom from slipping off the leaking semen into the vagina. Small amounts of semen are released before ejaculation and can result in pregnancy. Source: Prentice Hall, Review and Rationales Series for Nursing by Hogan, p.74 3. A woman using a diaphragm for contraception should be instructed to leave it in place for at least how long after intercourse? a. b. c. d. 1 hour 6 hours 12 hours 28 hours
rd

Answer. B The diaphragm should remain in place for at least 6 hours after intercourse but not longer than 12 hours to avoid the possibility of toxic shock syndrome. Source: Lippincotts Review Series Maternal and Newborn Nursing by Stright,3rd edition p.79.
For more nursing reviewers, contact ebookwholeseller@yahoo.com 09394837323

60

Generated by Foxit PDF Creator Foxit Software http://www.foxitsoftware.com For evaluation only.

4. The client has completed an at-home pregnancy test with positive results. Which of the following indicates that the client understands the meaning of the test results? a. b. c. d. I understand that this means I have ovulated in the past 24 hours. I understand that this means I am not pregnant. I understand that this means I might be pregnant. I understand that this means I am pregnant.

Answer. C A positive at-home pregnancy test indicates the presence of growing trophoblastic tissue and not necessarily a uterine pregnancy. Source: Prentice Hall, Review and Rationales Series for Nursing by Hogan, p.118 5. When describing to a client how a pregnancy test works, the nurse understands that which of the following hormones is being evaluated? a. b. c. d. Human chorionic gonadotropin Estrogen Follicle-Stimulating hormone Progesterone

Answer. A Human chorionic gonadotropin is the hormone present during a pregnancy and is the basis for the pregnancy test. Estrogen, follicle stimulating hormone, or progesterone are not the basis for pregnancy test. Source: Lippincotts Review Series, Maternal- Newborn Nursing, by Barbara R. Stright, 3rd edition, p 310. 6. A pregnant client asks about the function of the placenta. Which of the following should the nurse include in the teaching plan? a. b. c. d. The placenta filters fetal urine Fetal and maternal blood mix in the placenta to exchange nutrients The placenta filters alcohol from the mothers blood Substances are exchanged by the placenta without mixing maternal and fetal blood.

Answer. D Fetal gas exchange occurs in the intervillus spaces of the placenta through simple diffusion of oxygen, carbon dioxide and carbon monoxide. Substance exchange between the maternal and fetal blood occurs without mixing of the blood. Fetal products are excreted via the placenta, but urine is excreted by the fetus into the amniotic fluid. While the placenta is capable of filtering some substances, most substances consumed by the mother are exchanged with the fetus, including alcohol. Source: Prentice Hall, Review and Rationales Series for Nursing by Hogan, p.100. 7. A client is pregnant with twins, a boy and a girl, and she asks if they will be identical. The nurses best response is: a. b. c. d. They are not identical because the ultrasound showed one was bigger than the other. Ill discuss this with the doctor and give you a call later. We wont know until the babies are delivered. The twins are not identical. Identical twins are always the same sex.

Answer. D
For more nursing reviewers, contact ebookwholeseller@yahoo.com 09394837323

61

Generated by Foxit PDF Creator Foxit Software http://www.foxitsoftware.com For evaluation only.

Twins of opposite sex are at ways fraternal because it indicates two sperm were involved in fertilization, one carrying a Y chromosome and one carrying an X chromosome. Identical twins develop from one ovum and one sperm. Therefore, the genotype is the same, including sex. Identical twin s may be different sizes because one twin may receive a greater amount of placental circulation than the other. Source: Prentice Hall, Review and Rationales Series for Nursing by Hogan, p.100 8. Which of the following hormones stimulates the ovary to produce estrogen during the menstruation cycle? a. b. c. d. Follicle stimulating hormone (FSH) Gonadotropic releasing hormone (GnRH) Luteinizing hormone (LH) Human chorionic gonadotropin hormone (HCG)

Answer. A FSH is a pituitary hormone that stimulates the ovary to develop ovarian follicle that secrete estrogen. GnRH is a hormone released by the hypothalamus, which stimulates the anterior pituitary to secrete FSH and LH. LH is a hormone released by the anterior pituitary, which acts with FSH to cause ovulation and enhance development of the corpus luteum. HCG is a hormone secreted by the placenta, which stimulates the ovaries to produce estrogen and progesterone to maintain a healthy pregnancy. Source: Lippincotts Review Series, Maternal- Newborn Nursing, by Barbara R. Stright, 3rd edition, p.28 9. A 24-year old woman comes to the physicians office for a routine check-up at 34 weeks gestation. Abdominal palpation reveals the fetal position as right occipital anterior (ROA). To which of the following sites would the nurse expects to find the fetal heart tones. a. Below the umbilicus, on mothers left side. b. Below the umbilicus, on mothers right side. c. Above the umbilicus, on mothers left side. d. Above the umbilicus, on mothers right side. Answer. B Occiput and back are pressing against right side of mothers abdomen; FHT would be heard below umbilicus on the right side. C and D found in breech presentation. A is found on LOA. Source: NCLEX-RN International Edition by Kaplan, page 227 10. The client has come to the clinic because she suspects that she is pregnant. Which of the following would be the most definitive way to confirm the diagnosis? a. b. c. d. Clients report of amenorrhea for 3 months Positive Hegars sign Pigmentation changes of the breasts Palpation of fetal movement by the care provider.

Answer. D Palpation of the fetal movement is considered to be a completely objective sign of pregnancy that cannot have any other cause. The other signs listed here could have another etiology. Source: Prentice Hall, Review and Rationales Series for Nursing by Hogan, p.119.

11. The clients prenatal education includes danger signs to report. Which of the following, if reported, would indicate that the client understood the teaching?
For more nursing reviewers, contact ebookwholeseller@yahoo.com 09394837323

62

Generated by Foxit PDF Creator Foxit Software http://www.foxitsoftware.com For evaluation only.

a. b. c. d.

Dizziness and blurred vision Occasional nausea and vomiting No bowel movement for 3 days Ankle edema.

Answer. A Dizziness and blurred vision can be symptoms of pregnancy-induced hypertension, a complication which requires further assessment and medical management. Source: Prentice Hall, Review and Rationales Series for Nursing by Hogan, p.118

12. The nurse in the prenatal clinic is planning care for a pregnant 15-year old client. The nurse knows that this adolescent is at risk for which of the following maternal complications? a. Postpartum hemorrhage b. Hypoglycemia c. Cesarean birth d. Pre-eclampsia Answer: D Adolescents are at risk for pre-eclampsia. Postpartum hemorrhage is a complication of multiparty. Hypoglycemia is a complication of diabetes. Cesarean birth is a high risk factor for clients over 35 years of age. Source: www. Prenhall. Com. Maternal & child review series.

13. Which of the following nursing actions would take priority when caring for the woman with a suspected ectopic pregnancy? a. b. c. d. Administer oxygen Monitor vital signs Obtaining surgical consent Providing emotional support.

Answer. B The client with a suspected ectopic pregnancy may be at risk for the development of hypovolemic shock. Assessment is the first step of the nursing process and airway, breathing, and circulation are the priorities. Option A and D are possible later interventions, and option C is the surgeons responsibility. Source: Prentice Hall, Review and Rationales Series for Nursing by Hogan, p.164.

14. A client with pre-eclampsia is receiving magnesium sulfate and oxytocin (Pitocin) to induce labor at 38 weeks. What is the main indication of the magnesium sulfate for this client? a. b. c. d. Lower blood pressure Prevent convulsions Provide sedation Soften stools

Answer. B Magnesium sulfate is a CNS depressant used to prevent convulsions in the pre-eclamptic client. The other options may occur but are not the indication for the drug. Source: Prentice Hall, Review and Rationales Series for Nursing by Hogan, p.165.

For more nursing reviewers, contact ebookwholeseller@yahoo.com

09394837323

63

Generated by Foxit PDF Creator Foxit Software http://www.foxitsoftware.com For evaluation only.

15. The nurse is counseling a prenatal client regarding the need to take folic acid supplements during pregnancy. The nurse also encourages the client to eat foods high in folic acid, such as: a. b. c. d. Fruits and fruit juice Rice and Pasta Eggs and yogurt Fresh green leafy vegetables and legumes

Answer. D Fresh green leafy vegetables and legumes are good sources of folic acid. Fruits and fruit juice, rice and pasta, eggs, and yogurt are not sources of folic acid. Source: www. Prenhall. Com. Maternal & child review series. 16. On which of the following areas would the nurse expect to observe chloasma? a. b. c. d. Breast, areola, and nipple Chest, neck, arms, and legs Abdomen, breast, and thighs Cheeks, forehead, and nose

Answer D Chloasma, also called the mask of pregnancy, is an irregular hyperpigmented area found on the face. It is not seen on the breast, areola, nipple, chest, neck, arms, abdomen or thigh. Source: Lippincotts Review Series Maternal and Newborn Nursing by Stright, 3rd edition p.94. 17. The client is concerned about facial chloasma that has developed since her last prenatal visit. The best response by the nurse is: a. You should apply a facial skin bleach twice a day. b. Avoiding sun exposure may keep the pigmentation from getting any darker. c. This is a permanent condition caused by hormonal changes. You may be able to cover it with makeup. d. This is a condition associated with the development of skin cancer. I will make an appointment for you with a dermatologist. Answer. B Increased pigmentation during pregnancy is a response to increased estrogen levels. It can be worsened by the sun, is harmless, and generally fades after the pregnancy ends. Source: Prentice Hall, Review and Rationales Series for Nursing by Hogan, p.119

18. An antepartal client at 29 weeks gestation is assessed in the prenatal clinic. All assessment data are within normal limits. When should the nurse schedule the clients next appointment? a. b. c. d. In one week In 2 weeks In 3 weeks In 4 weeks

Answer. B The clients next appointment, if all assessment data are within normal limits, should be scheduled in 2 weeks. Weekly appointments are recommended after 36 weeks gestation. Every fourth week is the recommended interval for the first 28 weeks gestation. Source: www. Prenhall. Com. Maternal & child review series
For more nursing reviewers, contact ebookwholeseller@yahoo.com 09394837323

64

Generated by Foxit PDF Creator Foxit Software http://www.foxitsoftware.com For evaluation only.

19. When PROM occurs, which of the following provides evidence of the nurses understanding of the clients immediate needs? a. b. c. d. The chorion and amnion rupture 4 hours before the onset of labor. PROM removes the fetus most effective defense against infection. Nursing care is based on fetal viability and gestational age. PROM is associated with malpresentation and possibly incompetent cervix.

Answer. B PROM can precipitate many potential and actual problems; one of the serious is the fetus loss of an effective defense against infection. This is the clients most immediate need at this time. Typically, PROM occurs about 1 hour, not 4 hours, before labor begins. Fetal viability and gestational age are less immediate cervix may be causes of PROM. Source: Lippincotts Review Series, Maternal- Newborn Nursing, by Barbara R. Stright, 3rd edition, p 245. 20. A client who is 34 weeks gestation has been having contractions every 10 minutes regularly. In addition to instructing her to lie down and rest while continuing to time contractions, the nurse should also tell her to: a. b. c. d. Refrain from eating or drinking anything Take slow deep breathes with each contraction Go to the hospital if contractions continue for more than 1 hour Drink 3 to 4 cups of water.

Answer. D Hydration has been shown to decrease premature labor contractions. Therefore, drinking water or other non-caffeinated beverage is recommended. If contractions continue at 10 minutes apart or less for an hour with rest, the client should call her healthcare provider. 21. The nurse is caring for a laboring client with a known history of cocaine abuse. What complication is most likely for this client? a. b. c. d. Placenta previa Prolapsed cord Abruption placenta Polyhydramnios

Answer. C Abruptio placenta is the most likely complication for a client with a known history of cocaine abuse. The incidence of abruption placenta is approximately 1 to 100 births and occurs more frequently in pregnancies complicated by hypertension and cocaine abuse. Placenta previa may be a complication for women with multiple prior cesarean births. Prolapsed cord may be a complication with hydramnios, a small fetus, and a breech presentation. Polyhydramnios may be a complication of women with diabetes. Source: www. Prenhall. Com. Maternal & child review series. 22. When taking an obstetrical history on a pregnant client who states, I had a son born at 38 weeks gestation, a daughter born at 30 weeks gestation, and I lost a baby at about 8 weeks, the nurse should record her obstetrical history as which of the following? a. b. c. d. G2 T2 P0 A0 L2 G3 T1 P1 A0 L2 G3 T2 P0 A0 L2 G4 T1 P1 A1 L2
09394837323

For more nursing reviewers, contact ebookwholeseller@yahoo.com

65

Generated by Foxit PDF Creator Foxit Software http://www.foxitsoftware.com For evaluation only.

Answer. D The client has been pregnant four times, including current pregnancy (G). Birth at 38 weeks gestation is considered full term (T), while birth from 20 weeks to 38 weeks is considered preterm (P). A spontaneous abortion occurred at 8 weeks (A). She has two living children (L). Source: Lippincotts Review Series, Maternal- Newborn Nursing, by Barbara R. Stright, 3 edition, p 300. 23. A pregnant client states that she waddles when she walks. The nurses explanation is based on which of the following as the cause? a. b. c. d. The large size of the newborn Pressure on the pelvic muscles Relaxation of the pelvic joint Excessive weight gain
rd

Answer. C During pregnancy, hormonal changes cause relaxation of pelvic joint, resulting in the typical waddling gait. Changes in posture are related to the growing fetus. Pressure on the surrounding muscles causing discomfort is due to the growing uterus. Weight gain has no effect on gait. rd Source: Lippincotts Review Series Maternal and Newborn Nursing by Stright,3 edition p.94. 24. According to Rubin, during which of the following periods would the new mother frequently review her labor and delivery experience? a. b. c. d. Letting-down Letting-go Taking-hold Taking-in

Answer. D Rubin identifies three stages: taking-in, taking-hold, and letting-go. According to Rubin, during the takingin period, the new mother may review her labor and delivery experience frequently. Many mothers do experience a let-down feeling after giving birth related to the magnitude of birth experience and doubts about the ability to cope effectively with the demands of childrearing. However, Rubin does not describe a letting-down period. The letting-go stage, which generally occurs after the new mother returns home, is a time of family reorganization. During the taking-hold stage, the mother becomes concerned with her ability to parent successfully and accepts increasing responsibility for the newborn. Source: Lippincotts Review Series Maternal and Newborn Nursing by Stright,3rd edition p.168. 25. The nurse discovers a loop of the umbilical cord protruding through the vagina when preparing to perform a vaginal examination. The most appropriate intervention is to: a. b. c. d. Call the physician immediately Place a moist clean towel over the cord to prevent drying Immediately turn the client on her side and listen to the fetal heart rate. Perform the vaginal examination and apply upward digital pressure to the presenting part while having the mother assume a knee-chest position.

Answer. D Pressure on the cord must be relieved to save the life of the fetus. Applying upward manual pressure to the presenting part and having the mother assume a knee-chest position are appropriate emergency actions, followed by starting oxygen and calling the physician. Option B and C do nothing to relieve the cord occlusion. Source: Prentice Hall, Review and Rationales Series for Nursing by Hogan, p. 215
For more nursing reviewers, contact ebookwholeseller@yahoo.com 09394837323

66

Generated by Foxit PDF Creator Foxit Software http://www.foxitsoftware.com For evaluation only.

26. A nurse is planning to perform Leopolds maneuvers on a laboring client. What should be the nurses initial action? a. b. c. d. Position client in a supine position Have the client void Wash hands in warm water Apply sterile lubricant to the abdomen

Answer. B Having the client void before performing Leopolds maneuvers provides for improved comfort during the evaluation for the laboring client. Positioning the client on her back is the correct position, but this is not the initial action. The examiners hands should be warm, but this is not the initial action. Applying sterile lubricant to the abdomen is not part of the procedure. Source: www. Prenhall. Com. Maternal & child review series. 27. One hour after delivery, assessment reveals the clients uterus is one-finger breath below the umbilicus and deviated to the right of midline. Which of the following would be the nurses priority action at this time? a. b. c. d. Assist the mother to void Vigorously massage the fundus Administer additional oxytocin to contract the uterus Give a tocolytic drug intravenously

Answer. A A distended bladder will elevate and displace the uterus to the right. Therefore the nurse should assist the mother to void. A displaced uterus is usually caused by a full bladder. Vigorous massage of the fundus will not correct this and may cause unnecessary discomfort. Oxytocin would be used if the uterus was not contracting. There is no data to suggest a need for that at this time. A tocolytic would be used if the uterus required relaxation, such as in premature labor. Source: Lippincotts Review Series, Maternal- Newborn Nursing, by Barbara R. Stright, 3 edition, p 311. 28. The nurse is assessing the fundal height of a client at 26 weeks gestation. The nurse should expect the fundus to be: a. b. c. d. Level with the umbilicus Halfway between symphysis and umbilicus Slightly below ensiform cartilage At 26cm.
rd

Answer. D Fundal height in centimeters correlates well with weeks of gestation between 22-24 weeks and 34 weeks. Thus, at 26 weeks gestation, fundal height is probably about 26 cm. Source: www. Prenhall. Com. Maternal & child review series. 29. The plan of care for the pregnant client who experienced an unexplained intrauterine fetal demise during her last pregnancy should include: a. b. c. d. Education on the cause of intrauterine fetal demise given to both parents Encouragement to think positively and not dwell on the previous fetal loss Support for increased fears as this fetus reaches the gestational age of the previous fetal loss. Facilitation of grieving of the lost fetus through carrying a photo and a lock of hair at all times.
09394837323

For more nursing reviewers, contact ebookwholeseller@yahoo.com

67

Generated by Foxit PDF Creator Foxit Software http://www.foxitsoftware.com For evaluation only.

Answer. C Parents report increased stress around the time of the previous fetal loss during subsequent pregnancies. The nurse should ask open-ended questions to determine the parents stress level and grieving, and provide support as indicated. Source: Prentice Hall, Review and Rationales Series for Nursing by Hogan, p.326.

30. The nurse is evaluating an intrapartal clients lab results. Which laboratory finding should the nurse report to the physician or nurse-midwife? a. b. c. d. Hematocrit: 45% Leukocyte count: 19,000/mm Platelets: 120,000/mm White blood count: 11,000/mm

Answer. C The platelet (120,000/mm) should be reported as abnormally low, also called thrombocytopenia (normal: 250-500,000/mm). The hematocrit, leukocyte count, and white blood count are within normal limits for a laboring woman. Source: www. Prenhall. Com. Maternal & child review series. 31. The client has been having contractions every 5 minutes for 7 hours. Which factor is used to determine if this is true or false? a. b. c. d. The cervix is effacing and dilating This is the clients second baby The contractions are becoming more intense and lasting longer. The membranes have ruptured.

Answer. A The change in the cervix is the only indicator of true labor Source: Prentice Hall, Review and Rationales Series for Nursing by Hogan, p.184.

31. After 4 hours of active labor, the nurse notes that the contractions of a primigravid client are not strong enough to dilate the cervix. Which of the following would the nurse anticipate? a. b. c. d. Obtaining an order to begin IV oxytocin Administering a light sedative to allow the patient to rest for several hours Preparing for cesarean section for failure to progress Increasing the encouragement to the patient when pushing begins.

Answer. A The clients labor is hypotonic. The nurse should call the physician and obtain an order for an infusion of oxytocin, which will assist the uterus to contract more forcefully in an attempt to dilate the cervix. Administering a light sedative would be done for hypertonic uterine contractions. Preparing for cesarean section is unnecessary at this time. Oxytocin would increase the uterine co tractions and hopefully progress labor before a cesarean would be necessary. It is too early to anticipate client pushing with contraction. Source: Lippincotts Review Series, Maternal- Newborn Nursing, by Barbara R. Stright, 3rd edition, p 311. 32. During which of the following stages of labor would the nurse asses crowning?
For more nursing reviewers, contact ebookwholeseller@yahoo.com 09394837323

68

Generated by Foxit PDF Creator Foxit Software http://www.foxitsoftware.com For evaluation only.

a. b. c. d.

First stage Second stage Third stage Fourth stage

Answer. B Crowning, which occurs when the newborns head or presenting part appears at the vaginal opening, occurs during the second stage of labor. During the first stage of labor, cervical dilation and effacement occur. During the third stage of labor, the newborn and placenta are delivered. The fourth stage of labor last from 1 to 4 hours after birth, during which time the mother and newborn recover from the physical process of birth and the mothers organs undergo the initial readjustment to the non-pregnant state. Source: Lippincotts Review Series, Maternal- Newborn Nursing, by Barbara R. Stright, 3rd edition, p 152 33. The highest priority in nursing care of the laboring client is: a. b. c. d. Pain relief measures are offered that are acceptable to the client. The clients partner is involved with the labor and delivery Appropriate fluid intake is monitored Fetal response to the labor is assessed.

Answer. D The fetal heart rate response to contraction is a physiologic assessment that indicates the presence or absence of fetal well-being. The other options are appropriate for the laboring client, but safety of the fetus is the priority. Source: Prentice Hall, Review and Rationales Series for Nursing by Hogan, p.184. 34. Which of the following fundal heights indicates less than 12 weeks gestation when the date of the LMP is unknown? a. b. c. d. Uterus in the pelvis Uterus at the xiphoid Uterus in the abdomen Uterus at the umbilicus

Answer. A When the LMP is unknown, the gestational age of the fetus is estimated by uterine size or position (fundal height). The presence of the uterus in the pelvis indicates less than 12 weeks gestation. At approximately 12 to 14 weeks, the fundus is out of the pelvis above the syphysis pubis. The fundus is at the level of the umbilicus at approximately 20 weeks gestation and reaches the xiphoid at term or 40 weeks. Source: Lippincotts Review Series, Maternal- Newborn Nursing, by Barbara R. Stright, 3rd edition, p 131. 35. Which of the following danger signs should be reported promptly during the antepartum period? a. b. c. d. Constipation Breast tenderness Nasal stuffiness Leaking amniotic fluid

Answer. D Danger signs that require prompt reporting are leaking of amniotic fluid, vaginal bleeding, blurred vision, rapid weight gain, elevated blood pressure. Constipation, breast tenderness, and nasal stuffiness are common discomforts associated with pregnancy. Source: Lippincotts Review Series, Maternal- Newborn Nursing, by Barbara R. Stright, 3rd edition, p 131.
For more nursing reviewers, contact ebookwholeseller@yahoo.com 09394837323

69

Generated by Foxit PDF Creator Foxit Software http://www.foxitsoftware.com For evaluation only.

36. FHR can be auscultated with a fetoscope as early as which of the following? a. b. c. d. 5 weeks gestation 10 weeks gestation 15 weeks gestation 20 weeks gestation

Answer. D The FHR can be auscultated with the fetoscope at about 20 weeks gestation. FHR usually is auscultated at the midline suprapubic region with a Doppler ultrasound transducer at 10 to 12 weeks gestation. FHR cannot be heard any earlier than 10 weeks gestation. Source: Lippincotts Review Series, Maternal- Newborn Nursing, by Barbara R. Stright, 3 edition, p 130. 37. A client at 8 weeks gestation calls complaining of slight nausea in the morning hours. Which of the following client interventions should the nurse question? a. b. c. d. Taking 1 teaspoon of bicarbonate of soda in an 8-ounce glass of water Eating a few low-sodium crackers before getting out of bed Avoiding the intake of liquids in the morning hours Eating six small meals a day instead of three large meals
rd

Answer. A Using bicarbonate would increase the amount of sodium ingested, which can cause complications. Eating low-sodium crackers would be appropriate. Since liquids can increase nausea, avoiding them in the morning hours when nausea is usually the strongest is appropriate. Eating six small meals a day would keep the stomach full, which often decreases nausea. Source: Lippincotts Review Series, Maternal- Newborn Nursing, by Barbara R. Stright, 3 edition, p 309. 38. A client with severe pre-eclampsia is admitted with a BP 160/110, proteinuria, and severe pitting edema. Which of the following would be mot important to include in the clients plan of care? a. b. c. d. Daily weights Seizure precautions Right lateral positioning Stress reduction
rd

Answer. B Women hospitalized with severe pre-eclampsia need decreased CNS stimulation to prevent a seizure. Seizure precautions provide environmental safety should a seizure occur. Because of edema, daily weight is important but not the priority. Pre-eclampsia causes vasospasm and therefore can reduce uteroplacental perfusion. The client should be placed on her left side to maximize blood flow. Reduce blood pressure, and promote diuresis. Interventions to reduce stress and anxiety are very important to facilitate coping and a sense of control, but seizure precautions are the priority. Source: Lippincotts Review Series, Maternal- Newborn Nursing, by Barbara R. Stright, 3 edition, p 308. 39. The client is receiving intravenous magnesium sulfate at 2 g/h to stop premature labor. The most important nursing assessments of this client include: a. b. c. d. Intake and output, level of consciousness, and blood pressure. Blood pressure, pulse, and uterine activity Deep tendon reflex, hourly urine output, and respiratory rate Intake and output, blood pressure, and reflexes.
09394837323
rd

For more nursing reviewers, contact ebookwholeseller@yahoo.com

70

Generated by Foxit PDF Creator Foxit Software http://www.foxitsoftware.com For evaluation only.

Answer. C Early signs of magnesium toxicity that may lead to respiratory arrest are loss of patellar reflexes and decreased respiratory rate (<12/min). Since magnesium is excreted from the body through the renal system, hourly urine output should be assessed. Although blood pressure is a standard assessment for most antepartum clients, there is minimal blood pressure change, if any, associated with administration of magnesium sulfate. Source: Prentice Hall, Review and Rationales Series for Nursing by Hogan, p.215.

40. A clients amniotic fluid is greenish-tinged. The fetal presentation is vertex. Fetal heart rate (FHR) and uterine activity have remained within normal limits. At the time of delivery, the nurse should anticipate the need for: a. b. c. d. An infant laryngoscope and suction catheters Forceps A transport isolette Emergency cesarean set-up

Answer. A Meconium released by the fetus causes amniotic fluid to be greenish-tinged. Although the presence of meconium is associated with fetal distress, there is no evidence of immediate danger to the fetus during labor in this case. However, the infant is at risk for aspirating meconium at the time of delivery. Steps to prevent aspiration include thorough suctioning of the nasopharynx including visualization of the vocal cords to remove cords to remove meconium particles before the first breath. Source: Prentice Hall, Review and Rationales Series for Nursing by Hogan, p.213. 41. Following amniotomy, the most important nursing action is to: a. b. c. d. Reposition the mother on her left side Place a clean underpad on the bed Listen to fetal heart tones Observe the color and consistency of the amniotic fluid

Answer C The risk of umbilical cord compression or prolapse increases when amniotic fluid is released. Listening to fetal heart tones after amniotomy will quickly detect the presence of cord compression. Observing color and consistency of the fluid should be done next. Placing a clean under pad on the bed and repositioning the mother is important in providing comfort but is not the first priority. Source: Prentice Hall, Review and Rationales Series for Nursing by Hogan, p.213.

42. Assessment of a normal episiotomy immediately post delivery is most likely to reveal: a. b. c. d. Gaping between the sutures Slight bruising Pus coming from the sutures Edema that makes the tissue looks shiny.

Answer. B Moderate ecchymosis and edema are a normal response to the trauma of child birth, as well as to the presence of sutures. Sutures should be closely aligned without gaps and here should be no pus-like drainage indicating infection. Edema severe enough to cause the tissue to look shiny or taut is abnormal. Source: Prentice Hall, Review and Rationales Series for Nursing by Hogan, p.185.
For more nursing reviewers, contact ebookwholeseller@yahoo.com 09394837323

71

Generated by Foxit PDF Creator Foxit Software http://www.foxitsoftware.com For evaluation only.

43. If the fetal head is determined to be presenting in a position of complete extension, the nurse should anticipate a: a. b. c. d. Precipitous labor and delivery Prolonged labor and possible cesarean delivery Normal labor and spontaneous vaginal delivery Forceps-assisted vaginal delivery.

Answer. B The normal attitude of the fetal head is one of moderate flexion. Changes in fetal attitude, particularly the position of the head, present larger diameters to the maternal pelvis, which contributes to a prolonged and difficult labor and increases the likelihood of cesarean delivery. Source: Prentice Hall, Review and Rationales Series for Nursing by Hogan, p.186. 44. A nurse is caring for four postpartum clients who each have an order for Methergine (methylgonovine maleate). Based on the data collected during the nurses initial shift assessment, which client would not receive the medication? a. b. c. d. The client with a blood pressure of 156/94 The client with a hematocrit of 33% The client with a white blood cell count of 22,000 The client with a temperature of 101F

Answer. A Hypertension is a side effect of this medication; therefore, Methergine is contraindicated for women with high pressure. Elevated temperature and elevated blood count are not contraindications for administering Methergine. Because Methergine is given to prevent or reverse postpartum hemorrhage, it may also help prevent a decrease in hematocrit levels. Source: www. Prenhall. Com. Maternal & child review series. 45. A 22-year old woman is admitted to the hospital and delivers a healthy 7lbs 2 oz girl. The mother decides to bottle-feed her infant. Which of the following statements, if made by the mother after a teaching session, indicates to the nurse that the patient needs further teaching? a. b. c. d. Ill pump my breast and use warm packs to relieve breast pain. Ill use a tight bra and ice packs to relieve engorgement discomfort. Ill take the medication prescribed by the doctor for pain. Ill take the pills ordered by my doctor to help stop the production of milk.

Answer. A This stimulates hormonal responses thereby increasing production of milk causing engorgement. Options B, C, and d are all correct management for engorgement in mothers not breastfeeding. Ice packs relieve discomfort. Parlodel prescribed to prevent lactation. Source: NCLEX-RN International Edition by Kaplan ,page 172 46. The nurse in the postpartum unit cares for a 27-year old woman who delivered her first child the previous day. During her assessment of the patient, the nurse notes multiple varicosities on the patients lower extremities. The nurse should: a. b. c. d. Teach the patient to rest in bed when the baby sleeps Encourage early and frequent ambulation Apply warm soaks for 20 minutes every four hours perform passive range of motion exercises three times daily
09394837323

For more nursing reviewers, contact ebookwholeseller@yahoo.com

72

Generated by Foxit PDF Creator Foxit Software http://www.foxitsoftware.com For evaluation only.

Answer. B It facilitates emptying of blood vessel in the lower extremities. A is incorrect because bed rest can cause thrombophlebitis. C is not preventive but an intervention which needs physicians order. D is incorrect because early ambulation is more effective. Source: NCLEX-RN International Edition by Kaplan, page 100

47. It is most important for the nurse to have which drug readily available when the client is being treated with heparin therapy for thrombophlebitis? a. b. c. d. Calcium gluconate Aquamephyton Protamine sulfate Ferrous sulfate

Answer. C Protamine sulfate is a drug used to combat bleeding problems related to heparin overdose. Option A raises serum calcium levels. Option B. is the antidote for warfarin. Option D is an iron supplement. Source: Prentice Hall, Review and Rationales Series for Nursing by Hogan, p.259. 48. A priority intervention that the nurses do immediately after delivery is suctioning out the babys mouth and nares. Why? a. Suctioning decreases surface tension and prevents alveolar collapse. b. Suctioning assists with increasing the pulmonary vascular resistance in the lungs, resulting in a decrease in the blood flow to the pulmonary bed. c. Suctioning removes 80-110 ml of fluids that remain in the respiratory passages, permitting adequate movement of air. d. Suctioning is not necessary because the birth process squeezes all fluid out of the lungs Answer. C It is stated that 80-110 ml of fluid remains in the respiratory passages that must be removed to permit adequate movement of air. Surfactant decreases surface tension and prevents alveolar collapse. Although the initial chest recoil assists in clearing fluid from the airways and permits further inspiration, most clinicians believe mucus and fluid should be suctioned from the newborns mouth, nose, and throat. Suctioning does not increase the pulmonary vascular resistance. Source: www. Prenhall. Com. Maternal & child review series.

49. The nurse is admitting a neonate 2 hours after delivery. Which assessment data should the nurse be concerned about? a.Hands and feet blue b. Nasal flaring c.Minimal response to verbal stimulation d.Apical heart rate 156 Answer. B Nasal flaring could be a sign of respiratory distress and requires immediate intervention. The other assessment data are normal findings for a neonate at 2 hours of age. Source: Prentice Hall, Review and Rationales Series for Nursing by Hogan, p.310.

For more nursing reviewers, contact ebookwholeseller@yahoo.com

09394837323

73

Generated by Foxit PDF Creator Foxit Software http://www.foxitsoftware.com For evaluation only.

50. The nurse assesses the postpartum vaginal discharge (lochia) on four clients. Which of the following assessments would warrant notification of the physician? a. b. c. d. A dark red discharge on a 2-day postpartum client A pink to brownish discharge on a client who is 5 days postpartum Almost colorless to creamy discharge on a client 2 weeks after delivery A bright red discharge 5 days after delivery

Answer. D Any bright red vaginal discharge would be considered abnormal, but especially 5 days after delivery, when the lochia is typically pink to brownish. Lochia rubra, a dark red discharge, is present for 2 to 3 days after delivery. Bright red vaginal bleeding at this time suggest late postpartum hemorrhage, which occurs after the first 24 hours following delivery and is generally caused by retained placental fragments or bleeding disorders. Lochia rubra is the normal dark red discharge occurring in the first 2 and 3 days after delivery, containing epithelial cells, erythrocytes, leukocytes, and deciduas. Lochia serosa is a pink to brownish serosanguineous discharge occurring from 3 to 10 days after delivery that contains deciduas, erythrocytes, leukocytes, cervical mucus, and microorganisms. Lochia Alba is an almost colorless to yellowish discharge occurring from 10days to 3 weeks after delivery and containing leukocytes, deciduas, epithelial cells, fats, cervical mucus, cholesterol crystals and bacteria.

51. The nurse assesses the vital signs of the client, 4 hours postpartum that are as follows: BP 90/60; temperature 100.4F; pulse 100 weak, thready; R 20 per minute. Which of the following should the nurse do first? a. b. c. d. Report the temperature to the physician Recheck the blood pressure with another cuff Assess the uterus for firmness and position Determine the amount of lochia

Answer. D A weak, thready pulse elevated to 100BPM may indicate impending hemorrhagic shock. An increased pulse is a compensatory mechanism of the body in response to decreased fluid volume. Thus, the nurse should check the amount of lochia present. Temperatures up to 100.48F in the first 24 hours after birth are related to the dehydrating effects of labor and are considered normal. Although rechecking the blood pressure may be a correct choice of action, it is not the first action that should be implemented in light of the other data. The data indicate a potential impending hemorrhage. Assessing the uterus for firmness and position in relation to the umbilicus and midline is important, but the nurse should check the extent of vaginal bleeding first. Then it would be appropriate to check the uterus, which may be a possible cause of the hemorrhage. Source: Lippincotts Review Series, Maternal- Newborn Nursing, by Barbara R. Stright, 3rd edition, p 303. 52. When preparing to listen to the fetal heart rate at 12 weeks gestation, the nurse would use which of the following? b. c. d. e. Stethoscope placed midline at the umbilicus Doppler placed midline at the suprapubic region Fetoscope placed midway between the umbilicus and the xiphoid process External electronic fetal monitor placed at the umbilicus

Answer. B At 12 weeks gestation, the uterus rises out of the pelvis and is palpable above the symphysis pubis. The Doppler intensifies the sound of the fetal pulse rate so it is audible. The uterus has merely risen out of the pelvis into the abdominal cavity and is not at the level of umbilicus. The fetal heart rate at this age is not audible with a stethoscope. The uterus at 12 week is just above the symphysis pubis in the abdominal cavity, not midway between the umbilicus and xiphoid process. At 12weeks the FHR would be difficult to auscultate with a fetoscope. Although the external electronic fetal monitor would project the FHR, the uterus has not risen to the umbilicus at 12 weeks.
For more nursing reviewers, contact ebookwholeseller@yahoo.com 09394837323

74

Generated by Foxit PDF Creator Foxit Software http://www.foxitsoftware.com For evaluation only.

Source: Lippincotts Review Series, Maternal- Newborn Nursing, by Barbara R. Stright, 3rd edition, p 300.

53. Which of the following additional assessment findings would be most suspicious and lead the nurse to suspect postpartum blues in a client who is anxious and crying? a. b. c. d. Loss of appetite, constipation, abdominal pain Despondency, loss of appetite, difficulty sleeping Increased appetite, urinary retention, diarrhea Poor concentration, constipation, diarrhea

Answer. B Assessment findings most characteristic of postpartum blues includes crying, anxiety, despondency, loss of appetite, poor concentration and difficulty sleeping. Constipation, abdominal pain, increased appetite, urinary retention, and diarrhea may occur during the postpartum period, but these conditions are not suspicious for postpartum blues. Source: Lippincotts Review Series Maternal and Newborn Nursing by Stright,3rd edition p.168.

54. Which of the following would the nurse expect to find when assessing a client who delivered a newborn 12 hours ago? a. b. c. d. Lochia alba Soft boggy fundus Transient tachycardia Complaints of hunger

Answer. D Following delivery, the nurse would expect to find complaints of hunger and thirst. Additional assessment findings for this time period include lochia rubra; a fundus that is firm, located midline and at the level of the umbilicus or slightly lower; and transient bradycardia. Source: Lippincotts Review Series Maternal and Newborn Nursing by Stright,3 edition p.168. 55. Which of the following intervention results in convection heat loss in the newborn? a. b. c. d. Removal from an incubator for procedures Placing the newborn on a cold surface, such as scale Giving a bath Placing the isolette near a cold surface such as window or outside wall
rd

Answer. A Convection is defined as loss of heat from the warm body surface to the cooler air currents. The other options are examples of radiation, evaporation, and conduction. Source: www. Prenhall. Com. Maternal & child review series. 56. The initial respirations in the newborn are a result of which of the following? a. b. c. d. A rise in temperature. A change in pressure gradients Increased blood pH Decreased blood CO2 level

Answer B
For more nursing reviewers, contact ebookwholeseller@yahoo.com 09394837323

75

Generated by Foxit PDF Creator Foxit Software http://www.foxitsoftware.com For evaluation only.

Initial respirations are triggered by physical, sensory, and chemical factors. Physical factors include the change in pressure gradients. Sensory factors include a drop in temperature, noise, light and sound. Chemical factors include the decreased oxygen level, increased carbon dioxide level, and decreased pH as result of the transitory asphyxia that occurs during delivery. Source: Lippincotts Review Series Maternal and Newborn Nursing by Stright,3rd edition p.190. 57. The nurse determines that teaching about sudden infant death syndrome (SIDS) has been effective when the client states: a. b. c. d. No definite cause of death is found at autopsy. The cause is a brain malformation. Breast- feeding causes sudden infant death. Genetic disorders are the cause of SIDS.

Answer. A Autopsy rules out other causes of death, but in cases of SIDS, autopsy findings are normal. Source: Prentice Hall, Review and Rationales Series for Nursing by Hogan, p.326. 58. Which nursing diagnosis should be the highest priority when caring for a preterm newborn? a. b. c. d. Ineffective thermoregulation related to lack of subcutaneous fat Anticipatory grieving related to loss of perfect delivery Imbalanced nutrition related to immature digestive system Risk for injury related to thin epidermis

Answer. A Newborns compensate for hypothermia by metabolizing brown fat. This process requires glucose and oxygen. Preterm newborns are at risk for hypoglycemia and respiratory distress, so hypoglycemia can further increase their need s for oxygen and glucose and cause serious complications. The other diagnoses are appropriate but not the highest priority. Source: Prentice Hall, Review and Rationales Series for Nursing by Hogan, p.311. 59. The parents of a 28-week-gestation neonate ask the nurse, Why does he have to be fed through a tube in his mouth? The nurses best response is that: a. b. c. d. It allows an accurate assessment of intake The babys sucking, swallowing, and breathing are not coordinated yet The babys stomach cannot digest formula It helps prevent thrush

Answer. B Neonates generally arent able to effectively coordinate sucking, swallowing, and breathing until 34 to 36 weeks gestation. If fed orally before that time, they are at greater risk of aspiration. Typically they will be fed through a gavage tube until they are able to drink from a bottle- or breast-feed. Intake can be accurately assessed with oral and gavage feedings. The stomach of a preterm infant can digest small amounts of formula or breast milk. Thrush is an oral yeast infection commonly caused during passage through the birth canal, and gavage feeding will not prevent it from occurring. Source: Prentice Hall, Review and Rationales Series for Nursing by Hogan, p.310. 60. A mother is crying at babys bedside. The most therapeutic response by the nurse is: a. Dont worry. Everything will be fine. b. Why are you upset?
For more nursing reviewers, contact ebookwholeseller@yahoo.com 09394837323

76

Generated by Foxit PDF Creator Foxit Software http://www.foxitsoftware.com For evaluation only.

c. Would you like me to call the hospital chaplain? d. This must be hard for you. Answer. D Reflection allows the client to verbalize their feelings. The nurse should not give the client false hope. Clients often do not know why they feel the way they do, and it is not helpful to ask them to determine this. Some clients may find comfort in a religious leader, but care should be taken not to stereotype the clients religious beliefs. Source: Prentice Hall, Review and Rationales Series for Nursing by Hogan, p.311.

61. Which behavior observed by the nurse indicates good bottle-feeding technique? The mother: a. b. c. d. Keeps the nipple full of formula throughout the feeding Props the bottle on a rolled towel Points the bottle at the infants tongue Enlarges the nipple hole to allow for a steady stream of formula to flow

Answer. A Keeping the nipple full of formula prevents the infant from sucking air. Option B and D can cause aspiration of formula and option C could cause the infant to gag and vomit. Source: Prentice Hall, Review and Rationales Series for Nursing by Hogan, p.277 62. Which of the following criteria of gestational age must be assessed within 2 hours of birth for the results to be valid: a. b. c. d. Breast tissue Posture Soles of feet creases Scarf sign

Answer. C After 12 hours, the edemas of tissue present in most newborns begin to resolve and creases appear; these creases do not have the same predictive value as those assessed before resolution of newborn edema. All of the criteria in Options A, B, and D remain predictive beyond the first 12 hours after birth. Source: Prentice Hall, Review and Rationales Series for Nursing by Hogan, p.277 63. The nurse test the newborns Babinski reflex by: a. b. c. d. Touching the corner of the newborns mouth or cheek Changing the newborns equilibrium Placing a finger in the palm of the newborns hand Stroking the lateral aspect of the sole from the heel upward and across the ball of the foot.

Answer. D A Babinski reflex is elicited by stroking the lateral aspect of the sole of the heel (in the newborn) of fanning the toes and dorsiflexing the big toe is an indicator of fetal well-being. Touching the corner of the mouth or cheeks elicits the rooting reflex. Changing the newborns equilibrium elicits the Moro reflex. Placing a finger in the palm of the newborns hand elicits the Palmar grasp reflex. Source: Prentice Hall, Review and Rationales Series for Nursing by Hogan, p.277.

64. The nurse conducts a new parent support group for her community. Two mothers ask how their 8month-old children can be so different in height and weight? What is the appropriate response?
For more nursing reviewers, contact ebookwholeseller@yahoo.com 09394837323

77

Generated by Foxit PDF Creator Foxit Software http://www.foxitsoftware.com For evaluation only.

a. b. c. d.

This is an abnormality that should be referred to the physician. One of the children is displaying a "growth spurt. Rates of growth vary and individual differences occur for each child. The sequence of growth and development is unpredictable for each child.

Answer: C. Rates of growth vary and individual differences occur for each child. Although there are general norms for growth and development rates, each child is an individual who will progress at his or her own individual pace. Source: Reviews and Rationales Series for Nursing; Nursing and Child Care by Mary Ann Hogan and Judy E. White, p. 3 & 4 65. Children are usually brought to the clinic for health care by a parent. At what age is it appropriate for the nurse to question the child about presenting symptoms? a. b. c. d. 3 years 5 years 7 years 9 years

Answer: C 7 years By age 7, most children are able to clearly and in chronological order describe symptoms. Their vocabulary is extensive enough to have words to describe what they are feeling, time of onset, changes from the norm, etc. Source: Reviews and Rationales Series for Nursing; Nursing and Child Care by Mary Ann Hogan and Judy E. White, p. 19, 20 &37 66. When sharing the purpose of the Denver Development Screening Test (Denver II) with parents of an 18-month-old, the nurse should explain that: a. b. c. d. The Denver II is a test that will predict future intellectual ability. The Denver II is a screening test used to detect children who may be slow in development. The Denver II is used for early detection of speech disorders. The Denver II measures psychological, cognitive, and social development.

Answer: B The Denver II is a screening test used to detect children who may be slow in development. The Denver II is used to screen children for possible developmental delays in the areas of gross-motor skills, language, fine-motor skills, and personal-social development. Source: Reviews and Rationales Series for Nursing; Nursing and Child Care by Mary Ann Hogan and Judy E. White, p. 49-50 67. 4-year-old scores two failures on the Denver II. Which of the following statements is most accurate? a. b. c. d. The child is not as intelligent as expected for age and should be referred to a learning specialist. The child has a speech problem and should be referred to a speech therapist. The child is at risk for school problems and should be retested. The failures are to be expected in preschoolers who may not be cooperative with testing.

Answer: C The child is at risk for school problems and should be retested. The Denver II is a screening test, not a diagnostic test; therefore children who score a failure should be retested. The child is considered at-risk until other diagnostic indicators can determine a specific problem. Source: Reviews and Rationales Series for Nursing; Nursing and Child Care by Mary Ann Hogan and Judy E. White, p. 52 68. What is the most important sign of readiness to watch for when toilet training the child?
For more nursing reviewers, contact ebookwholeseller@yahoo.com 09394837323

78

Generated by Foxit PDF Creator Foxit Software http://www.foxitsoftware.com For evaluation only.

a. b. c. d.

ability to walk able to indicate that the diaper is wet physical and psychological readiness exhibits willingness to please parents

Answer: C physical and psychological readiness It is the childs welfare that should be the paramount consideration in toilet training. The physical and psychological readiness of the child will make the activity successful. Source: Wongs Essentials of Pediatric Nursing 6 edition, Dona L. Wong et. al., p. 419 69. The mother of a 12-month-old infant who is hospitalized is upset that she must leave her baby to go home for a short time. What should the nurse suggest to this concerned parent? a. b. c. d. Return as soon as possible to attend to her daughter's needs. Leave a personal article with the child and reassure her that she will return. Call a family relative to stay at all times with the child when the mother leaves. Ask a nurse to sit at the child's bedside in her absence.
th

Answer: B Leave a personal article with the child and reassure her that she will return. The goal is to preserve the child's trust. Strategies such as leaving a personal article, picture, or favorite toy help minimize the anxiety of separation. Source: Reviews and Rationales Series for Nursing; Nursing and Child Care by Mary Ann Hogan and Judy E. White, p. 18 70. Piaget identifies that the 2- to 7-year-old child is in a preoperational stage. The nurse observes a toddler take a toy from another. The nurse recognizes the child unable to put him- or herself in the place of another is displaying: a. b. c. d. Centration. Negativism. Egocentrism. Selfishness.

Answer: C Egocentrism The child in the preoperational stage is egocentric and is unable to see things from another's perspective. Logic is not well developed. Magical thinking is common. Centration is focusing on only one particular aspect of a situation. Negativism is a common toddler response of "no" to situations and requests. Selfishness is a negative behavior exhibited by the child who refuses to share with another. Source: Reviews and Rationales Series for Nursing; Nursing and Child Care by Mary Ann Hogan and Judy E. White, p. 16 71. The nurse is discussing STIs with a 17-year-old student. To correctly plan the teaching lesson, the nurse utilizes Piaget's theory to determine the adolescent's cognitive abilities. The educational plan should be based on the: a. b. c. d. Sensorimotor reactions. Limited cause and effect understanding. Concrete thinking. Mature abstract thinking

Answer: D Mature abstract thinking The adolescent is in the formal operational stage and is capable of mature, abstract thought. The nurse should give clear and complete information; mature thought leads to greater understanding. Source: Reviews and Rationales Series for Nursing; Nursing and Child Care by Mary Ann Hogan and Judy E. White, p. 17
For more nursing reviewers, contact ebookwholeseller@yahoo.com 09394837323

79

Generated by Foxit PDF Creator Foxit Software http://www.foxitsoftware.com For evaluation only.

72. The mother discusses with the nurse that her toddler asks every night for a bedtime story. The mother asks why the child does this. The nurse would explain that this behavior demonstrates: a. b. c. d. Ritualism. Object permanence. Dependency. Conservation.

Answer: A Ritualism. The toddler insists on sameness (such as a nightly bedtime story). Ritualism allows the toddler to have a sense of control, and to feel more secure and confident. The child may experience distress if this routine is not followed. Source: Reviews and Rationales Series for Nursing; Nursing and Child Care by Mary Ann Hogan and Judy E. White, p. 11 73. The nurse provides anticipatory guidance to parents of a 3-year-old child. Instructions should include: a. b. c. d. To restrain the child in the car seat facing rear in the back seat of the car. The use of syrup of ipecac for accidental poisonings. Drug and alcohol education. The proper use of sports equipment.

Answer: B The use of syrup of ipecac for accidental poisonings. Nurses are instrumental in teaching parents how to make the toddler's environment safe by providing instructions about keeping syrup of ipecac available, having the Poison Control Center number close to the phone, using child-resistant containers and cupboard safety closures, and keeping medicines and other poisonous materials locked away. Source: Reviews and Rationales Series for Nursing; Nursing and Child Care by Mary Ann Hogan and Judy E. White, p. 12 74. A teenager refuses to wear the clothes his mother bought for him. He states he wants to look like the other kids at school and wear clothes like they wear. The nurse explains this behavior is an example of teenage rebellion related to internal conflicts of: a. b. c. d. Autonomy vs. shame and doubt. Trust vs. mistrust. Identity vs. role confusion. Initiative vs. inferiority.

Answer: C Identity vs. role confusion. Erikson's theory of psychosocial development states that the child is faced with conflicts that need to be resolved. Erikson identifies stages of personality development. Identity vs. role confusion (12 to 19 years) is a period when adolescents search for answers regarding their future. During this time, the child rejects the identity presented by his parents and attempts to create his own identity. Identity is often based on peers. Source: Reviews and Rationales Series for Nursing; Nursing and Child Care by Mary Ann Hogan and Judy E. White, p. 17&18 75. Hospitalization of a child results in disturbance of the dynamics in family life. The most appropriate nursing diagnosis is: a. b. c. d. Diversional activity deficit related to separations from siblings and peers. Sleep patterns disturbance related to unfamiliar surroundings. Altered family processes related to hospitalization. Ineffective individual coping related to procedures.

Answer: C Altered family processes related to hospitalization.


For more nursing reviewers, contact ebookwholeseller@yahoo.com 09394837323

80

Generated by Foxit PDF Creator Foxit Software http://www.foxitsoftware.com For evaluation only.

Identification of nursing diagnoses that apply to the specific problem(s) of the child and family is an essential step of the nursing process. Family-centered care addresses the needs of the family members, including the child's siblings. The primary goals are to maintain the relationship with the child and siblings during the period of separation while hospitalized and avoid boredom and distress for the hospitalized child. Source: Reviews and Rationales Series for Nursing; Nursing and Child Care by Mary Ann Hogan and Judy E. White, p. 26 76. The charge nurse is developing plans to reduce the stress of hospitalized, chronically ill children. Coping for these children will be improved if: a. b. c. d. They are allowed 24-hour open visitation with their peers. They are assigned a primary nurse. They avoid making all decisions while hospitalized. All tutoring is postponed until discharge

Answer: B They are assigned a primary nurse. Primary nursing gives the child who is hospitalized frequently a sense of consistency. Care is provided by the same nurse(s), with whom the child develops trust and rapport. Source: Wongs Essentials of Pediatric Nursing 6 edition, Donna L. Wong, et. al., p 668 77. What should the nurse do first when preparing to do a physical assessment on a sleeping 8-monthold baby? a. b. c. d. Measure the occipital-frontal head circumference. Auscultate the heart and lungs. Check the eyes for the red reflex. Wake the baby
th

Answer: B Auscultate the heart and lungs. Auscultation is always easiest in a sleeping or quiet baby. Checking the eyes is considered invasive and should be saved for the end of the examination. There is no need to awaken the child because he or she will begin to stir once the examination begins. Source: Reviews and Rationales Series for Nursing; Nursing and Child Care by Mary Ann Hogan and Judy E. White, p. 39 & 54 78. The nurse is preparing an 8-year-old child for a procedure. What is the most appropriate nursing intervention? a. b. c. d. Provide visual aids, such as dolls, puppets, and diagrams in the explanation. Provide a written pamphlet for the child to review prior to the procedure. Discourage any display of emotional outbursts. Request that parents wait outside while the nurse provides instructions to the child.

Answer: A Provide visual aids, such as dolls, puppets, and diagrams in the explanation. Visual aids such as doll, puppets, and outlines of the body can be used to illustrate the cause and treatment of the child's illness. Use of such equipment provides information for the school-age child to understand and cope with feelings about the procedure. Written pamphlets should be given to the parents to review prior to the procedure. Children should be allowed to cry or verbalize their feelings without guilt as long as they hold still. Parents should be given a choice to accompany their child during the procedure Source: Reviews and Rationales Series for Nursing; Nursing and Child Care by Mary Ann Hogan and Judy E. White, p. 20 79. When assessing a child who complains of abdominal pain, what is the most appropriate nursing action?
For more nursing reviewers, contact ebookwholeseller@yahoo.com 09394837323

81

Generated by Foxit PDF Creator Foxit Software http://www.foxitsoftware.com For evaluation only.

a. b. c. d.

Palpate the most painful area first. Palpate for rebound tenderness. Avoid painful areas until the end of the assessment. Use deep palpation for abdominal tenderness.

Answer: C Avoid painful areas until the end of the assessment. Save the painful area for last to avoid abdominal guarding and to gain the child's trust. Always tell the child before touching a tender area. Source: Reviews and Rationales Series for Nursing; Nursing and Child Care by Mary Ann Hogan and Judy E. White, p. 54 80. When preparing to examine a preschool child, the nurse should: a. b. c. d. Give detailed explanations to alleviate the child's anxiety. Give reassurance and feedback to the child during the examination. Suggest that the child act like "the big kids" when he or she is examined. Say that the shirt is the only clothing that must be removed.

Answer: B Give reassurance and feedback to the child during the examination The preschooler may be somewhat anxious so the nurse should give feedback and reassurance about what will be done. Children do not need detailed explanations nor do they need to be told to act older than they are. Most children at this age are willing to remove clothing. Source: Reviews and Rationales Series for Nursing; Nursing and Child Care by Mary Ann Hogan and Judy E. White, p. 37 81. The pediatric nurse practitioner is working with a group developing school playgrounds. The playground designers must identify the major causes of potential injury for the school-aged child. The nurse explains that the most frequent accidents in school-age children involve: a. b. c. d. Motor vehicles, diving, and drugs and alcohol. Swing sets, drowning, and poisonings. Bicycles, skateboards, and in-line skates. Aspiration of food, plastic bags, and stairways.

Answer: C Bicycles, skateboards, and in-line skates. School-age children enjoy activities like skateboarding and biking that may cause injuries. Source: Reviews and Rationales Series for Nursing; Nursing and Child Care by Mary Ann Hogan and Judy E. White, p. 14 82. A father brings his 5-year-old to the doctor's office for a well-child visit. The father is embarrassed by his child's behavior during the visit. The father states that every time the child comes for an immunization she begins to cry and scream. An appropriate response to this father is: a. b. c. d. "All children have a major fear of needles; preschoolers often believe pain is a punishment. "Your child most likely had a traumatic experience at an early age. "Next time the mother should accompany the child for an immunization. "It is best to ignore this type of behavior as the child is seeking attention

Answer: A "All children have a major fear of needles, preschoolers often believe pain is a punishment. Preschoolers relate pain to an injury; they fear injections and do not believe an injection takes away pain. This is a normal response to cry and scream, kick and protest. Source: Reviews and Rationales Series for Nursing; Nursing and Child Care by Mary Ann Hogan and Judy E. White, p. 19 83. Whenever the parents of a 10-month-old leave their hospitalized child for short periods, he begins to cry and scream. The nurse explains that this behavior demonstrates that the child:
For more nursing reviewers, contact ebookwholeseller@yahoo.com 09394837323

82

Generated by Foxit PDF Creator Foxit Software http://www.foxitsoftware.com For evaluation only.

a. b. c. d.

Needs to remain with his parents at all times. Is experiencing separation anxiety. Is experiencing discomfort. Is extremely spoiled.

Answer: B Is experiencing separation anxiety. Infants and toddlers between the ages of 6 months and 30 months experience separation anxiety. Source: Reviews and Rationales Series for Nursing; Nursing and Child Care by Mary Ann Hogan and Judy E. White, p. 18 84. The mother of a 5-year-old expresses concern about her child who believes that "Grandma is still alive" 3 months after the grandmother's death. The nurse explains that: a. Magical thinking often accounts for a preschooler who believes that dead people will come back. b. There is a need for psychological counseling for this child and family. c. This is a form of regression exhibited by the preschooler. d. The child is in denial regarding Grandma's death. Answer: A Magical thinking often accounts for a preschooler who believes that dead people will come back. The preschooler believes that death is reversible. Their magical thinking and egocentricity often results in their belief that the deceased will come back to life. Preschoolers also often will blame themselves for the death of another Source: Reviews and Rationales Series for Nursing; Nursing and Child Care by Mary Ann Hogan and Judy E. White, p. 21 85. A mother asks the pediatric nurse about what she should begin to feed her 6-month-old infant. The correct response is: a. b. c. d. Egg whites are the least allergenic food to be introduced into the baby's diet. Rice cereal is the first solid introduced that is least allergenic of the cereals. Formula is the only source of nutrition given for the first year. Fruits and vegetables are good sources of iron.

Answer: B Rice cereal is the first solid introduced that is least allergenic of the cereals. Rice cereal is the first solid food because it is a rich source of iron and rarely induces allergic reactions. Source: Reviews and Rationales Series for Nursing; Nursing and Child Care by Mary Ann Hogan and Judy E. White, p. 7 86. The nurse would assess for which of the following as the most frequent cause of decreased hemoglobin and hematocrit levels in children a. b. c. d. Dietary deficiency Excess fluid intake Chronic blood loss Frequent cuts and bruises

Answer: A Dietary deficiency The major reason for low hemoglobin and hematocrit in infants and children is deficiency of iron intake through diet. Iron-fortified rice cereal is the first solid food recommended for infants beginning about 4 months of age as fetal iron stores are depleted. Source: Reviews and Rationales Series for Nursing; Nursing and Child Care by Mary Ann Hogan and Judy E. White, p. 370 87. A recently hospitalized 2-year-old client screams and shouts that he wants a "bottle." His parents are puzzled, and state that he has drank from a cup for the past year. The nurse explains that:
For more nursing reviewers, contact ebookwholeseller@yahoo.com 09394837323

83

Generated by Foxit PDF Creator Foxit Software http://www.foxitsoftware.com For evaluation only.

a. b. c. d.

Irritability is exhibited in all age groups. Temper tantrums often represent the child's need for parental attention. Various forms of punishment are necessary when such behaviors occur. Regression to an earlier behavior often helps the child cope with stress and anxiety.

Answer: D Regression to an earlier behavior often helps the child cope with stress and anxiety. Regression is common in toddlers; it lessens the threat of illness, hospitalization, or separation. A need to revert to use of the bottle, refusal to use the potty, or temper tantrums represent forms of behaviors exhibited as regression. Source: Reviews and Rationales Series for Nursing; Nursing and Child Care by Mary Ann Hogan and Judy E. White, p. 18 88. The nurse discusses dental care with the parents of a 3-year-old. The nurse explains that by the age of 3, their child should have: a. b. c. d. 5 "temporary" teeth. 10 "temporary" teeth. 15 "temporary" teeth. 20 "temporary" teeth.

Answer: D 20 "temporary" teeth. Children have 20 deciduous teeth that erupt between 6 months and 30 months of age. Source: Reviews and Rationales Series for Nursing; Nursing and Child Care by Mary Ann Hogan and Judy E. White, p. 12 89. When observing an 18-month-old child, the nurse notes a rounded belly, sway back, bowlegs, and slightly large head. The nursing conclusion is that: a. b. c. d. The child appears to be a normal toddler. The child is likely developmentally delayed. The child may be malnourished, especially with respect to calcium. The enlarged head is of great concern and requires a thorough neurological exam.

Answer: A The child appears to be a normal toddler. The typical toddler has lordosis and a protruding belly. The head still appears somewhat large in proportion to the rest of the body. Because these are normal findings, there is no need to be concerned about developmental delays, malnutrition, or neurological problems. Source: Reviews and Rationales Series for Nursing; Nursing and Child Care by Mary Ann Hogan and Judy E. White, p. 45 90. When using the otoscope to examine the ears of a 2-year-old child, the nurse should: a. b. c. d. Pull the pinna up and back. Pull the pinna down and back. Hold the pinna gently but firmly in its normal position. Hold the pinna against the skull.

Answer: B Pull the pinna down and back. The ear canal in infants and young children is shorter, wider, and more horizontally positioned than in older children. To adequately examine the tympanic membrane in young children the pinna must be pulled back and down Source: Reviews and Rationales Series for Nursing; Nursing and Child Care by Mary Ann Hogan and Judy E. White, p. 42 91. When assessing a 4-year-old child with a persistent cough, the nurse would assess respirations by observing which muscle group?
For more nursing reviewers, contact ebookwholeseller@yahoo.com 09394837323

84

Generated by Foxit PDF Creator Foxit Software http://www.foxitsoftware.com For evaluation only.

a. b. c. d.

Thoracic Abdominal Accessory Intercostal

Answer: B Abdominal Infants and young children use the diaphragm and abdominal muscles for respiration, so the nurse would watch the rise and fall of the abdomen to count respirations. Use of accessory or intercostal muscles may be observed in respiratory distress. Source: Reviews and Rationales Series for Nursing; Nursing and Child Care by Mary Ann Hogan and Judy E. White, p. 40 92. Screening for strabismus and amblyopia should be part of the physical assessment of which children? a. b. c. d. All children under 18 Infants Preschool children School-age children

Answer: C Preschool children Strabismus is detected with the cover-uncover test that can first be reliably administered to children over the age of 2. It is important to detect the problem early to prevent amblyopia. By school age, vision loss would have occurred. Source: Reviews and Rationales Series for Nursing; Nursing and Child Care by Mary Ann Hogan and Judy E. White, p. 55 93. At what age is it appropriate to change the sequence of the examination of the child from that of chest and thorax first to head-to-toe? a. b. c. d. Infant Toddler Preschool child School-age child

Answer: D School-age child The school-age years are the first time a child is able to reliably cooperate with the examiner and not squirm, talk, or otherwise interrupt the exam. Source: Reviews and Rationales Series for Nursing; Nursing and Child Care by Mary Ann Hogan and Judy E. White, p.37 94. To assess the height of an 18-month-old child who is brought to the clinic for routine examination, the nurse should: a. b. c. d. Measure arm span to estimate adult height. Use a tape measure. Use a horizontal measuring board. Have the child stand on an upright scale and use the measuring arm.

Answer: C Use a horizontal measuring board. Children younger than 2 or 3 should be measured lying down, preferably on a horizontal measuring board, to get an accurate assessment of height. A tape measure would be used to measure head circumference. An arm-span measure is not an appropriate estimation of adult height. Source: Reviews and Rationales Series for Nursing; Nursing and Child Care by Mary Ann Hogan and Judy E. White, p. 38
For more nursing reviewers, contact ebookwholeseller@yahoo.com 09394837323

85

Generated by Foxit PDF Creator Foxit Software http://www.foxitsoftware.com For evaluation only.

95. The nurse who is examining a child understands that visual acuity of 20/20 as measured by the Snellen chart is reached by age: a. 2 years. b. 4 years. c. 6 years. d. 8 years. Answer: C 6 years. While difficult to assess directly in infants and young children, visual acuity does not approach that of adults until school age or about 6 years. Source: Reviews and Rationales Series for Nursing; Nursing and Child Care by Mary Ann Hogan and Judy E. White, p. 42 96. A 1-year-old male child is scheduled for a routine exam at the pediatric clinic. The child's birth weight was 8 lbs. 2 oz. The child now weighs 18 pounds, 4 oz. The nurse knows that this weight is: a. b. c. d. Below the expected weight. Appropriate for the child's age. Above the expected weight. Individualized and thus unpredictable.

Answer: A Below the expected weight. The first year of life is one of rapid growth. The birth weight usually doubles by 6 months and triples by the end of the first year. Source: Reviews and Rationales Series for Nursing; Nursing and Child Care by Mary Ann Hogan and Judy E. White, p. 5 97. A 6-month-old child returns from surgery. PRN orders are available for pain management. The nurse would administer the pain medication when the baby is observed: a. b. c. d. Crying loudly, grimacing, restlessness. Displaying a change of color, decreased temperature. Demonstrating shortness of breath, lack of responsiveness. Sleeping more, refusing to eat.

Answer: A Crying loudly, grimacing, restlessness. A child's response to pain depends on his or her developmental stage. The infant is unable to describe or quantify pain because of limited vocabulary. Infant behaviors, such as crying, facial expressions, and change of activity are used to identify pain and distress. Source: Reviews and Rationales Series for Nursing; Nursing and Child Care by Mary Ann Hogan and Judy E. White, p. 18 98. A mother of a 4-year-old tells the nurse that her son is a "picky eater." The nurse should inform the mother that she should: a. b. c. d. Increase the amount of carbohydrates in the daily menu plan. Administer vitamins twice a day to her child. Be more concerned with the quantity of food than the quality of food. Recognize this is common for preschoolers as their caloric requirements have decreased slightly.

Answer: D Recognize this is common for preschoolers as their caloric requirements have decreased slightly. The preschooler will be influenced by others' eating habits and demonstrate their likes and dislikes for food preferences. The caloric requirement decreases slightly, to 90 kcal/kg/day. Quality, not quantity, is important. It is not necessary to give vitamins after infancy unless the child is at nutritional risk.

For more nursing reviewers, contact ebookwholeseller@yahoo.com

09394837323

86

Generated by Foxit PDF Creator Foxit Software http://www.foxitsoftware.com For evaluation only.

Source: Reviews and Rationales Series for Nursing; Nursing and Child Care by Mary Ann Hogan and Judy E. White, p. 12

99. When examining the child, the nurse should remember that tonsillar tissue: a. b. c. d. Enlarges until adolescence and then shrinks. Continues to enlarge throughout childhood and adolescence. Is readily visible in toddlers. Normally has a small amount of exudate.

Answer: A Enlarges until adolescence and then shrinks. Tonsils enlarge throughout childhood and gradually begin to shrink with puberty. Exudate should not be present on tonsils. Source: http://en.wikipedia.org/wiki/Tonsils 100. In discussing sexual maturation with a health class, the nurse would include the information that secondary sex characteristics begin to appear at: a. b. c. d. 10 years in girls, 12 years in boys. 12 years in girls, 16 years in boys. 10 years in boys, 10 years in girls. 12 years in girls and boys.

Answer: A 10 years in girls, 12 years in boys. Secondary sex characteristics begin at 10 to 12 years for girls and 12 to 14 years for boys. Source: http://en.wikipedia.org/wiki/Puberty

For more nursing reviewers, contact ebookwholeseller@yahoo.com

09394837323

87

Generated by Foxit PDF Creator Foxit Software http://www.foxitsoftware.com For evaluation only.

NURSING PRACTICE IV 1. A client with pulmonary edema is started on furosemide (Lasix). What would the nurse include in the discharge teaching? a. A decrease in urine output is to be expected. b. The client should eat foods with plenty of potassium. c. The client should expect an increase in swelling in the hands and feet. d. The client should take the medication at bedtime. Ratio: answer: B Furosemide (Lasix) is a loop diuretic that will increase urine output and decrease edema. Give furosemide early in the day so that the increased urination will not disturb the clients sleep. Arrange for a potassium rich diet or potassium supplements as needed due to the loss of potassium with the increased diuresis. 2. If airflow is obstructed while attempting to ventilate a victim during CPR, what should the rescuer do? a. Give two slow breathes followed by 15 chest compressions. b. Perform a finger sweep. c. Perform five chest compressions. d. Reposition the victims head and reattempt to ventilate. Ratio: answer: D If the victim cannot be ventilated the first time, reposition the head and try to ventilate again. If the victim cannot be ventilated after respositioning the head, the rescuer should proceed with maneuverse to remove any foreign bodies that may be obstructing the airway. 3. A client is wearing a nasal cannula. The flow rate is set at 2 L/min. The nurse understands the O2 concentration that the client is receiving is: a. 28% b. 45% c. 50% d. 60% Ratio: answer: A A flow rate of 2 L/min gives an O2 concentration of approximately 28%. Face masks will deliver O2 concentrations of 35-50% with flow rates of 6-12 L/min. A nonrebreathing mask, which delivers high concentrations of O2 and deliver O2 concentrations of 60-90%. 4. Analysis of arterial blood gasses (ABGs) and oxymetry are the best methods to assess which of the following? a. Acid-base balance. b. Adequate oxygenation. c. The efficiency of gas transfer in the lungs. d. Mixed venous gas sample.
For more nursing reviewers, contact ebookwholeseller@yahoo.com 09394837323

88

Generated by Foxit PDF Creator Foxit Software http://www.foxitsoftware.com For evaluation only.

Ratio: answer: C Two methods that are used to assess the efficiency of gas transfer in the lungs are analysis of ABGs and oxymetry. ABGs are used to measure acid-base balance,but oxymetry is not. An assessment of PaO 2 or SaO2 is usually sufficient to determine adequate oxygenation. Blood drawn from a pulmonary artery catheter is termed a mixed venous blood gas sample because it consists of venous blood that has returned to the heart from tissue beds and mixed in the right ventricle. 5. The nurse has just reviewed instructions for an oral glucose tolerance test (OGTT) with a client. Which of the following statements made by the client indicate a need for more teaching? a. I will eat a light breakfast the morning of the test. b. I will expect to take 100 mg of glucose at the start of the rest. c. I can expect to have my blood drawn at 30 and 60 minute intervals during the rest. d. I will report any symptoms of dizziness, sweating, and/or weakness if they occur during the test. Ratio: answer: A An oral glucose tolerance test (OGTT) is a fasting test and the client will be NPO after midnight prior to the test. All the other responses identify appropriate client responses regarding to the test. 6. A client presents with a diagnosis of hypopituitarism. When performing the history and physical exam, which of the following findings should the nurse anticipate? a. Increase cardiac output b. Truncal obesity c. Increase blood pressure d. Hyperactivity or increase energy levels Ratio: answer: B In hypopituitarism, there is decreased cardiac output, decreased blood pressure, and decrease energy level (fatigue). These symptoms occur due to an absence of hormones resulting from the decreases pituitary activity and truncal obesity is commonly associated with this disorder. 7. Following a hypophysectomy, the client complains of clear nasal drainage. What is the most appropriate initial action for the nurse? a. Notify the surgeon immediately b. Encourage the client to blow his nose to clear the sinuses c. Check the nasal drainage for glucose d. Place the client in Trendelenberg position Ratio: answer: C A cerebral spinal leak is suspected and testing the fluid for the presence of glucose would confirm this. Most leaks heal spontaneously, but occasionally surgical repair is needed. Packing the nose will not heal the leak at this site. The hedad of the bed should be elevated to decrease pressure on the graph site and blowing the nose is contraindicated. 8. Following a hypophysectomy, the nurse teaches the client to report which of the following? a. Cushings disease b. Graves disease c. Diabetes mellitus d. Hypopituitarism Ratio: answer: 4 After a hypophysectomy (surgical removal of the pituitary gland) there is a return to normal pituitary secretion. Hypopituitarism can cause a deficit of growth hormone, gonadotropins, thyroid stimulating hormones, and ACTH. The client needs to watch for changes in mental status, energy level, muscle strength, and cognitive function. Cushings disease is a disorder of hypersecretion. Graves disease is a hypersecretion of the thyroid gland. Diabetes mellitus is related to the function of the pancreas and is not related to the function of the pituitary.
For more nursing reviewers, contact ebookwholeseller@yahoo.com 09394837323

89

Generated by Foxit PDF Creator Foxit Software http://www.foxitsoftware.com For evaluation only.

9. Vasopressin (Pitressin) is ordered for the client with diabetes insipidus in order to do which of the following? a. Stimulate the pancreas to secrete insulin b. Slow the absorption of glucose in the intestine c. Increase reabsorption of water in the tubules. d. Increase blood pressure. Ratio: answer: C Vasopressin (Pitressin) is an antiduiretic hormone and is given to a client with diabetes insipidus to increase urine concentration by increasing the tubular reabsorption of water. Vasopressin does not increase blood pressure or affect either insulin production or intestinal absorption of glucose. 10. Dietary management of the client with Addisons disease includes which of the following? a. High protein, high calcium, low calorie, high nutrition. b. Low protein, high calcium, low calorie, high nutrition c. Low protein, high calcium, high calorie, high nutrition d. Low protein, low calcium, high calorie, high nutrition Ratio: answer: A Excess corticoids in the individual with Addisons disease contribute to weight gain and calcium and protein loss. So the recommended diet for these individuals is one of high protein and calcium intake while maintaining lower caloric intake to prevent weight gain. 10. The client with Addisons disease is ordered glucocorticoid therapy. Which of the following statements indicates hat the client has a correct understanding of the medication regimen? a. Dosage adjustments in my medication dosages may be needed. b. On days I feel good, I will not need to take the medication. c. I will adjust my dosages based on my home blood glucose test results. d. I am on an every-other-day dosing regimen. Ratio: answer: A Glucocorticoid medication therapy is established with a basal dose. The typical regimen begins with twothirds of the daily dose taken in the morning (8 AM) and the remaining one-third later (4 PM) in the day. This regimen closely resembles the diurnal pattern of secretion. Glucocorotoid medications do not have a cumulative effect and must be taken daily. Glucocorticoid needs fluctuate according to daily life and/or stressors. 11. Which of the following lab results would be typical of the client with Addisons disease? a. Blood urine nitrogen (BUN) of 3.5 mg/dl. b. Sodium (NA) of 185 mEq/L. c. Fasting blood glucose (FBS) of 55 mg/dl. d. Potassium (K) of 2.7 mEq/L Ratio: answer: C Decreased heptic glucosneogenesis and increased glucose uptake in the tissue cause hypoglycemia, not hyperglycemia. Elevated glucose is associated with cortisol excesss, as in Cushings disease. Hyperkalemia and hyponatremia are characteristic of Addisons disease. There is decreased renal perfusion and excretion of waste products, which cause an elevated BUN. 12. The client with Addisons disease may present with which of the following signs and symptoms? a. Muscle spasms b. Hunger. c. Fatigue and emotional labiality d. Weight gain.
For more nursing reviewers, contact ebookwholeseller@yahoo.com 09394837323

90

Generated by Foxit PDF Creator Foxit Software http://www.foxitsoftware.com For evaluation only.

Ratio: answer: C With adrenocortical insufficiency, muscle weakness, fatigue, nausea, and vomiting, irritability and mood changes are all signs and symptoms tat occur. The other options listed are not symptoms of Addisons disease. 13. A client presents with a diagnosis of Cushings disease. Physical assessment by the nurse reveals which of the following findings? a. Bruised areas on the skin b. Postural hypotension c. Weight loss. d. Decreased body hair. Ratio: answer: A In Cushings disease, skin bruising occurs caused by hypersecretion of glucocorticoids. Fluid retention causes hypertension. Hair on the head thins, while body hair increases. Weight gain also occurs. 14. After pituitary surgery, the nurse should carefully assess the client and report which of the following findings immediately? a. A urine test positive for glucose and ketones. b. A blood glucose level greater than 450 mg/dl. c. Urine output of 1-2 liters/day d. Urine specific gravity less than 1.010 Ratio: answer: D Urine test that are positive for glucose and ketones as well as BG levels less than 450 mg/dl are diagnostic for diabetes mellitus rather than diabetes insipidus. A urinary output of 1-2 liters is a normal daily output. Polyuria is a manifestation of diabetes insipidus. In diabetes insipidus, there is a lack of antidiuretic hormone (ADH), which causes insufficient water reabsorption in the kidneys. These causes polyuria and results in decreased urine specific gravity (1.001-1.010). The client may consume and excrete 5-40 L of fluid a day. 15. Which of the following lab results are consistent with a diagnosis of Cushings disease? a. A two hour postprandial blood glucose of 40 mg/dl b. A urinary calcium level of 6.2 mg/dl. c. A potassium (K) level of 2.7 mEq/L d. A sodium (Na) level of 128 mEq/L Ratio: answer: D Clients with Cushings disease have hypertremia, not hyponatremia, and this sodium retention is typically accompanied by potassium depletion. Bone reabsorption of calcium increases the urine calcium level. The secretion of aldosterone results in hypertension, hypokalema, and edema. In addition, hyperglycemia rather than hypoglycemia is seen due to alteration in glucose metabolism. 16. A client with chronic lymphocytic leukemia has a central venous access device. She has a tunneled central catheter called a Hickman. The nurse knows that this catheter is inserted surgically and threaded to the subclavian, and then is advanced into the superior vena cava just above the junction with the a. b. c. d. left atrium right atrium left ventricle right ventricle

Rationale: The right atrium is the correct answer. The superior vena cava brings deoxygenated blood to the right atrium. The central venous catheter is threaded into the superior vena cava approximately 2 to 3 cm above the junction with the right atrium.
For more nursing reviewers, contact ebookwholeseller@yahoo.com 09394837323

91

Generated by Foxit PDF Creator Foxit Software http://www.foxitsoftware.com For evaluation only.

17. A 2 day old neonate is receiving phototherapy for hyperbilirubinemia. During this therapy, it is essential that the nurse: a. b. c. d. decrease the units level of light every 2 hours keep the neonate well covered to prevent cold stress maintain the neonates position for 4 hours at a time place an opaque mask over the neonates eyes

Rationale: Correct answer: D An opaque mask is placed over the neonates eyes to prevent retinal damage from the lights. The mask should be removed for 2-5 minutes every 8 hours to assess for irritation or redness. 18. In performing CPR, the primary goal of the nurse is to a. b. c. d. return the heart to normal rhythm maintain acid-base balance maintain circulation to vital organs correct fluid volume deficit

Rationale: Correct answer: C The goal of CPR is to maintain circulation to vital organs until more advanced forms of life support can be initiated. 19. A client with a peripheral IV line is about to receive a blood transfusion of packed red blood cells due to anemia. The nurse administering the transfusion will a. b. c. d. infuse the blood slowly over a period of 5 to 6 hours initially infuse at a rapid rate and check pulse frequently obtain the clients vital signs prior to the transfusion prime the tubing with a D5W solution prior to infusing blood

Rationale: Correct answer: C The nurse must obtain baseline vital signs for this client just prior to starting the transfusion. Then the nurse will continue to monitor his vital signs as per protocol to evaluate for signs of a transfusion reaction. 20. The nurse is caring for an adult admitted to the coronary care unit with a myocardial infarction. During the second night in the CCU, the client develops congestive heart failure. A Swan-Ganz catheter is inserted to monitor the client for left ventricular function because a. b. c. d. It provides information about pulmonary resistance. It measures myocardial oxygen consumption. It controls renal blood flow. It controls afterload.

Rationale: Correct answer: A The Swan-Ganz catheter measures pulmonary artery and capillary wedge pressures, which are good indicators of pulmonary pathology. The Swan-Ganz catheter does not measure myocardial oxygen consumption and does not control renal blood flow. 21. Mrs. J. is admitted to the cardiac care unit with a myocardial infraction. The morning after admission she and her husband tell the nurse that she must be home tonight to care for the children when Mr. J. goes to work. The problem identified at this point would be a. Anxiety related to physical limitations.
09394837323

For more nursing reviewers, contact ebookwholeseller@yahoo.com

92

Generated by Foxit PDF Creator Foxit Software http://www.foxitsoftware.com For evaluation only.

b. c. d.

Alteration in cardiac output. Inability of client/family to understand disease process. Safety needs related to inability to cope.

Rationale: Correct answer: C The nurse should assess both Mr. and Mrs. J.s understanding of the disease and rehabilitation processes. They both exhibit the need for information in order to be able to make rational decisions. 22. Ms. H. is admitted to the coronary care unit to rule out a myocardial infarction. She tells the nurse she is sure it is just angina and cannot understand what the difference is between angina and infarct pain. Which response is most appropriate for the nurse to make? a. b. c. d. Anginal pain usually only lasts 3-5 minutes. Anginal pain produces clenching of the fists over the chest while acute MI pain does not. Anginal pain requires morphine for relief. Anginal pain radiates to the left arm while acute MI pain does not.

Rationale: Correct answer: A Anginal pain is of short duration. It is usually relieved by rest. The usual treatment for anginal pain is nitroglycerin. Anginal pain and the pain of an acute MI can both radiate to other locations 23. Which assessment finding in the elderly is caused by decreased vessel elasticity and increased peripheral resistance? a. b. c. d. Confusion An erratic pulse rate An increase in blood pressure Wide QRS complexes on the ECG

Rationale: Correct answer: C The blood pressure increases in response to the thickening of vessels and less-distensible arteries and veins. There is also an impedance to blood flow and increased systemic vascular resistance, contributing to hypertension. Confusion could be caused by a decreased oxygenation to the brain or by the interaction of multiple medications. An erratic pulse is not caused by decreased vessel elasticity and increased peripheral vascular resistance. An erratic pulse could be a sign of cardiac disease, a side effect of a prescribed medication, or a sign of the interaction of multiple medications. A wide QRS complex on an ECG is present in arrhythmias arising from the ventricles or in the presence of conduction defects of the ventricles. 24. A nurse is caring for a client during the recovery phase following a myocardial infarction. A cardiac catheterization, using the femoral artery approach, is performed to assess the degree of coronary artery thrombosis. Which nursing action following the procedure is unsafe for the client? a. b. c. d. Placing the clients bed in the Fowlers position Encouraging the client to increase fluid intake Instructing the client to move the toes when checking circulation Resuming prescribed pre-catheterization medications

Rationale: Correct answer: A Immediately following a cardiac catheterization with femoral artery approach, the client should not flex or hyperextend the affected leg to avoid blood vessel occlusion or hemorrhage. Fluids are encouraged to assist in removing the contrast medium from the body. Asking the client to move their toes assess motion, which could be impaired if a hematoma or thrombus were developing. The pre-catheterization medications are needed to treat acute and chronic conditions. [Some
For more nursing reviewers, contact ebookwholeseller@yahoo.com 09394837323

93

Generated by Foxit PDF Creator Foxit Software http://www.foxitsoftware.com For evaluation only.

facilities may require the MD to reorder all pre-procedural medications. Check your facility policy & procedures.] Keywords for this question are unsafe and femoral artery approach. 25. A nurse admits a client transferred from the emergency room. The client, diagnosed with a myocardial infarction, is complaining of substernal chest pain, diaphoresis and nausea. The first action by the nurse should be a. b. c. d. Order an EKG Administer morphine sulphate Start an IV Measure vital signs

Ratio: The correct answer is: B Decreasing the clients pain is the most important priority at this time. As long as pain is present there is danger in extending the infarcted area. Morphine will decrease the oxygen demands of the heart and act as a mild diuretic as well. Smeltzer, S.C. and Bare, B.G. (2004). Medical surgical nursing. (10th edition). Philadelphia, PA. Lippincott Williams & Wilkins. Wilson, B.A., Shannon, M.T., and Stang, C.L. (2004). Nurses drug guide. Upper Saddle River, New Jersey: Pearson Prentice Hall. 26. A client is getting ready to go home after having a myocardial infarction (MI).The client is is asking questions about his medications, and wants to know why metoprenolol (Lopressor) was prescribed. The nurses best response would be which of the following? a. Your heart was bearing too slowly, and Lopressor increase your heart rate. b. Lopressor helps to increase the blood supply to the heart by dilating your coronary arteries. c. This medication helps make your heart beta stronger to supply more blood to your body. d. It slows your heart rate and decreases the amount of work it has to do so it can heal. Ratio: correct answer: D Metropolol (Lopressor) is a beta blocker, and it slows heart rate; the main therapeutic effect after a MI is to reduce cardiac workload. It does not dilate the coronary arteries, and it actually decreases the contractility (strength of the heart beat). 27. A client is taking digoxin (Lanoxin) and furosemide (Lasix) for heart failure. Which of the following would be the best menu choices for this client? a. Chicken with baked potato and cantaloupe b. Eggs and ham c. Grilled cheese sandwhich and French fried potatoes d. Pizza with pepperoni Ratio: correct answer: A A prudent diet would be high in potassium because digoxin and furosemide can both deplete potassium. The diet needs to be low in sodium to prevent additional fluid overload with heart failure. Chicken, potato, and cantaloupe are all potassium-rich foods; options 2,3, and 4 are higher in sodium. 28. A client is prescribed sublingual nitroglycerine for the treatment of angina pectoris. What response from the client indicates that the client understands this medication?
For more nursing reviewers, contact ebookwholeseller@yahoo.com 09394837323

94

Generated by Foxit PDF Creator Foxit Software http://www.foxitsoftware.com For evaluation only.

a. Will the physician give me a years supply of nitroglycerine tablets? b. I will carry my nitroglycerine tablets in the inside pocket of my jacket, so they are always close. c. I usually take three of my nitroglycerine tablets at the same time. I find that they work better that way. d. I have a small labeled case for a few nitroglycerine tablets that I carry with me when I go out. Ratio: answer: D Nitroglycerine loses potency over time when exposed to light and heat. They should be kept cool, dry, and in a dark container. Clients should get a new bottle every 6 months, and store them in a cool place; tablets should be taken 5 minutes apart, taking more that one tablet at a time can actually decrease the effectiveness of the drug and may cause severe hypotension. 29. A client is being evaluated for a possible myocardial infarction. The nurse performs a 12-lead ECG for an episode of new chest pain. The nurse will monitor for which sign of acute myocardial Injury? a. ST depression b. ST elevations c. New Q wave d. New U wave Ratio: answer: B ST elevations indicate immediate myocardial injury; ST depressions indicate myocardial ischemia; a Q wave forms several days after a myocardial infarction; a U wave is a sign of hypokalemia. 30. The nurse is caring for a client with new onset atrial fibrillation. The nurse anticipates that which of the following is a possible treatment for this dysrhythmia when it first develops? a. External pacemaker application b. Insertion of automatic internal cardiac defibrillator (AICD) c. Synchronized cardioversion d. Defibrillation Ratio: answer: C Synchronized cardioversion is most effective with new onset atrial fibrillation. Pacemakers are indicated for heart block, AICDs are used for ventricular dysrhythmias, and defibrillation is indicated for ventricular fibrillation and pulseless ventricular tachycardia. 31. The nurse is assessing a client the morning of a scheduled cardiac stress test. The client reports trhat no breakfast was delivered this morning and the client is hungry. Which of the following is the nurses best action. a. Bring the client coffee and toast. b. Explain that a client should have no food the morning of cardiac stress test c. Call the nutrition department and get the clients regular full breakfast. d. Have the nursing assistant get the client cereal with milk and orange juice. Ratio: answer: D The client should have a light meal with no caffeine before a cardiac stress test. Options 1, 2, and 3 are incorrect because they do not follow this guideline. 32. A hospitalized client ha continuous ECG monitoring, and the monitor shows that the rhythm has changed to ventricular tachycardia. Which of the following is the first action that the nurse should take? a. Administer intravenous lidocaine according to emergency protocol b. Obtain the defibrillator and defribillate the client. c. Quickly assess the clients level of consciousness, blood pressure, and pulse. d. Administer a precordial thump. Ratio: answer: C The best first action is to assess the clients level of consciousness and assess if the ventricular tachycardia is perfusing the body (BP, pulse). With pulseless ventricular tachycardia, immediate
For more nursing reviewers, contact ebookwholeseller@yahoo.com 09394837323

95

Generated by Foxit PDF Creator Foxit Software http://www.foxitsoftware.com For evaluation only.

defibrillation is performed by an ACLS certified nurse. If the client has a good BP and pulse, is awake and alert, the nurse may administer lidocaine as prescribed or, in some cases, administer a precordial thump. 33. The physician has diagnosed a myocardial infarction on the basis of ECG changes for a client in the emergency room. The nurse is assessing the client frequently, and notes that the client seems forgetful, making the nurse repeat the explanations about the ECG and non-invasive blood pressure monitors. The nurse concludes that the clients response is most likely due to which of the following reasons? a. The client is showing signs of Alzheimers disease. b. The client is showing signs of fear and anxiety. c. Nurses in the emergency room are too busy to properly edxplain the purpose of equipment. d. Memory lapses are common with clients experiencing myocardial infarctions. Ratio: answer: B Anxiety and fear are common responses to a diagnosis of myocardial infarction because of the possibility of death. This prevents the client and family from absorbing the detailed explanations about the care being provided. Memory lapses are not a common symptom of myocardial infarction, and there is not adequate information to determine that this memory lapse is associated with Alzheimers disease. Nurses in the emergency room are able to explain procedures well to their clients. 50 percent of people over the age of 50 develop varicose veins and a major risk factor is standing for long periods of time at work. The other responses do not address these concerns. 34. When assessing a client, the nurse determines the capillary refill time to be 7 seconds. The nurse determines the client may be experiencing: a. Normal signs of aging b. Impending stroke c. Decreased cardiac output d. Hypokalemia Ratio: answer: C Blanching of the nailbed for more than 3 seconds after of pressure may be indicate reduced arterial capillary perfusion, which may be an indication of decreased cardiac output. The other options are incorrect for the time frame indicated or do not apply. 35. After the first dose of an antihypertensive agent, your client suddenly becomes hypotensive. You should position the client: a. In a semi-Fowlers position b. In a side-lying position c. In Trendelburg position d. With legs elevated 30 degrees Ratio: answer: D Elevating the legs increases venous return to the heart and will assist in raising the blood pressure. A semi-Fowlers position could lower the blood pressure even further. A side-lying position will have no beneficial effect, and the Trendelburg position could impair respirations by causing upward pressure on the diaphragm, by gravity. 36. The nurse is planning to instruct a client on the side effect of nifedipine (Procardia) for hyprtension. Which side effect should the nurse include? a. Hypokalemia b. Dizziness c. Bleeding d, Tachycardia Ratio: answer: B
For more nursing reviewers, contact ebookwholeseller@yahoo.com 09394837323

96

Generated by Foxit PDF Creator Foxit Software http://www.foxitsoftware.com For evaluation only.

Calcium channel blockers relax arterial smooth muscle, which lowers peripheral resistance through vasodilation. Dizziness is a common side effect because of orthostatic hypotension. Clients need to be taught to change position slowly to prevent falls. 37. The nurse explains to a client that the goal of anti-coagulation therapy in a client with a deep vein thrombosis is to: a. Prevent embolization b. Dissolve the clot. c. Allow immediate ambulation d. Prevent infection. Ratio: answer: A Anticoagulation therapy is used for deep vein thrombosis to prevent propagation of the clot, development of a new thrombus, and embolization. It does not dissolve the clot. It has no effect on infection and does not allow for immediate ambulation. 38. The nurse needs to explore with a client her understanding of treatment options for varicose veins that were just described by the physician. Which treatment would the nurse plan to include in this discussion? a. Endarterectomy b. Venography c. Sclerotherapy d. Plethysmography Ratio: answer: C Scelorotherapy, the injection of a sclerosing agent into a varicose vein followed by compression with a compression bandage for a period of time, is a common procedure for varicose veins. 39. Which of the following statements would indicate a positive outcome for a client with chronic arterial occlusive disease? a. I will keep my feet elevated above the level of my heart when I sleep. b. I will wear my compression stockings when awake. c. I will keep walking even when I feel pain in my legs to increase circulation. d. I will check the temperature of my bathwater with my hands before getting into the water. Ratio: answer: D Sensation in the feet may be diminished in clients with arterial occlusive disease. Teach the client to check the bathwater with the hands top prevent the risk of burn injury. The client should stop and rest when pain is experienced (option C). Options A and B are useful treatments for venous disease. 40. What is the correct reference point that the nurse would use to measure a clients central venous pressure (CVP)? a. Right side, mid-clavicular line where it intersects with the fifth intercostals space. b. Mid-auxillary line at the level of the fifth intercostals space. c. Left midsternal border at the level of the fourth intercostals space. d. Anterior aspect of the thoracic cavity, left side at the fifth intercostals space. Ratio: answer: B The level of the right artrium must be determined, and each successive reading must be determined from the same point of reference on the client. This area is also called the phlebostatic axis. 41. What is an important nursing action in the safe administration of heparin? a. Check the prothrombin time (PT) and administer the medication if it is below 20 seconds. b. Use a 20-gauge, 1-inch needle and inject into the deltoid muscle and gently massage the area.
For more nursing reviewers, contact ebookwholeseller@yahoo.com 09394837323

97

Generated by Foxit PDF Creator Foxit Software http://www.foxitsoftware.com For evaluation only.

c. Dilute in 50 ml 5% dextrose in water (D 5 W) and infuse by intravenous piggyback (IVPB) over 15 minutes; check the clotting time one half hour after infusion. d. Use a 25-gauge, - inch needle and inject the medication into the subcutaneous tissue of the abdomen. Ratio: answer: D Medication should be administered with a small gauge needle (25 gauges) into the subcutaneous tissue, without aspirating or massaging the area. Partial thrombolastin time (PTT) is used to monitor the effects of heparin. Heparin is not infused by IVPB. 42. While discussing her diagnosis of hypertension, a client asks the nurse how long she is going to have to take of the medications that have been prescribed. On what principle is the nurses response based? a. The client will be scheduled for an appointment in 2 months; the doctor will decrease her medications at that time. b. As soon as her blood pressure (BP) returns to normal levels, the clients will be able to stop taking her medications. c. to maintain stable control of her BP, the client will have to take the medications indefinitely. d. The nurse cannot discuss the medications with the client; the client will need to talk with the doctor. Ratio: answer: C Noncompliance with blood pressure medications is a common problem in the treatment of hypertension. The client must understand that the only way to keep her blood pressure under control is to discontinue taking her medications. She is not going to be able to discontinue the medications unless there is significant change in her condition as a result of weight loss, an exercise program, and /or decreased stress. 43. The nurse is caring for a client who is 6 hours post partum. What nursing actions are directed toward the prevention of postpartum thrombophlebitis? a. Encourage the early ambulation and increased fluid intake. b. Bathroom privileges only and elevates the lower extremities. c. Administer anticoagulants and evaluate the clotting factors. d. Encourage her to breast-feed the infant as soon as possible. Ratio: answer: A Early ambulation is the most effective and safe way to prevent thrombophlebitis with any type of client. This promotes venous return and prevents venous stasis. Anticoagulants are not routinely given postpartum unless there is another pathological condition present. The legs should be elevated when the client is in a sitting position. 44. A client diagnosed with peripheral vascular disease is being discharged. Which of the clients risk factors would be most important to discuss? a. Orthostatic hypotension b. Age c. Smoking d. Hypoglycemia Ratio: answer: C Smoking causes vasoconstriction, which increases the complications brought about by PVD. This is a modifiable risk factor that will assist in increasing circulation. Age cannot be modified. The diabetic client needs to maintain good control of diabetic clients needs to maintain good control of diabetes, but PVD is a complication of the disease process. Orthostatic hypotension is not a factor in this client. 45. Four hours after aortic-femoral bypass graft surgery, the nurse assesses the client and is unable to palpate pulses in the operative leg. The client complains of pain in the leg. What is the first nursing action? a. Massage the leg and apply warm towels.
For more nursing reviewers, contact ebookwholeseller@yahoo.com 09394837323

98

Generated by Foxit PDF Creator Foxit Software http://www.foxitsoftware.com For evaluation only.

b. Elevated the leg and recheck the pulse. c. Call the physician immediately. d. Assist the client to ambulate. Ratio: answer: C Occlusion to the aortic/femoral bypass graft is considered an immediate medical emergency, and physician notification is imperative. No other nursing options would alleviate the problem. Massaging the leg and having the ambulate would be contraindicated. The nurse should not wait to call the physician if the pulses cannot be palpated and the client is experiencing pain. 46. The nurse is administering a fluid challenge to a client in hypovolemic shock. What nursing assessment data are most important in determining whether the client is responding favorably to the fluid replacement? a. Urine output increases from 25 ml/hr to 40 ml/hr. b. systolic BP increases from 80 mm Hg to 90 mm Hg. c. Central venous pressure increases from 5 cm H2O to 7cm H2O. d. The PaO2 increases to 90% saturation. Ratio: answer: C First-Degree heart block can only be evaluated with an ECG or monitor tracing because the distinguishing factor is a prolonged P-R interval; all beats are being conducted. Other options do not assess first-degree block. 47. The client returns to his room after a thoracotomy. What will the nursing assessment reveal if hypovolemia from excessive blood loss is present? a. CVP of 5cm H2O and urine output of 20 ml/hr. b. Jugular vein distention with the head elevated 45 degrees. c. Chest tube drainage of 50 ml/hr in the first 4 hours. d. Increased BP and increased pulse pressure. Ratio: answer: A A low-range CVP reading and the decrease in urine output would be associated with hypovolemia caused by hemorrhage. The decrease in urine output is reflective of poor renal perfusion. 48. A client with hypertension asks the nurse what type of exercise she should do each day. What is the nurses best response? a. Exercise for an hour, but only three times a week. b. Walk on the treadmill for 45 minutes every morning. c. Begin walking; increase distance as you tolerate it. d. Exercise only in the morning; stop when you get tired. Ratio: answer: C A complication of hypertension is congestive heart failure, which may be first seen as dyspnea on exertion. The client should exercise as tolerated and stop when she gets tired or begins to have shortness of breath, regardless of the amount of time she has already exercised. 49. The nurse is monitoring an IV infusion of sodium nitroprusside (Nipride). Fifteen minutes after the infusion is started, the clients BP goes from 190/120 mm Hg to 120/90 mm Hg. What is a priority nursing action? a. Recheck the BP and call the doctor. b. Decrease the infusion rate and recheck the BP in 5 minutes. c. Stop the medication and keep the Iv open with D5 W. d. Assess the clients tolerance of the current level of BP. Ratio: answer: B
For more nursing reviewers, contact ebookwholeseller@yahoo.com 09394837323

99

Generated by Foxit PDF Creator Foxit Software http://www.foxitsoftware.com For evaluation only.

Nipride is a very powerful, rapid vasodilator. The nurse should decrease the infusion first before the pressure drops further, then assess the clients response to decreased rate. If the clients urinary output remains adequate and there is no dizziness or neurological change, then the client is probably tolerating the blood pressure level. 50. The nurse is teaching a client with hypertension about his antihypertensive medications, furosemide (Lasix) and captopril (capotene). What is important to include in this teaching? a. Stand up slowly to decrease problem with dizziness. b. Increase fluid intake because of increased loss of body fluids. c. When you begin to feel better, the doctor will decrease your medications. d. Stay out of the sunshine and make sure you have adequate sodium intake. Ratio: answer: A A common side effect of a combination of hypotensive and diuretic medications is postural hypotension. It is important to teach the client how to deal with it. The client should not increase intake of fluids because the diuretics are being given to decrease excess fluid. The client should decrease intake of sodium. When the client is feeling better, the medications are working. 51. The nurse is preparing a client for a cardiac catheterization. What is the best explanation regarding the purpose of a cardiac catheterization with coronary angiography? a. Evaluate the exercise tolerance. b. Study the conduction system. c. Evaluate coronary artery blood flow. d. Measure the pumping capacity of the heart. Ratio: answer: C In cardiac catheterization with angiography, contrast dye is injected into the coronary arteries, which allows visualization of the coronary arteries and provides information of their patency. Exercise tolerance is a stress test, and an electrocardoagram (EKG) is a study of the conduction system. Pumping capacity can be determined during a catheterization, but the question specifically asked about cardiac angiography, which is a study of cardiac vessels. 52. The nurse is caring for a client with cor pulmonale. What nursing assessment information correlates with an increase in venous pressure? a. Jugular vein distension with client sitting at a 45 degree angle. b. Crackling sounds over the lower lobes with client in an upright position. c. Bradycardia, restlessness, and an increase in respiratory rate. d. Jugular vein distension with the client supine and the head of the bed flat. Ratio: answer: A Jugular vein distension with the client in a sitting position, or with a 45-degree head elevation, is indicative of an increase in the central venous pressure. Many clients experience jugular vein distension when in a supine position, and it is not indicative of an increase in central venous pressure. Adventitious breath sounds, bradycardia, restlessness, and tachypnea are not directly associated with increased jugular vein distention but may occur if the client develops right-sided heart failure. 53. In discharge planning for the client with CHF, the nurse discusses the importance of adequate rest. What information is most important? a. A warm, quite room is necessary. b. Bed rest promotes venous return. c. A hospital bed is necessary. d. Adequate rest decreases cardiac workload. Ratio: answer: D

For more nursing reviewers, contact ebookwholeseller@yahoo.com

09394837323

100

Generated by Foxit PDF Creator Foxit Software http://www.foxitsoftware.com For evaluation only.

In order to decrease pulmonary congestion and dyspnea, it is desirable to decrease cardiac workload by encouraging adequate rest; the client should not exert himself to the point of fatigue. Bed rest does promote venous return, but that is not the purpose of bed rest in the client with CHF. 54. The nurse is evaluating a clients progress. What information would be indicative of a cardiac compensatory mechanism? a. Ventricular dilation, hypertrophy, and tachycardia. b. Hepatomegaly, splenomegaly, and cardiac hypertrophy. c. Headache, drowsiness, and confusion, d. Bradycardia, restlessness, and hyperventilation. Ratio: answer: A Compensatory mechanisms assist the failing heart to maintain an adequate cardiac output and blood flow to the tissues. These changes will initially maintain the blood flow in clients with a decrease in cardiac output. Increase in cardiac rate and size wit ventricular dilation all increase the cardiac output. 55. The nurse is taking the history from a client with CHF caused by hypertension. The nurse identifies what data as supportive of the clients medical diagnosis? a. Dyspnea after walking one block. b. weight loss of 15 pounds over last 3 months. c. Lower extremity edema in the evenings. d. dizziness and fainting when rising too quickly. Ratio: answer: A Dyspnea on exertion is a classic sign of left ventricular problems, regardless of the precipitating cause. Lower extremity edema is also characteristic but not as much as the dyspnea. 56. What would be the home care goal for a client who has bacterial endocarditis? a. To begin an exercise regimen as soon as possible. b. To monitor urinary output. c. To continue antibiotic therapy. d. To decrease activity until pulse stabilizes. Ratio: answer: C Antibiotics (usually administered by IVPB) are indicated for bacterial endocarditis. The home care nurse will monitor this clients daily IVPBs. This is to prevent vegetation growth on the valves. Other options are not specific to bacterial endocarditis.

57. If the nurse notes cloudy drainage 2 days post insertion of a Tenckhoff catheter for peritoneal dialysis, what other data does the nurse need to collect before reporting this finding? a. b. c. d. bowel sounds breath sounds temperature urine output

Rationale: The correct answer is number: C You are correct! This finding indicates potential infection so temperature is essential to evaluate before notification of the care provider.

For more nursing reviewers, contact ebookwholeseller@yahoo.com

09394837323

101

Generated by Foxit PDF Creator Foxit Software http://www.foxitsoftware.com For evaluation only.

58. Lactulose (Chronulac) has been prescribed for a client with advanced liver disease. Which of the following assessments would the nurse use to evaluate the effectiveness of this treatment? a. b. c. d. An increase in appetite A decrease in fluid retention A decrease in lethargy A reduction in jaundice

Rationale: The correct answer is number: 3 Due to physiological changes in the elderly, as well as conditions such as dehydration, hyperthermia, immobility and liver disease, the metabolism of drugs may decrease. As a result, drugs can accumulate to toxic levels and cause serious adverse reactions. 59. An adult is scheduled to undergo an exploratory laparotomy in one hour. The nurse has just received the order to administer his preoperative medication. What assessment is essential for the nurse before administering the medication? a. b. c. d. The clients ability to cough and deep breathe. Any drug hypersensitivity or allergy. The clients understanding of the surgical procedure. Whether the clients family is present and supportive.

Ratio: Correct Answer B 1. His ability to cough and deep breathe should be assessed earlier so that further teaching can take place if needed. Once preoperative medications are administered, the clients ability to retain information is impaired. 2. A complete drug history on every preoperative client is essential because of potential reactions to drugs. Drug hypersensitivity and allergic reactions must be assessed before preoperative medications are administered. 3. The clients understanding should be assessed earlier so the nurse can do further teaching if indicated. This should be done before the operative consent is signed. 4. While it is optimal to have the family present, medication should be given as ordered so that the timing of the peak action is most beneficial to the client. 60. A client asks the nurse how she can she live without her gallbladder. In order to respond to this client, the nurse must have which understanding of the hepatobiliary system? a. The liver produces about 1000 mL of bile per day b. The gallbladder produces about 90 mL of bile per day c. The liver concentrates bile more than 10 times d. the gallbladder dilutes and release bile Ratio: Correct answer: A The liver produces between 700 and 1000 mL of bile per day. The gallbladder stores and concentrates bile and then releases it when stimulated, but is not an essential structure. 61. The client is diagnosed with obstructive jaundice. The nurse should ask the client about which of this manifestation? a. b. c. d. Clear, pale urine Clay-colored stools Lactose intolerance Ankle edema

Ratio: Correct answer: B


For more nursing reviewers, contact ebookwholeseller@yahoo.com 09394837323

102

Generated by Foxit PDF Creator Foxit Software http://www.foxitsoftware.com For evaluation only.

Clay-colored stools indicate that no bile is reaching the intestine and suggest obstructive jaundice. Option A and C are unrelated to the question. Option D can be present due to cardio vascular disease or as an indirect consequence of portal hypertension with impaired venous return, but there is insufficient information in the question to support the opinion. 62. A client has jaundice. Which of the following comfort measures would be appropriate fort he nurse to implement? a. b. c. d. Offer hot beverages frequently Encourage taking a hot bath or shower Keep the air temperature at approximately 68 to 70 F Suggest the use of alcohol based skin lotion

Ratio: Correct answer: C Jaundice frequently causes pruritis. Comfort measures include keeping the air temperature cool (68 to 70 F) and the humidity at 30 to 40 percent. Tepid baths (not hot) with colloidal agents decrease itching (option b). Use of an emollient lotion is also helpful, but anything drying should be avoided (option D). Hot beverages (option A) are of no benefit as a comfort measure for pruritus due to jaundice. 63. The client has just had a liver biopsy. Which of the following nursing action would be the priority after the biopsy? a. b. c. d. Monitor pulse and blood pressure every 30 minutes until stable then hourly up to 24 hours Ambulate every 4 hours for the first day as long as the client van tolerate this Measure urine specific gravity every 8 hours for the next 48 hours Maintain NPO status for 24 hours post-biopsy

Ratio: Correct answer: A Complications of liver biopsy include hemorrhage or accidental penetration of biliary canniculi. The nurse should assess for for sign of hemorrhage (increased pulse, decreased blood pressure) every 30 minutes for the first few hours and then hourly 24 hours. The client should be monitored for every 4 hours and remain on bed rest for 24 hours. 64. Lactulose (Cephulac) is ordered for the client with cirrhosis. Which of the following serum laboratory test should the nurse monitor to determine if the drug is having the desired effect? a. b. c. d. Albumin Ammonia Sodium Lactate

Ratio: Correct answer: B Lactulose (Cephulac) is a disaccharide laxative used to decrease the absorption of ammonia in the intestines, thereby lowering the serum ammonia and resulting in improvement in hepatic encephalopathy. 65. The client is admitted to the hospital for possible cholelithiasis. While taking the history, the nurse notes that the client has which of the following risk factors for development of gallstones: a. Black race b. History of hypertension c. Age of 37 years d. Use of oral contraception Ratio: Correct answer: D Factors that increase the risk of gallstone formation include female gender, aging, use of oral contraceptives, pregnancy, and rapid weight loss, high cholesterol level, and diseases of the ileum.
For more nursing reviewers, contact ebookwholeseller@yahoo.com 09394837323

103

Generated by Foxit PDF Creator Foxit Software http://www.foxitsoftware.com For evaluation only.

66. A client with cirrhosis is admitted to the hospital. Which of the following assessments made by the nurse would indicate the development of portal hypertension? a. b. c. d. Hematemesis Asterixis Elevated blood pressure Confusion

Ratio: Correct answer: A In cirrhosis, the liver becomes fibrotic, which obstructs the venous blood flow through the liver. This increases the vascular pressure in the portal system, and causes congestion in the spleen and development of variscosities in the esophagus. Bleeding esophageal varices are a complication of portal hypertension and result in vomiting of blood and possible hemorrhage and death. 67. The nurse is doing discharge teaching for a client who has cirrhosis and ascites. Which of the following foods used by the client as snacks should the nurse instruct the client to avoid? a. b. c. d. Whole wheat bread Cookies Potato chips Hard candy

Ratio: Correct answer: C A low-sodium diet is recommended for client that has cirrhosisand ascites. Potato chips are high in sodium. Cookies and hard candy are high in sugar, while bread is high in complex carbohydrates. 68. The client who has disease asks the nurse why the bruises bso easily. Which of the following information should the nurse include in the response? a. Your liver is unable to make the proteins that are neede to making clotting factors. b. Your liver can no longer metabolize drugs and render them inactive. c. Your liver is breaking down blood cells too rapidly. d. Your liver cant store vitamin C any longer. Ratio: Correct answer: A The liver synthesizes clotting factirs I, II, VII, IX and X as well as prothrombin and fibrinogen. These substances are needed for adequate clotting, so their reduction leads to increased risk of bleeding. The other responses do not address these concerns. 69. A client is seen in the clinic for a routine physical examination and the laboratory test results indicate are elevated HBsAg. In order to plan teaching this lab result to mean: a. b. c. d. The client has immunity to hepatitis B The client has active hepatitis B The client has resolving hepatitis B The client has had the hepatitis B vaccine.

Ratio: correct answer: B HBsAg is hepatitis surface antigen and is usually present before symptoms manifest. It indicates acute disease. The other options are incorrect conclusions regarding this test results. 70. The client who has esophageal varies is receiving a vasopressin infusion. Which of these findings would indicate a complication of this therapy? a. b. c. d. Chest pain Tinnitus Flushed skin Polyuria
09394837323

For more nursing reviewers, contact ebookwholeseller@yahoo.com

104

Generated by Foxit PDF Creator Foxit Software http://www.foxitsoftware.com For evaluation only.

Ratio: correct answer: A Vasopressin causes vasoconstriction and may precipitate an acute anginal attack or myocardial infarction, especially in those with known cardiovascular disease. The other options are unrelated to the questions. 71. The client who has cholelithisis is scheduled for extracorporeal shock wave lithotripsy. The nurse should tell the client about which of these symptoms that may occur after this procedure? a. b. c. d. Colic-type pain Headache Diarrhea Hiccups

Ratio: correct answer: A After the extracorporeal shock wave lithotripsy, the nurse should monitor for biliary colic and nausea. The colicky pain is caused by passage of stone fragments through the biliary tree into the small intestine. Headache, diarrhea, and hiccups are unrelated manifestations. 72. The client is admitted to the hospital with acute pancreatitis. The nurse taking a history question the client about which of these risk for developing pancreatitis? a. b. c. d. Inflammatory bowel disease Alcoholism Diabetes mellitus High-fibre diet

Ratio: correct answer: B Pancreatitis is associated with alcoholism in men and gallstones in women. This disorders in option A and C are not associated with increased risk of pancreatitis, while option D promtes health. 73. A client with a subtotal gastrectomy is scheduled for discharge. Which of these instructions should the nurse give the client to reduce the possibility of dumping syndrome? a. b. c. d. Be sure to eat foods high in complex carbohydrates. It is helpful to take a walk after eating. Avoid drinking fluids your meal. Dont lie down for at least 2 hours after eating.

Ratio: correct answer: C Dumping syndrome is the rapid of food into the jejunum without proper missing and digestion. Interventions that help to minimize dumping syndrome are lying down after eating, eating a diet high in far and protein and low in carbohydrates, and no fluids with meal. 74. The nurse teaches the client with gastroesophageal reflux disease (GERD) about ways to minimize symptoms. Which of the following statements made by the client indicates that more teaching is needed? a. b. c. d. I will be sure to drink tea instead of coffee. I will take a walk after I eat. I will try to eat smaller mmeals more frequently. I will sleep with the head of the bed elevated about 12 inches.

Ratio: correct answer: the client with GERD is encourage to eat smaller, low-fat frequent meals and to avoid lying down after eating. Clients are instructed to not eat for at least 2 nhours before bedtime and avoid foods that decrease lower esophageal sphincter pressure, such as anything containing caffeine (coffee, tea, cola, chocolate)

For more nursing reviewers, contact ebookwholeseller@yahoo.com

09394837323

105

Generated by Foxit PDF Creator Foxit Software http://www.foxitsoftware.com For evaluation only.

75. The client with a gastric ulcer is admitted to the hospital. The nurse should assess the client for intake of which of these substances that increases the risk of developing a gastric ulcer? a. b. c. d. Aspirin Spicy food Acetaminophen (Tylenol) Coffee

Ratio: correct answer: D Famotidine (Pepcid) is a histamine-2 receptor antagonist and reduces the secretions of gastric cid. This class of drugs does not have a direct effect on reflux, LES tone, or GI motility. 76. The client is admitted ti the hospital with ulcerative colitis. The nurse should assess the client for which complication of the disease? a. b. c. d. Anemia Steotorrhea Cholelithiasis Thromboxcytopenia

Ratio: correct answer: A Hemorrhage and bleeding is a common feature of ulcerative colitis, and over time this can lead to significant loss of RBCs. The client should be assessed for possible anemia. 77. A client is to receive gavage feeding through an NG tube. Which of the following nursing actions should be instituted to prevent complications? a. b. c. d. Flush with 20 mL of air Place client in high-Fowlers position Advance tube 1 cm Plug the air vent during feeding

Ratio: correct answer: B Keeping the client in a high Fowlers position minimizes the risk of aspiration. The other options do not address this priority issue of care. 78. The nurse is caring for a client with Sengstaken Blakemore tube. Which of the following actions should the nurse take first if the client suddenly experiences difficulty breathing? a. b. c. d. Elevate the head of the bed Apply oxygen with a nasal cannula Listen to the clients lungs Cut and remove the tube

Ratio: correct answer: D Scissors should be kept at the bedside of all clients with an esophagogastric tube and the tube should be cut if the client experiences respiratory compromise. Maintaining the clients airway is the first priority. 79. The client returns to the nursing unit postoperatively after a colostomy. Which of the following assessments would require immediate action by the nurse? a. b. c. d. Stoma is bright red Stoma is blush Stoma is draining serous fluid Stoma is draining fluid

Ratio: correct answer: B A healthy stoma is red to reddish-pink, moist, and shiny. A stoma that appears dark red, bluish, or black indicates ischemia or necrosis. This finding must be reported immediately because the viability of the tissue is at risk. Option C and D are of no concern immediately.
For more nursing reviewers, contact ebookwholeseller@yahoo.com 09394837323

106

Generated by Foxit PDF Creator Foxit Software http://www.foxitsoftware.com For evaluation only.

80. A client who had a Billroth I procedure is beginning to eat solid foods. The nurse should assess the client for the development of dumping syndrome by determining the presence of which of the following? a. b. c. d. Bradychardia Diarrhea Dyspnea Coughing

Ratio: correct answer: B Symptoms of dumping syndrome can occur within 5 minutes to 3 hours after eating and include nausea, vomiting, tachycardia, diaphoresis, abdominal pain, syncope, and hyperactive bowel sounds. 81. The nurse is planning care for the client scheduled for gastroduodenoscopy and a barium swallows. What will the nursing plan include? a. Anticipating the client will receive a low-residue diet in the evening and then receive nothing by mouth (NPO status) 6 to 12 hours before the test. b. Discussing with the client the nasogastric tube and the importance of gastric drainage of 24 hours the test. c. Explaining to the client that he will receive nothing by mouth (NPO status) for 24 hours after the test to make sure his stomach can tolerate his food. d. Discussing the general anesthesia and explaining that he will wake up in the recovery room. Ratio: answer: A NPO status before a barium swallow and a gastroduodenoscopy and low-residue diet the evening before the procedures are routine orders for these test. 82. In planning discharge teaching for the client who has undergone gastrectomy, the nurse includes what information regarding dumping syndrome? a. The syndrome will be a permanent problem and the client should eat 5 to 6 small meals per day. b. The client should decrease the amount of fluid consumed with each meal and for 1 hour after each meal. c. The client should increase the amount of complex carbohydrates and fiber in his diet. d. Activity will decrease the problem; it should be scheduled about 1 hour after meals. Ratio: answer: B The syndrome is self-limiting. Decreasing fluid intake with and after meals, eating small meals, and decreasing carbohydrate and salt intake will decrease the dumping effect. 83. What is the priority nursing action for the client who is complaining of nausea in the recovery room after gastric resection? a. Evaluate the nasogastric tube for patency. b. Call the physician for an antiemetic order. c. Place client in semi-Fowlers position so he will not aspirate. d. Medicate the client with a narcotic analgesic. Ratio: answer: A Evaluate the nasogastric tube patency; it is very important to assess the client and determine the source of the nausea before calling the doctor. 84. The nurse knows that a conclusive diagnosis of pyloric stenosis will be made by what test? a. A flat plate of the abdomen b. A colonoscopy. c. An altered electrolyte balance.
For more nursing reviewers, contact ebookwholeseller@yahoo.com 09394837323

107

Generated by Foxit PDF Creator Foxit Software http://www.foxitsoftware.com For evaluation only.

d. An upper GI series. Ratio: answer: D An upper gastrointestinal series will indicate delayed gastric empty and an elongated pyloric channel. 85. A client is admitted with duodenal ulcers. What will the nurse anticipate the history to include? a. Recent weight loss. b. Increasing indigestion after meals. c. Awakening with pain at night. d. Episodes of vomiting. Ratio: answer: C Duodenal ulcers are characterized by high gastric acid secretion and rapid gastric emptying. Food buffers the effect of the acid. Therefore pain increases when the stomach is empty. 86. The nurse is conducting discharge dietary teaching for a client with diverticulosis who is recovering from an acute episode of diverticulosis. The nurse would determine that the client understood his dietary teaching by which statement? a. I will need to increase my intake of protein and complex carbohydrates to increase healing. b. Peanuts, fruits, and vegetables with seeds can cause problems, and I should avoid them. c. I will not put any added salt on my food, and I will decrease intake of foods that are high in saturated fat. d. Milk and milk products can cause a lactose intolerance. If this occurs, I need to decrease my intake of these products. Ratio: answer: B The primary problem with diverticula is food or indigestible fiber that gets caught in the poouches. The client should avoid this type of fiber. 87. The nurse is caring for a client who has a bleeding duodenal ulcer. The nurse identifies what assessment data is indicative of a gastric perforation? a. Increasing abdominal distention and tight abdomen. b. Decreasing hemoglabin and hematocrit with bloody stools. c. Diarrhea with increased bowel sounds and hypovolemia. d. Decreasing blood pressure with tachycardia and disorientation. Ratio: answer: A Perforation is characterized by increasing distention and board-like abdomen. The other option may be seen with hemorrhage. 88. The nurse prepares a client for a colonscopy and directs the client to move to which position? a. Prone b. Sims lateral. c. Slight Trendelenburg d. Flat with lithotomy stirrups. Ratio: answer: B Either Sims lateral or a knee-chest position is used for best access and visualization as well as for the clients comfort. 89. The nurse teaches the client which of the information regarding home collection of a stool specimen for Hemoccult testing? a. Three stimultaneous specimens should be sent to lab. b. Diet should be low in fiber and low in residue to quiet the bowel. c. Any slide positive finding requires additional evaluation.
For more nursing reviewers, contact ebookwholeseller@yahoo.com 09394837323

108

Generated by Foxit PDF Creator Foxit Software http://www.foxitsoftware.com For evaluation only.

d. Any red color on or near the specimen is considered positive. Ratio: answer: C Three consecutive specimens should be acquired and sent. Diet should be high residue. A blue color is positive. 90. Icteric skin occurs as a result of a. High levels of toxin exposure. b. Systemic changes in melanin production. c. Prolonged exposure to cold temperature. d. Abnormal amounts of bilirubin in the blood. Ratio: answer: D Bilirubin is the product of hemoglabin breakdown, and high amounts in skin result in a yellowish green hue to skin, which is called icterus. 91. One day after cholecystectomy, a 36-year-old client is to ambulate to the nurses station. She complains of too much pain and refuses. The best nursing action is to: a. Let her walk later that day. b. Express empathy and assess pain status. c. Notify the physician of her non-compliance. d. Allow her control in deciding when to walk. Ratio: answer: B Postoperative clients require appropriate psychological support and do experience pain. Providing both will improve client outcomes. 92. The nurse is to administer an enema to an adult client. The tube should be inserted how many inches? a. 1 to 2 inches b. 2 to 3 inches c. 3 to 4inches d. 4 to 6 inches Ratio: answer: C Three to four inches is required for an adult to clear the rectal sphincter. 93. The nurse is caring for a client with chronic hepatitis B (HBV). What will the teaching plan for this client include? a. Avoid sexual activity b. Report any clay-colored stools c. Eat a high-protein diet d. Perform daily urine bilirubin check. Ratio: answer: A HBV is spread by sexual contact. The client should not be sexually active until the HbsAB antibodies (antibodies to antigen) are present. There will be no bilibrubin in the urine or stools; clay-colored stools are expected, so they would not be reported. 94. Which position is best for the client who has undergone abdominal cholecystectomy? a. Side-lying position, to prevent aspiration b. Semi-Fowlers position, to facilitate breathing c. Supine, to decrease strain on the incision line. d. Prone, to reduce nausea.
For more nursing reviewers, contact ebookwholeseller@yahoo.com 09394837323

109

Generated by Foxit PDF Creator Foxit Software http://www.foxitsoftware.com For evaluation only.

Ratio: answer: B A semi-Fowlers position improves lung expansion. The incision for cholecystectomy is high and may interfere with respiratory exchange. The other positions would probably interfere with respiration. 95. The nurse is making a home visit to a client with hepatitis A (HAV). Before assessing the client, the nurse will gather the equipment and perform what action next? a. Wipe the bedside table with alcohol preps. b. Place the supplies on the bedside. c. Place paper towels on the bedside table. d. Put on a gown and gloves. Ratio: answer: C Paper towels are used to create a clean area surface. Alcohol preps are not effective. The gown and gloves are not indicated for assessment. 96. While talking with a client with a diagnosis of end-stage liver disease, the nurse noticed the client was unable to stay awake and seemed to fall asleep in the middle of a sentence. The nurse recognizes these symptoms to be indicative of what condition? a. Hyperglycemia b. Increased bile production c. Increased blood ammonia levels. d. Hypocalcemia Ratio: answer: C In end-stage liver disease, the liver cannot break down ammonia by-products of protein metabolism. The increased ammonia levels in the serum cross the blood-brain barrier, causing uncontrolled drowsiness and confusion. 97. What is the name of the accumulation of fluid in the peritoneal cavity in individual liver disease? a. Portal hypertension b. Ascites c. Peritonitis d. Cirrhosis of the liver. Ratio: answer: B Portal hypertension, peritonitis, and cirrhosis of the liver are all causes of ascites, which is collection of fluid in the peritoneal cavity. 98. The nurse is caring for a client with hepatitis A. Which type of infection precautions are appropriate for this client? a. Standard precautions. b. Droplets precautions. c. Contact precautions. d. Bloodborne precautions. Ratio: answer: A Standard precautions are the appropriate type of infection precautions for all clients with hepatitis. Droplets precautions are not necessary for clients with hepatitis. Because hepatitis A is transmitted by the oral-fecal route, contact precautions are not necessary, except for the methods provided by standard precautions. Bloodborne precautions (part of standard precautions) are necessary for clients with hepatitis B and C (which are bloodborne), as well as for clients with hepatitis D. 99. A client asks how her body continue to function normally after a laparoscopic cholecystectomy. Which of the following answers demonstrates understanding of the ensuing education? a. I should never eat foods with fat again.
For more nursing reviewers, contact ebookwholeseller@yahoo.com 09394837323

110

Generated by Foxit PDF Creator Foxit Software http://www.foxitsoftware.com For evaluation only.

b. The primary function of the gallbladder is concentration and storage bile. c. The liver takes over the functions of the gallbladder. d. The gallbladder has no known function in the human body. Ratio: answer: B The fuction of the gallbladder is the concentration and storage of bile. With the gallbladder absent, the liver will continue to produce bile to emulsify fats, but there wont be an excess of bile available. Clients who have undergone a cholecystectomy should eat a low-fat diet do not need to make any other dietary adjustment. 100. After administering diuretics to a client with ascites, which of the following nursing actions most ensures safe care? a. Measures serum potassium for hyperkalemia. b. Assess the client for hypervolemia. c. Weigh client weekly. d. Document accurate intake and output. Ratio: answer: D Accurate intake and output measurements are essential for clients receiving diuretics. Hypokalemia, not hyperkalemia, is a frequent occurrence with diuretic therapy, and hypovolemia is a much greater risk with an increased urine output. Clients should be weighed daily.

For more nursing reviewers, contact ebookwholeseller@yahoo.com

09394837323

111

Generated by Foxit PDF Creator Foxit Software http://www.foxitsoftware.com For evaluation only.

NURSING PRACTICE TEST V 1. Which of the following responses by the nurse is the best example of clarifying? a. b. c. d. Tell me about what you were thinking before you went to talk to him. When did you first notice these feelings. Instead of talking about your mother, I want to know how you feel. Im having difficulty understanding. Could you explain that to me?

Answer. D Option D is seeking clarification after the nurse was unable to understand the client. Option A is exploring, option b is placing event in time or sequence, and option C is focusing. Source: Review and Rationale Series for nursing Mental Health by Hogan and Smith page 26 2. An emergency psychiatric client presents with hyperthermia and unexplained loss of appetite. The nurse concludes that these symptoms are consistent with trauma to which area of the brain? a. b. c. d. Thalamus Hypothalamus Cerebrum Cerebellum

Answer: B The hypothalamus is located in the deincephalon and Is responsible for regulating temperature, appetite, and the integration of the autonomic nervous system. The thalamus (option A) is also located in the diencephalons, and is functions are concerned primarily with sensation. The cerebrums (option C) primary functions include higher-order thinking, abstract reasoning, visual functions, judgment, memory, and sensory function. The cerebellum (option D ) is primarily responsible for balance and coordination. Source: Review and Rationale Series for nursing Mental Health by Hogan and Smith page 27 3. A client who is unable to cope with the sudden loss of a job and who is feeling confused and unable to make decisions is to be experiencing which of the following? a. b. c. d. Adventitious crisis. Maturational crisis Situational crisis Social crisis

Answer: C A situational crisis is one that is often sudden and unavoidable. The stressful event threatens a persons physical, emotional, or social integrity. An adventitious crisis (option A) occurs from an accidental or sporadic event. A maturational crisis (option B) occurs because of the situation occurring from the maturing process, such as in the adolescents or older adults. A social crisis (option D) is a crisis that occurs within a social context. Source: Review and Rationale Series for nursing Mental Health by Hogan and Smith page 45.

For more nursing reviewers, contact ebookwholeseller@yahoo.com

09394837323

112

Generated by Foxit PDF Creator Foxit Software http://www.foxitsoftware.com For evaluation only.

4.

In assessing a client in crisis, it is important for the nurse to first assist the client to identify: a. b. c. d. The clients feeling The realistic nature of the event. Others who might be affected by the event An immediate action plan

Answer: A It is helpful for the client to identify the feelings he or she has about the crisis in order to feel validated and begin work on the problem. The realistic nature of the event (option B), others impacted by the events (option C), and a plan of action (option D) all are important next steps once the client has identified hi or her own feelings. Source: Review and Rationale Series for nursing Mental Health by Hogan and Smith page 45. 5. When working with the client in crisis, which of the following is the most a. b. c. d. important?

Obtaining a complete assessment of the clients past history Remaining focused on the immediate problem Determining whether the client may have had a part in the emergence of the crisis. Assisting the client to identify what is similar about crisis to other crises in the clients life.

Answer: B The nurse must remain focused on the immediate problem as there is not enough time and no need to delve into the complete past history (option A). The clients role in the current crisis (option C) is not relevant at this time, although it may be more important in learning to prevent future crisis situations. Assisting the client to identify that is similar about this crisis to other crisis (option D) may be a usual next step. Source: Review and Rationale Series for nursing Mental Health by Hogan and Smith page 45 6. A 23-year-old client whos life partner died recently from complications of AIDS has just found out that he is HIV- positive. He has been referred to the outpatient crisis unit from his doctors office because he shut down after finding out his HIV status. The nurse meets with the client, provides comfort measures, and begins the assessment. An immediate priority is to evaluate if the client: a. b. c. d. Is at risk for self-directed violence Has an altered thought process Has a psychiatric provider Has a fear of dying

Answer: A While some clients will not talk about thoughts of self- harm, they will usually talk about suicidal thoughts when asked. Safety is priority and suicidal clients should not be left alone. Altered thought process (option B) , psychiatric providers (option C) and feeling toward dying (option D) are important assessment areas after the clients safety has been ensured. Source: Review and Rationale Series for nursing Mental Health by Hogan and Smith page 46. 7. A 52-year-old client who was admitted to the hospital 5days ago with major depression and suicidal ideations is now preparing for discharge. Which of the following statements for the client demonstrates she has met one of her outcome/evaluation measures When I go home: a. b. c. d. Ill finally get some sleep Ill be able to take care of my plants again. I have a list of people that I can call if I start to feel poorly. Ill cook for myself.

Answer: C
For more nursing reviewers, contact ebookwholeseller@yahoo.com 09394837323

113

Generated by Foxit PDF Creator Foxit Software http://www.foxitsoftware.com For evaluation only.

One desired outcome is for the client to have enhanced social support. The client should be sleeping better by discharge (option 1). Taking care of her plants (option B) and cooking for herself (option D) do not necessarily indicate the clients level of recovery. Source: Review and Rationale Series for nursing Mental Health by Hogan and Smith page 47. 8. A 3-year-old client has been diagnosed with attention deficit/ hyperactivity disorder (ADHD). Which medication is most likely to be prescribed? a. b. c. d. Amitriptyline (Elavil) Paroxetene (paxil) Methylphenidate (Ritalin) Pemoline (Cyclert)

Answer: C Central nervous system stimulants such as Ritalin are the most frequently used medications for ADHD. These medications increase the ability to focus attention by blocking out irrelevant thoughts and impulses. Antidepressants (options A and B) may be used, but venlafaxine (Effexor) and fluvoxamine (Luvox) seem to be the most effective. Cyclert (option D) is used for ADHD, may have fewer side effects, but is used less often because it takes up to 8 weeks to take effect. Source: Review and Rationale Series for nursing Mental Health by Hogan and Smith page 60 9. The client is being admitted to the inpatient psychiatric unit. You determine that which of the following must be present in order to be diagnosed with major depression? a. b. c. d. Suicidal thoughts or plans of suicide reported over at least the last 2 weeks History of one depressive episode within the last 2 years Loss of appetite for more than 3 days Loss of interest in previously enjoyed activities.

Answer: D DSM-IV-TR states that depressed mood or loss of interest in previously enjoyed activities must be present in order to qualify for a diagnosis of major depression. Although each of the other options may be present with depression, these criteria must be met first. Source: Review and Rationale Series for nursing Mental Health by Hogan and Smith page 93 10. The nurse should consider the irregularities in which of the following body systems before an accurate diagnosis of mood disorder can be assigned? a. b. c. d. Integumentary Cardiovascular Respiratory Endocrine

Answer. D Any client who is being evaluated for a mood disorder should have a workup to rule out the possibility of a pathophysiologic disorder being overlooked. The body systems listed in options A, B, and C would not have actual irregularities that would indicate the same signs and symptoms as mood disorders. Source: Review and Rationale Series for nursing Mental Health by Hogan and Smith page 93 11. During an assessment interview the client tells the nurse, I cant stop worrying about my makeup. I cant go anywhere or do anything unless my makeup is fresh and perfect. I wash my face and put on fresh makeup at least once and sometimes twice an hour. This behavior is most likely a sign of a (n): a. Acute stress disorder
For more nursing reviewers, contact ebookwholeseller@yahoo.com 09394837323

114

Generated by Foxit PDF Creator Foxit Software http://www.foxitsoftware.com For evaluation only.

b. Generalized anxiety disorder c. Obsessive-compulsive disorder d. Panic disorder Answer: C Frequent and repetitive worries and behaviors are signs of an obsessive-compulsive disorder. Acute stress disorder and generalized anxiety disorders are characterized by a great deal of difficulty controlling unrealistic, excessive anxiety associated with common daily experiences or activities. Panic disorders are characterized by recurrent panic attacks and the source of the anxiety may not be identified. Source: Review and Rationale Series for nursing Mental Health by Hogan and Smith page 117 12. The spouse of a woman diagnosed with somatization disorder asks the nurse if this wife has so many health problems on purpose. The best response is: b. c. d. e. Have you tried asking her? I think shed tell you the truth. Your wife is trying to gain your attention. She doesnt have the problem on purpose; however, this is probably difficult for both of you. She has some significant emotional problems that she cannot admit.

Answer: C Family members must understand the mechanism of somatization disorder, and to have their own needs addressed. The chronic nature of the physical complaints is very frustrating and disrupting of family functioning. Source: Review and Rationale Series for nursing Mental Health by Hogan and Smith page 134. 13. The most appropriate nursing diagnosis for a client with a somatoform disorder is: a. b. c. d. Altered role performance Knowledge deficit: medication. High risk for violence, self-directed Acute trauma reaction

Answer: A Somatoform disorders result in altered role performance because the illness interferes with the usual responsibilities in life. There is not enough data to support knowledge deficit or risk for selfdirected violence. Acute trauma reaction does not exist. Source: Review and Rationale Series for nursing Mental Health by Hogan and Smith page 135. 14. A female client with a 15-year history of somatization disorder is to be discharged from her first psychiatric hospitalization. Which statement would indicate that nursing care has been effective? a. I need to make sure that all of my medications are sent home with me. b. I see now that when I get stressed, my body speaks for me. c. My family is good to me when I am sick like this. d. There are so many illness that you nurses simply do not know about. Answer: B The clients statement indicates accurate awareness of mind-body interaction. She does not suggest the symptoms are something to medicate, nor does she persist in identifying her illness as physical in origin, both of which are characteristic of somatization disorder. Source: Review and Rationale Series for nursing Mental Health by Hogan and Smith page 135. 15. The best initial approach to take with a self- accusatory, guilt ridden client would be to: a. Contradict the clients persecutory delusions b. Accept the clients statements as the clients beliefs.
09394837323

For more nursing reviewers, contact ebookwholeseller@yahoo.com

115

Generated by Foxit PDF Creator Foxit Software http://www.foxitsoftware.com For evaluation only.

c. Medicate the client when these thoughts are expressed. d. Redirect the client whenever a negative topic is mentioned ANSWER: B The nurse must accept the clients statement and beliefs as real to the client to develop trust and move into a therapeutic relationship. A Clients cant be argued out of delusions C These feelings and thoughts are constant; this would result in an overdose. D Redirecting the clients conversation whenever negative topics are brought up adds to the clients feelings that negative thoughts are correct. QUESTION # 77 p. 268 16. A client treated for hypochondriasis has an upsetting phone conversation with her husband and subsequently requests an analgesic. My head is killing me, and I know there is a tumor in there somewhere or it wouldnt hurt like this. The nurses best response is: a. b. c. d. You have no brain tumor. It is just your anger towards your husband. Ill get your vital signs and then call your doctor if they are abnormal. You must try not to rely on the pain pills so much since they are addictive. Ill get your medication and then lets talk about what just happened.

Answer: D The nurse should provide physical care for the client in a matter-of-fact manner and, at the same time, should help the client note how symptoms increase at the time of stress and can be a way of coping with stress. Source: Review and Rationale Series for nursing Mental Health by Hogan and Smith page 135. 17. A client with a somatization disorder has been attending group therapy. Which statement indicates that care has been effective? a. a. b. c. I think Id better get some pain pills. My back hurts from sitting in group. The other women in the group have mental problems! I havent said much, but I get a lot out of listening. I feel better physically just from getting a chance to talk.

Answer: D Participating in group therapy offers a chance to talk and to gain support from others, both of which free up energy. Source: Review and Rationale Series for nursing Mental Health by Hogan and Smith page 136. 18. The client, although oriented to person, place and time, cannot remember being extracted from his burning automobile the day before. His inability to remember events surrounding the accident is best described as: a. a. b. c. Denial Localized amnesia Confabulation continuous amnesia

Answer: B A localized amnesia is characterized by the inability to recall all events associated with a stressful event; whereas continuous amnesia would include the present (and the client is oriented to person, place, or time). Denial is an unconscious defense mechanism in which emotional conflict and anxiety are avoided by refusing to acknowledge those thoughts, feelings, or desires. Confabulation is the replacement of gaps in memory with imaginary information.
For more nursing reviewers, contact ebookwholeseller@yahoo.com 09394837323

116

Generated by Foxit PDF Creator Foxit Software http://www.foxitsoftware.com For evaluation only.

Source: Review and Rationale Series for nursing Mental Health by Hogan and Smith page 148. 19. A client recently released from prison for embezzlement has a history of blaming others for his problems and becoming defensive and angry when criticized. He has expressed no remorse for his actions nor any response to his conviction. He claims his actions were justified since his employer did not treat him fairly. He is displaying characteristics of which personality disorder? a. a. b. c. Narcissistic Histrionic Antisocial Borderline

Answer: C The described behavior reflects DSM-IV diagnostic criteria for antisocial personality disorder. His behavior is not characteristic of individuals diagnosed with narcissistic, histrionic, or borderline personality disorder. Source: Review and Rationale Series for nursing Mental Health by Hogan and Smith page 167. 20. A 35-year-old client is being interviewed by the nurse. The clients history indicates that she has few friends, fears criticism and rejection from others, and withholds information about her thoughts and feelings because she anticipates a negative reaction. Based on the data, the nurse suspects that the client may have which of the following personality disorder? a. b. c. d. Schizotypal Paranoid Avoidant Schizoid

Answer: C The described behavior reflects DSM-IV diagnostic criteria for avoidant personality disorder. His behavior is not characteristic of individuals diagnosed with schizotypal, paranoid or schizoid personality disorder. Source: Review and Rationale Series for nursing Mental Health by Hogan and Smith page 167. 21. Which nursing diagnosis may be a priority of care at the time of admission for a client diagnosed with antisocial personality disorder? a. b. c. d. Personal identity disturbance Fear Risk for violence directed at others Social isolation

Answer: C Individuals diagnosed with antisocial personality disorder display decreased impulse control, can be irritable and aggressive, and lack remorse or their action. Recognizing the potential risk for violence and maintaining client safety is the first priority of nursing care. The other nursing diagnoses do not reflect the behavioral pattern associated with individuals diagnosed with antisocial personality disorder. Source: Review and Rationale Series for nursing Mental Health by Hogan and Smith page 167. 22. The client diagnosed with borderline personality disorder tends to label certain persons on the staff as being good or bad. This behavior is an example of: a. a. b. c. Secondary gain Acting out Passive aggression Dichotomous thinking
09394837323

For more nursing reviewers, contact ebookwholeseller@yahoo.com

117

Generated by Foxit PDF Creator Foxit Software http://www.foxitsoftware.com For evaluation only.

Answer: D Individuals diagnosed with borderline personality disorder frequently display a tendency to dichotomous thinking or splitting. They perceive the self and others as all good or all bad. The acting out, and passive aggressive behavior do not involve a tendency to perceive the self and others as all good or bad. Source: Review and Rationale Series for nursing Mental Health by Hogan and Smith page 168. 23. In evaluating the progress of the client whose interpersonal relationships are based on manipulation, the most important criteria are the clients: a. b. c. d. Plans Promises Actions Words

Answer: C Plans, Promises, and words do not reflect actual behavioral change. Change is reflected in action. Source: Review and Rationale Series for nursing Mental Health by Hogan and Smith page 168. 24. A client with a diagnosis of schizophrenia is speaking in a group by putting rhyming words that have no meaning together. This speech pattern is known as: a. a. b. c. Echopraxia Echolalia Clang association Neologism

Answer: C Clang associations are association disturbances in which schizophrenic clients rhyme words in a sentence that make no sense. Echopraxia is meaningless imitation of motions made by others (option A). Echolalia is involuntary parrot-like repetition of words spoken by others (option B). Neologism is the coining of a new word that is meaningless to anyone but the client (option D). Source: Review and Rationale Series for nursing Mental Health by Hogan and Smith page 185. 25. The nurse administering atypical antipsychotic medication is aware that they have been defined as having which of the following characteristics? a. b. c. d. High risk for tardive dyskinesia Minimal to no risk for extrapyramidal effects Effective in treating only positive symptoms of schizophrenia Effective in treating only negative symptoms of schizophrenia.

Answer: B Atypical antipsychotic medications are helpful in treating both negative (option D) and positive (option C) symptoms of schizophrenia. This class of medications has minimal to no risk for extrapyramidal side effects, which includes tardive dyskinesia (option A). Source: Review and Rationale Series for nursing Mental Health by Hogan and Smith page 185. 26. A client taking antipsychotic medications for treatment of schizophrenia complains to the nurse of feeling nervous. The nurse notices that the client is pacing the long hallway and is unable to remain still even when other clients are talking with him. This client is most likely experiencing: a. Akathisia b. Akinesia
For more nursing reviewers, contact ebookwholeseller@yahoo.com 09394837323

118

Generated by Foxit PDF Creator Foxit Software http://www.foxitsoftware.com For evaluation only.

c. Dystonia d. Tardive dyskinesia Answer: A Akathisia is an extrapyramidal side effect of antipsychotic medications that may manifest as subjective and objective restlessness. Akinesia (option B), dystonia (option C), and tardive dyskinesia (option D) are also extrapyramidal side effects of antipsychotic medications, but are not characteristic of this clients symptoms. Source: Review and Rationale Series for nursing Mental Health by Hogan and Smith page 185. 27. What nursing diagnosis is most likely to be associated with a client diagnosed as having schizophrenia, disorganized type? a. Impaired verbal communication a. Sleep pattern disturbance b. Social isolation c. Self-care deficit Answer: A Schizophrenia, disorganized type, is characterized by disorganized speech patterns. Other manifestations of this diagnosis include disorganized behavior and inappropriate affect. Sleep pattern disturbance (option B), Social isolation (option C), and Self-care deficit (option D) are possible, but not classic for disorganized schizophrenia. Source: Review and Rationale Series for nursing Mental Health by Hogan and Smith page 167. 28. Which of the following is considered to be a positive symptom associated with schizophrenia? a. b. c. d. Alogia Avolition Social withdrawal Loose association

Answer: D Loose associations are considered to be a positive symptom associated with schizophrenia because they indicate a distortion or excess of normal functioning. Alogia (option A), avolition (option B), and social withdrawal (option C) are considered negative symptoms of schizophrenia. Negative symptoms indicate a loss or lack of normal functioning. Negative symptoms develop over time and hinder the clients ability to endure life tasks. Source: Review and Rationale Series for nursing Mental Health by Hogan and Smith page 186. 29. Family members have noticed that during the bath, a client tries to chew on a bar of soap. Which term best describes this behavior? a. a. b. c. Hyperactivity Hyperamorphosis Hyperorality Hyperemesis

Answer: C During stage 2 of Alzheimers disease, clients have a need to place objects in the mouth so they can taste or chew them, causing a health hazard. This behavior is called hyperorality. Hyperactivity is behavior characterized by decreased attention span, increased impulsivity, and emotional lability. Hyperamorphosis is the need to compulsively touch and examine every object in the environment. Hyperemesis is characterized by excessive vomiting. Source: Review and Rationale Series for nursing Mental Health by Hogan and Smith page 206.

For more nursing reviewers, contact ebookwholeseller@yahoo.com

09394837323

119

Generated by Foxit PDF Creator Foxit Software http://www.foxitsoftware.com For evaluation only.

30.

The nurse administering which of the following medications to a client realizes that it increases the availability of acetylcholine in the synapse and leads to the recovery of some mental functioning for the clients with dementia? a. b. c. d. Fluoxetine (Prozac) Trazodone (Desyrel) Haloperidol (Haldol) Donepezil (Aricept)

Answer: D Donepezil (Aricept) is a cholinesterase inhibitor that appears to slow down cognitive deterioration in individual with mild to moderate dementia. All other options may be prescribed for clients with dementia but have no proven effect for regaining cognitive function. Source: Review and Rationale Series for nursing Mental Health by Hogan and Smith page 206. 31. Client who is fighting against his restraints and shouting incoherently is brought by ambulance to the Emergency Department, accompanied by his girlfriend. She reports that he seemed fine until he took some pills that he had purchased that afternoon, but an hour later he went crazy. Which of the following actions should the nurse take first? a. b. c. d. Take his vital signs Check his orientation Start intravenous (IV) fluids Administer sedative medication.

Answer: A The highest priority is given to nursing interventions that will maintain life; therefore basic physiological needs must be addressed initially with the baseline vital signs. Nutrition and fluid balance may be maintained by IV therapy once vital signs are evaluated and a physicians order is obtained. Checking the levels of orientation is important but does not provide any new information to the nurse. Sedative medications may complicate an attempt to identify the original cause of the confusion. Source: Review and Rationale Series for nursing Mental Health by Hogan and Smith page 207. 32. When working with a client suspected of having Alzheimers disease, the nurse needs to be alert for increasing agitation that worsen at night, known as: a. b. c. d. Pseudodementia Pseudodelirium Catastrophic reaction Sundown syndrome

Answer: D Clients with dementia often experience extreme agitation at the end of the day, probably as a result of tiredness and fewer orienting stimuli such as planned activities and contact with people. These restless and agitated behaviors worsen at night and are commonly referred to as Sundown syndrome. Pseudodementia is a reversible disorder that mimics dementia. Pseudodelirium is characterized by symptoms of delirium without any identifiable organic cause. Catastrophic reaction is the overreaction toward minor stresses that occurs in demented clients. Source: Review and Rationale Series for nursing Mental Health by Hogan and Smith page 207. 33. A nurse explains to a mental health care technician that the clients obsessive-compulsive behaviors are related to unconscious conflict between id impulses and the superego (or conscience). On which of the following theories does the nurse base this statement? a. Behavioral theory b. Cognitive theory
For more nursing reviewers, contact ebookwholeseller@yahoo.com 09394837323

120

Generated by Foxit PDF Creator Foxit Software http://www.foxitsoftware.com For evaluation only.

c. Interpersonal theory d. Psychoanalytic theory Answer: D Psychoanalytic theory is based on Freuds belief regarding the importance of unconscious motivation for behavior and the role of the id and superego in opposition to each other. Behavioral, educational, and interpersonal theories do not emphasize unconscious conflicts as the basis for symptomatic behavior. Source: Lippincotts Review Series, Mental Health and Psychiatric Nursing. page 280. 34. A client comes to day treatment intoxicated but says he is not. The nurses evaluation of his symptomatology reveals: a. b. c. d. Denial Reaction formation Transference Countertransference

Answer: A It would not be unusual for a client who has severe addiction to come to day treatment intoxicated and deny it. Denial would cause a client to insist he or she is not intoxicated or doesnt have a problem with alcoholism despite concrete evidence of the problem. Reaction formation is a defense mechanism that causes people to act exactly opposite to the way they feel (option B). Transference is the unconscious process of displacing feelings for significant people in the past unto the nurse in the recent relationship (option C). Countertransference is the nurses emotional reaction to client base on feelings of significant people in the nurses past (option D). Source: Review and Rationale Series for nursing Mental Health by Hogan and Smith page 230. 35. The nurse working in obstetrics is reinforcing the physician health teaching about the risks of using substances during pregnancy. The client states that she only drinks a little beer and wine and would never use any dangerous drugs. The nurse then assess for use of which drug that causes the most physical, cognitive, and growth and developmental problems to the fetus? a. b. c. d. Benzodiazepines Hallucinogens Alcohol Cocaine

Answer: C Alcohol use during pregnancy causes dysmorphic prenatal and postnatal difficulties and CNS dysfunction. Other substances cause significant health concerns as well, but not quite as many different kinds of problems (option, B and D). Source: Review and Rationale Series for nursing Mental Health by Hogan and Smith page 231. 36. A young female presents for her school checkup. She denies any medical problem or taking any medications, but she does acknowledge daily laxative use. As the school nurse, what other symptoms or problems would you expect to find? a. b. c. d. Headaches Altered sleep patterns Abnormal eating patterns Intermittent chest pain.

Answer: C Laxative abuse is a method used to control weight by anorexic and bulimics. Eating disorder clients may have cardiac rhythm disturbances but not necessarily chest pain (option D), headaches (option A), or altered sleep (option B) as a result of their disordered eating.
For more nursing reviewers, contact ebookwholeseller@yahoo.com 09394837323

121

Generated by Foxit PDF Creator Foxit Software http://www.foxitsoftware.com For evaluation only.

Source: Review and Rationale Series for nursing Mental Health by Hogan and Smith page 232. 37. A nursing educator is teaching a group of community health nurses on moderating alcohol use. The nurse educator evaluates the groups understanding of harm reduction if the group is able to identify which group is not appropriate for harm reduction? a. b. c. d. Individuals with tolerance Individuals with alcohol skills Individuals unable to control use Individuals with high-dose use

Answer: C Clients who are unable to control their use or are unable to learn strategies to reduce intake and/or harm caused by their use, are not a good candidates for this approach. People with tolerance (option A), alcohol abuse (option B), and high dose use (option D) may be successful in decreasing the frequency and quantity of alcohol they drink. Source: Review and Rationale Series for nursing Mental Health by Hogan and Smith page 232. 38. Some adolescent clients relapse because they feel pressured by their peers. Which skill training could the nurse plan for adolescents in order to assist them in relapse prevention? a. b. c. d. Vocational skills Drinking refusal skills Problem-solving skills Communication skills

Answer: B The quality of an adolescents recovery environment can be helpful or hurtful to someone attempting to maintain sobriety. Friends or acquaintances may encourage a recovering person to use. The recovering adolescent may want to refuse but may not know how. Behavioral rehearsal, saying no thanks to an offer to engage in addictive behavior, can increase a recovering persons confidence. Vocational skills will not help the adolescent refuse a drink (option A ). Problemsolving skills (option C) and communication skills (option D) may be useful but not as helpful as skills directly related to refusing to drink. Source: Review and Rationale Series for nursing Mental Health by Hogan and Smith page 232. 39. A mother tells the nurse during an admission interview that her 2-year-old, who has numerous bruises, has fallen down stairs frequently. The mother is able to provide few details. The nurse evaluates this as: a. b. c. d. Possible child abuse Knowledge deficit pertaining to home safety Normal behavior for a 2-year-old Possible attention deficit disorder.

Answer: A The numerous bruises and the mothers vague explanations of the injuries indicate possible child abuse. Home safety is important but not as important as the childs safety (option B). Falling down stairs frequently is not normal behavior for a 2-year-old (option C), nor is it a symptom of attention deficit disorder (option D) Source: Review and Rationale Series for nursing Mental Health by Hogan and Smith page 256. 40. A nurse is teaching a class on domestic violence to high school students. Which of the following statements by a student would indicate to the nurse that further teaching is needed? a. Violence often begins in a dating relationship. b. The abuser will often apologize and promise to stop.
09394837323

For more nursing reviewers, contact ebookwholeseller@yahoo.com

122

Generated by Foxit PDF Creator Foxit Software http://www.foxitsoftware.com For evaluation only.

c. If you are educated and have money, abuse does not happen. d. Abusers are often excessively jealous and possessive. Answer: C Education and money do not make persons immune from violence. It crosses all socioeconomic lines. Violence does often begin in dating relationship (option A); abusers do apologize and promise to stop (option B); abusers are often excessively jealous and possessive (option D). Source: Review and Rationale Series for nursing Mental Health by Hogan and Smith page 256. 41. A 15-year-old female student comes into the school nurses office asking to be tested for [pregnancy. She confides to the nurse that her boyfriend forced her to have sex against her will. The most appropriate intervention by the nurse would be: a. b. c. d. Administer a pregnancy test. Do teaching on safe sex. Do teaching on birth control method. Identify the students immediate concerns.

Answer: D The client has been raped and nurse needs to respond to the clients immediate concerns. Testing (option A ) and teaching (option B and C ) are secondary interventions. Source: Review and Rationale Series for nursing Mental Health by Hogan and Smith page 256. 42. An adult survivor of child abuse state, Why couldnt I make him stop the abuse? If I were a stronger person, I would have been able to make him stop. Maybe it was my fault he abused me. Based on this data, which would be the most appropriate nursing diagnosis? a. b. c. d. Ineffective family coping Social isolation Chronic low self-esteem Anxiety

Answer: C Inappropriate self-blame and feelings that a child could have stopped an adults abuse indicate a low self-esteem. Option A, B, and D are possible diagnoses for adult survivors of abuse; there is not enough evidence supporting these diagnoses. More data would be needed. Source: Review and Rationale Series for nursing Mental Health by Hogan and Smith page 257. 43. The nurse is assessing a normal appearing 6-year-old brought to the Emergency Department by the mother, who reports that the child vomits every time she eats. The childs history reveals no positive findings as well as several previous similar visits. The mother is very concerned and insists that the child be admitted for a full GI workup. The nurse reports this as possible: a. b. c. d. Anxiety disorder Bulimia nervosa Munchausens syndrome by proxy. Severe food allergies.

Answer: C Munchausens Syndrome by Proxy is characterized by the caregiver reporting or producing symptoms in a child that require hospitalization and invasive procedures. The reports by the mother to have the child hospitalized point to Munchausens. The physical appearance of the child and previous negative physical findings would rule out anxiety disorder (option A), bulimia (option B), and food allergies (option D). Source: Review and Rationale Series for nursing Mental Health by Hogan and Smith page 257.
For more nursing reviewers, contact ebookwholeseller@yahoo.com 09394837323

123

Generated by Foxit PDF Creator Foxit Software http://www.foxitsoftware.com For evaluation only.

44.

In counseling parents who have recently lost a child to death, it is important for the nurse to have already dealt with personal feelings about death, grief, and loss in children. This self-awareness would: a. b. c. d. Assist the nurse in helping the parents to express their grief fully. Prevent the nurse from being personally affected by the loss Prevent the nurse from sharing any personal feelings with the parents. Assist the nurse in avoiding discussion of unpleasant feelings with the parents.

Answer: A The capacity of self-awareness allows the nurse to reflect and make choices. Nurses who understand their own feelings and beliefs will be able to be therapeutic when clients need to address issues which are disturbing and difficult. The death of the child will personally affect the nurse, and it is critical for the nurse to share these feelings with others, including the parents. The nurse must be available both physically and emotionally for the parents in discussing unpleasant and difficult feelings. Source: Review and Rationale Series for nursing Mental Health by Hogan and Smith page 277. 45. An African-American family gathered around their dying grandmothers bed refuses to allow a feeding tube to be removed and to stop feeding her, even after the healthcare team has stated that there was nothing else to be done. The nurse understands the familys resistance to removal of the feeding tube is most likely caused by: a. b. c. d. The refusal to accept the finality of death Their need to try every possible solution Their spiritual and cultural beliefs Their distrust of the healthcare system

Answer: C Spiritual beliefs and practices greatly influence both a persons and familys reaction to death and subsequent behavior. Although option A and B are correct, option C is more correct because their spiritual and cultural beliefs dictate these behaviors. The family may or may not trust the healthcare system. Source: Review and Rationale Series for nursing Mental Health by Hogan and Smith page 277. 46. Outcome criteria for successful counseling for the loss of a clients spouse would include the clients ability to: a. b. c. d. Avoid feelings about the spouses death Immediately memorialize the spouse Attend grief support groups Avoid sharing loss with significant others.

Answer: C A major outcome of grief counseling is to assist the client in sharing his or her loss and to accept support from others. It is critical for the spouse to share the feelings of loss and grief with others. It is too early to memorialize the spouse; the client must grieve the loss of client first. Source: Review and Rationale Series for nursing Mental Health by Hogan and Smith page 278. 47. The nurse working with terminally ill clients understands that culture influences a client and familys reaction to grief, loss, and death by: a. b. c. d. Ignoring inappropriate grieving behaviors Tolerating any expression of grief or loss Establishing symbolic rituals Supporting all individual responses.
09394837323

For more nursing reviewers, contact ebookwholeseller@yahoo.com

124

Generated by Foxit PDF Creator Foxit Software http://www.foxitsoftware.com For evaluation only.

Answer: C Culture dictates acceptable customs and rituals used in the expression of grief, as well as, delineate the appropriate expression of feelings and behaviors. Source: Review and Rationale Series for nursing Mental Health by Hogan and Smith page 278. 48. When questioned by a client about what an advance directive or living will is, the nurse should respond that it states: a. What treatment should be provided or omitted if the client becomes incapacitated b. The practitioners who are allowed to provide care at the end of life c. The caregivers role in providing care at the end of life d. The inheritance requirements for those relatives who are living. Answer: A Advance directives is a general term that refers to a clients written instructions about future medical care, in the event that the client becomes unable to speak or is incapacitated. Specific instructions about what medical treatment the client chooses to omit or refuse (e.g. Ventilator support) in the event that the client is unable to make those decisions is also included. The other options are not part of an advance directive. Source: Review and Rationale Series for nursing Mental Health by Hogan and Smith page 278. 49. When planning care for the client diagnosed with a chronic medical illness, the nurse can anticipate the client needing assistance with issues related to what area? a. b. c. d. Anger Anorexia Apathy Euphoria

Answer: A Option A, anger, is included in the stages of grief as clients grieve for what has been lost. Although clients may experience multiple emotional feelings in response to diagnosis of life changing medical illness, anger is one of the most common emotional responses because of the sudden and often dramatic change in lifestyle. Option B and C might occur but are not considered primary responses. Option D is inappropriate and typically does not occur. Source: Review and Rationale Series for nursing Mental Health by Hogan and Smith page 294. 50. What is an expected outcome related to increasing the level of social support for the terminally ill client? a. b. c. d. Increased number of friends Increased independence Expression of emotion Expression of hope

Answer: C The clients engagement with social supports woul hopefully results in increased ability to express emotions with others. Option A does not necessarily indicate a strong level of social support. Source: Review and Rationale Series for nursing Mental Health by Hogan and Smith page 294. 51. A client expressed feelings of hopelessness and helplessness about her husbands illness and her inability to care for him. Of the following issues, which would be the best for the client to focus on first? a. Her husbands present illness b. Her past losses of significant others c. Her loneliness and isolation in her new surroundings
For more nursing reviewers, contact ebookwholeseller@yahoo.com 09394837323

125

Generated by Foxit PDF Creator Foxit Software http://www.foxitsoftware.com For evaluation only.

d. Her future loss of her husband Answer: B The nurse should help the client identify her coping strategies and her experience with past losses in order to identify the clients strengths and past coping strategies. This helps the client draw on experiences of the past to help her cope and look at events rationally. Focusing on the clients husbands current illness (option A) will only keep the client struck in hopelessness and helplessness. Focusing on loneliness and desolation (option C) and the future loss of her husband (option D) may be appropriate, but the nurse and client need to first examine how the client has coped with the past losses. Source: Review and Rationale Series for nursing Mental Health by Hogan and Smith page 27. 52. While assessing the defense mechanisms used by the client, the nurse recognizes the clients use of defense mechanisms as adaptive when the: a. b. c. d. Mechanism used decreases anxiety Client seeks isolation to avoid stress. Anxiety is expressed in behaviors Client can identify the stressor

Answer: A The purpose of defense mechanism is to reduce anxiety levels and allow the client to function adequately. Seeking isolation to avoid stress (option B) is an unhealthy adaptive strategy. Anxiety is expressed in behavior (option C); however, that behavior can be harmful to clients or others. Recognition of a stressor (option D) is important but may not be used in the clients adaptive use of defense mechanism. Source: Review and Rationale Series for nursing Mental Health by Hogan and Smith page 27. 53. A nurse who practices subtle stereotyping or countertransference can expect the cultural assessment to: a. b. c. d. Be sensitive to the unmet needs of the culture Be open and honest, reflecting the clients concerns Reinforce the nurses prejudices about the culture Facilitate the treatment process.

Answer: C Stereotyping arises out of negative biases; stereotypes are images frozen in time that cause us to see what we expect to see, even when the facts differ from our expectations. Countertransference is the nurses emotional reaction to a client based on feelings for significant people in the nurses past. These would only reinforce the nurses prejudices about the culture and cause the nurse to be insensitive, not sensitive, to the clients need (option A). When governed by stereotyping or countertransference, the nurse is unable to be open and honest (option B) and unable to facilitate effective treatment (option D ). Source: Review and Rationale Series for nursing Mental Health by Hogan and Smith page 28. 54. The nurse should do which of the following as a primary nursing strategy for dealing effectively with the spiritual needs of clients? a. Refer clients to appropriate clergy b. Clarify own spiritual beliefs and values c. Use a spiritual assessment tool. d. Discuss own religiosity with the client. Answer: B The first priority of nurses in assisting clients to manage any area of their lives is to understand themselves and clarify their own spiritual beliefs and values. Referring clients to appropriate
For more nursing reviewers, contact ebookwholeseller@yahoo.com 09394837323

126

Generated by Foxit PDF Creator Foxit Software http://www.foxitsoftware.com For evaluation only.

clergy (option A) may be an effective intervention, but the nurse has adequate skills in meeting many spiritual needs of the clients. Use of a spiritual assessment tool (option C) is important but should be used after the nurse has done self-exploration. Discussing the nurses own religious beliefs (option D) is inappropriate and projects the nurses own religious beliefs onto the client. Source: Review and Rationale Series for nursing Mental Health by Hogan and Smith page 28. 55. Among the following symptoms reported by a grieving older adult, which should concern the nurse the most? a. b. c. d. Occasional shortness of breath Expressed thoughts of being better off dead Guilt about what was done at the time of a loved ones death A morbid preoccupation with worthlessness

Answer: B An older adult who expresses thoughts of death has priority over other choices- safety is always a priority. Everyone experiences grief differently. Older adults often normally experience grief somatically (option A). Guilt about actions or lack of action at the time of a loved ones death (option C) is not uncommon. A morbid preoccupation with worthlessness (option D) is a concern, but safety takes priority. Source: Review and Rationale Series for nursing Mental Health by Hogan and Smith page 28. 56. Primary nursing interventions effective for the impulsive, egocentric, and aggressive behaviors of children with conduct disorders are: a. b. c. d. Limit setting and consistency Open communication and flexible approach Open expression of feeling Assertiveness training

Answer: A Behavior modification is quite effective with children and adolescents. The child is told what is expected, what is not acceptable, and consequences for undesirable behaviors. Open communication is effective, but a flexible approach may be confusing to the child (option B). Open expression of feelings (option C) and assertiveness training (option D) are useful techniques; however, they are more effective within a contrived environment. Source: Review and Rationale Series for nursing Mental Health by Hogan and Smith page 60. 57. The nurse assesses for which of the following common anxiety disorders among children? a. b. c. d. Obsessive-compulsive disorder Simple phobia Separation anxiety disorder Post-traumatic stress disorder (PTSD)

Answer: C Separation anxiety may develop at age, although it is most common in children, with the peak onset between 7 and 9 years old. Obsessive-compulsive disorder (option A), simple anxiety (option B), and PTSD (option D) are less common in children. Source: Review and Rationale Series for nursing Mental Health by Hogan and Smith page 60. 58. In planning the care for a young child with oppositional defiant disorder, the psychiatric nurse would include: a. Reminiscence therapy b. Emotive therapy
For more nursing reviewers, contact ebookwholeseller@yahoo.com 09394837323

127

Generated by Foxit PDF Creator Foxit Software http://www.foxitsoftware.com For evaluation only.

c. Behavior modification d. Cognitive retraining Answer: C Behavior modification is quite effective with children and adolescents. The child is told what is expected, what is not acceptable and the consequences for undesirable behaviors. Reminiscence therapy (option A) is more effective in memory disorders. Emotive therapy (option B) and cognitive retraining (option D) are more effective with psychotherapy and other children. Source: Review and Rationale Series for nursing Mental Health by Hogan and Smith page 60. 59. One of the outcomes of play therapy is to enable the children to: a. b. c. d. Act out feelings in a constructive manner Learn to talk openly about themselves Learn how to give and receive feedback Learn problem-solving skills.

Answer: A Play therapy is especially useful for children under 12 because their developmental level makes them less able to verbalize thoughts and feelings. Learning to talk openly about themselves (option B), learning how to give and receive feedback (option C), and learning problem-solving skills (option D) are not the intended goals of play therapy. Those skills require more structured group and individual activities. Source: Review and Rationale Series for nursing Mental Health by Hogan and Smith page 60. 60. The school nurse who is planning a community education program would include information that one childhood psychiatric disorder that appears to be genetically transmitted is: a. b. c. d. Anxiety Sleepwalking Enuresis Mania

Answer: C Childhood disorders that appear to be genetically transmitted include enuresis, autism, mental retardation, some language disorders, Tourettes syndrome, and attention deficit/hyperactivity disorder (ADHD). Anxiety (option A), sleepwalking (Option B), and mania (option D) do not appear to be genetically transmitted for children. Source: Review and Rationale Series for nursing Mental Health by Hogan and Smith page 61. 61. When assessing an apparently anxious client, questions about anxiety should be: a. b. c. d. Abstract and non-threatening Avoided until the anxiety disappears Avoided until the client brings up the subject Specific and direct

Answer: C Because of shame, clients should be reluctant to talk about anxiety. Questions should be specific, direct, and individualized to the client. Option A is incorrect because when a client is experiencing anxiety abstract thinking and questions should be avoided. Option B and C are incorrect because the nurse should ask direct questions about the clients anxiety. Source: Review and Rationale Series for nursing Mental Health by Hogan and Smith page 117. 62. Which of the following nursing diagnoses has the highest priority for an anxious client?
09394837323

For more nursing reviewers, contact ebookwholeseller@yahoo.com

128

Generated by Foxit PDF Creator Foxit Software http://www.foxitsoftware.com For evaluation only.

a. b. c. d.

Defensive coping Ineffective denial Risk for loneliness Risk for self-directed violence

Answer: D Safety needs generally have a higher priority than psychosocial needs. Option A, B, and C are applicable nursing diagnoses for anxious clients, but safety has the highest priority. Source: Review and Rationale Series for nursing Mental Health by Hogan and Smith page 117. 63. The best goal for a client learning a relaxation technique is that the client will: a. b. c. d. Confront the source of the anxiety Experience anxiety without feeling overwhelmed Keep a journal as a self-monitoring technique Suppress anxious feelings

Answer: B The goal of teaching calming techniques is to assist the client to learn to experience anxiety without feeling threatened and overwhelmed. Relaxation therapy does not assist a client to confront sources of anxiety. Likewise, keeping a journal is a self-monitoring technique but is not used to measure the outcome of relaxation. The goal is not to suppress feelings but to make them more manageable. Source: Review and Rationale Series for nursing Mental Health by Hogan and Smith page 117. 64. The long-term goal, The client will learn new ways of coping with anxiety, is most appropriate at which level of anxiety? a. b. c. d. Mild Moderate Severe Panic

Answer B Long-term goals for moderate anxiety should focus on assisting the client to understand the causes of anxiety and learn new coping strategies. These goals cannot be accomplished when the anxiety level is high because the client cannot focus on learning at this anxiety level. Source: Review and Rationale Series for nursing Mental Health by Hogan and Smith page 117. 65. Which of the following would be the best nursing action for client who is having a panic attack? a. b. c. d. Remain with the client Teach the client to recognize signs of a panic attack Instruct the client to remain alone until the symptoms subside Involve the client in a physical activity

Answer: A To promote safety, nurses should stay with extremely anxious clients. During a panic attack a client is unable to focus on teaching. Physical activity should be avoided during a panic attach. Source: Review and Rationale Series for nursing Mental Health by Hogan and Smith page 117. 66. A client with Dissociative Identity Disorder (DID) is admitted after an overdose of alcohol and benzodiazepines, claiming that another alter did it. The priority nursing diagnosis is: a. Post-trauma response
For more nursing reviewers, contact ebookwholeseller@yahoo.com 09394837323

129

Generated by Foxit PDF Creator Foxit Software http://www.foxitsoftware.com For evaluation only.

b. Risk for self-directed violence c. Personal identity disturbance d. Anxiety Answer: B The overdose of alcohol and benzodiazepines is particularly lethal which demonstrates that the client is potentially harmful to self. The presenting personality may not be depressed, or may not have enough power to prevent the alter that is self-destructive from acting again, so substantial risk remain. Physical safety is priority over the other options. Source: Review and Rationale Series for nursing Mental Health by Hogan and Smith page 149. 67. A client is brought to the emergency room after a brutal physical assault. Although oriented and coherent, she cannot remember the assault or events surrounding it. The priority intervention is to provide: a. b. c. d. Frequent reality orientation Physical comfort an safety Thoughtful questioning for the police report Referral to a community support group

Answer: B The client needs to have physical needs met, including comfort, as the first priority. Creating a sense of safety after an assault is essential as anxiety may fluctuate. Although the other nursing interventions are relevant, they are not priorities and can be deferred to a later time. Source: Review and Rationale Series for nursing Mental Health by Hogan and Smith page 149. 68. A client with Dissociative Identity Disorder (DID) suddenly begins to speak with a childs vocabulary and voice. Which of the following is the most therapeutic response by the nurse? a. b. c. d. You must be feeling very needy. Here are some toys you might enjoy. Can you tell me what is happening? This behavior keeps you from working on your problems.

Answer: C Switching often occurs with increases in anxiety. Asking the client to explain more will help the nurse understand what is happening on a system level, and why the child alter was emergent. Option A and D will increase anxiety. Option B, although helpful, may not provide therapeutic outcome. Source: Review and Rationale Series for nursing Mental Health by Hogan and Smith page 149. 69. The priority nursing diagnosis for a client experiencing amnesia is: a. b. c. d. High risk for self-directed violence Powerlessness Ineffective individual coping Sensory/perceptual alteration

Answer: C Amnesias are result o f being unable to cope with high levels of anxiety. There is no data to suggest other nursing diagnoses. Source: Review and Rationale Series for nursing Mental Health by Hogan and Smith page 149. 70. A client reports episodic depersonalization experiences. Which of the following is an appropriate goal of care?
09394837323

For more nursing reviewers, contact ebookwholeseller@yahoo.com

130

Generated by Foxit PDF Creator Foxit Software http://www.foxitsoftware.com For evaluation only.

a. b. c. d.

The client will describe three stress management techniques by day 2. The client will report no suicidal thoughts by week 1 The client will create a chart of all personalities by week 1 The client will state five personal strengths by day 2.

Answer: A Reducing anxiety through the use of stress management techniques will prevent depersonalization that is a reaction to high levels of anxiety. There is no data to support suicidal thoughts or multiple identities. Improving self-concept is helpful, but is not a priority when anxiety leads to dissociation. Source: Review and Rationale Series for nursing Mental Health by Hogan and Smith page 117. 71. A nurse employed in managed care system collaboration with the treatment team in monitoring a clients progress from psychiatric inpatient care to a community-assisted living program. The role of the nurse can best be described as: a. b. c. d. Advanced practice nurse Case manager Nurse manager Staff nurse

Answer: B In a managed care system, the case manager is responsible for monitoring and ensuring of care, therefore collaborating with the treatment team. Although they provide different levels of care, both the staff nurse and the advanced practice primary care. A staff nurse involves supervision of other nursing personnel. Source: Lippincotts Review Series, Mental Health and Psychiatric Nursing, by Ann Isaac, 3rd edition, page 17. 72. When a nurse establishes a therapeutic relationship with a client, which of the following is the primary focus for the clients care? a. b. c. d. The medical diagnosis The clients needs and problems The nursing diagnosis The clients social interaction skills

Answer: B The nurse establishes the therapeutic relationship, which is a helping relationship, to assist the client in working on his needs and problems. Both medical and nursing diagnosis would be important in understanding the client. However, the nurse provides care for the person, not the diagnosis. Improving social interaction skills may be a focus of nursing intervention, but it is not the purpose of the relationship. Source: Lippincotts Review Series, Mental Health and Psychiatric Nursing, by Ann Isaac, 3rd edition, page 17. 73. Which of the following is the overall purpose of therapeutic communication? a. b. c. d. To analyze client problems To elicit client cooperation To facilitate a helping relationship To provide emotional support

Answer: C The purpose of therapeutic communication is to foster a helping relationship, so that the client can more effectively cope with problems. The other tasks described are part of the helping relationship but are not the overall purpose.
For more nursing reviewers, contact ebookwholeseller@yahoo.com 09394837323

131

Generated by Foxit PDF Creator Foxit Software http://www.foxitsoftware.com For evaluation only.

Source: Lippincotts Review Series, Mental Health and Psychiatric Nursing, by Ann Isaac, 3rd edition, page 17. 74. A nurse is interacting with a client from a different cultural background. Which of the following implementations would the nurse use to provide sensitive care? a. b. c. d. Confronting issues of noncompliance Use of therapeutic silence Use of therapeutic touch Validation of communication

Answer: D It is important for the nurse to frequently validate nurse-client communication to prevent cultural misunderstandings. Confronting noncompliance is inappropriate, because the nurses interpretation of this situation may be quite different from the clients perspective. Therapeutic silence is importance; however, validating communication will ensure culturally sensitive care. Touch must be used in a cautious manner when trying to understand a clients probable response. Touch is not appropriate for all clients. Source: Lippincotts Review Series, Mental Health and Psychiatric Nursing, by Ann Isaac, 3rd edition, page 17. 75. Shortly after a voluntary admission to a psychiatric inpatient unit, a client tells the nurse, I dont know if I should be here. What will my family think? Using reflection, which of the following is the most appropriate response from the nurse? a. Your family can visit you here, and they will see that this is a helpful place. b. You think your family will be upset because you have a psychiatric problem? c. There is still a stigma associated with mental illness. Hopefully your family wont feel this way. d. You are wondering if you made the best decision, and you are concerned about your family reaction. Answer: D Reflection involves rewording the clients statement to indicate a nurses understanding of the clients experience. Source: Lippincotts Review Series, Mental Health and Psychiatric Nursing, by Ann Isaac, 3rd edition, page 17. 76. A nurse is intervening with a client who experienced a crisis following the sudden death of a loved one. Which of the following actions would the nurse take after establishing initial rapport? a. b. c. d. The nurse would ask the client to describe his social support system The nurse would call the clients family to discuss the problem The nurse would encourage the client to describe in detail what happened. The nurse would refer the client to a bereavement support group.

Answer: C It is important for the nurse to assess the individuals perception of the crisis and the events preceding the crisis situation. It is the individuals perception of a problem that determines the crisis. Determining the social support system is important; however, this assessment would occur following the description of the problem. Crisis intervention best occurs in logical, problem-solving sequence and therefore problem description would be the first step. The nurse calling the family to discuss the problem or referring the client to a bereavement support group are interventions that may or may not be appropriate, depending on the clients perception of the problem. Source: Lippincotts Review Series, Mental Health and Psychiatric Nursing, by Ann Isaac, 3rd edition, page 224.
For more nursing reviewers, contact ebookwholeseller@yahoo.com 09394837323

132

Generated by Foxit PDF Creator Foxit Software http://www.foxitsoftware.com For evaluation only.

77.

The nurse would select which of the following approaches in order to best respond to a client in crisis? a. b. c. d. Behavior approach Behavior approach Problem-solving approach Supportive approach

Answer: C The problem-solving method is used in a systemic manner as part of crisis intervention. The behavioral approach or the nondirective approach would not be selected as part of crisis intervention. Although a supportive approach (e.g. supporting client strengths) is part of crisis intervention, the overall method guiding the nurse is the problem-solving approach. Source: Lippincotts Review Series, Mental Health and Psychiatric Nursing, by Ann Isaac, 3rd edition, page 224. 78. Which of the following best describes the role of the nurse as a member of a crisis intervention team? a. b. c. d. Assistive role Collaborative role Educative role Managerial role

Answer: B The nurse works as a member of a health team and therefore needs to collaborate with other professionals in helping the individual resolve the crisis. The nurse may assist the client and may also teach the client; however, the question is asking for the nurses role as a team member. The nurse may or may not be in a managerial role on the team. Source: Lippincotts Review Series, Mental Health and Psychiatric Nursing, by Ann Isaac, 3rd edition, page 224. 79. Which of the following symptoms common in individuals experiencing a crisis would a nurse expect to assess? a. b. c. d. Feeling of depersonalization, loose association, flat affect Lack of regard to social norms, apathy, hallucinations Mood swings, feeling of boundless energy, grandiose beliefs Somatic complaints, difficulty performing roles in life, poor concentration

Answer: D The client who is in crisis has difficulty performing usual role in life because of the acute distress experienced. Somatic symptoms and poor concentration are also common because of the influence of the physiologic stress response. All of the remaining symptoms would commonly occur with the onset of a mental illness. They are not typical of the response of an individual to a crisis. Source: Lippincotts Review Series, Mental Health and Psychiatric Nursing, by Ann Isaac, 3rd edition, page 225. 80. When a client is experiencing a crisis, what is the best rationale for the nurse identifying clients strength? a. b. c. d. It allows the nurse to better determine the nursing diagnosis It helps the nurse understand the clients unique personality The nurse can better educate on assessment of strengths Reinforcing the clients strengths will aid in coping.
09394837323

For more nursing reviewers, contact ebookwholeseller@yahoo.com

133

Generated by Foxit PDF Creator Foxit Software http://www.foxitsoftware.com For evaluation only.

Answer: D An important principle of crisis intervention is the strengthening and supporting of healthy aspects of an individuals functioning. This is important because the client needs to resolve the crisis and individual strengths aid coping. The remaining responses would be correct as general statements of rationale for a nurse assessing client strengths. However, in the situation of a crisis, the best rationale for the nurse identifying strengths is to aid in coping and therefore resolve crisis. Source: Lippincotts Review Series, Mental Health and Psychiatric Nursing, by Ann Isaac, 3rd edition, page 225. 81. When evaluating an imminent suicide risk, which of the following information given by the client would be most significant? a. b. c. d. At least a 2-yaer history of feeling depressed more days than not Divorced from spouse 6 months ago Feeling loss of energy and appetite Reference to suicide as best solution to identified problems

Answer: D An individual who talks about suicide as a solution to problems is at high risk. Suicide threats need to be taken seriously, because this individual does not see any other viable solutions to problems in living. All of the factors in the other answer choices would increase the clients risk for depression; however, actual statements about intent for suicide are red flags for the nurse of imminent danger. Source: Lippincotts Review Series, Mental Health and Psychiatric Nursing, by Ann Isaac, 3rd edition, page 116. 82. A client in an acute psychiatric hospital unit tells a nurse about his plans for suicide. The priority nursing intervention is to : a. b. c. d. Allow the client time alone for reflection Encourage client to use problem solving Follow agency protocol for suicide precautions Stimulate the clients interest in activities

Answer: C The nurse must act to safeguard the client from danger including self-harm. Implementation of specific agency protocol for suicide precautions would be protective for client. A client with suicide intent should not be left alone. One-to-one observations are generally part of suicide precautions. Encouraging the client to use problem solving and stimulating the clients interest in activities would be helpful for a client with depression; however, the priority intervention is to protect the client, and therefore the appropriate intervention is suicide precautions. Source: Lippincotts Review Series, Mental Health and Psychiatric Nursing, by Ann Isaac, 3rd edition, page 116. 83. The community nurse is speaking to a group of new mothers as part of a primary prevention program. Which of the following self-care measures would be most helpful as a strategy to decrease occurrence of mood disorder? a. b. c. d. Keeping busy, so as not confront problem areas Medication with antidepressant Use of crisis intervention services Verbalizing rather than internalizing feelings.

Answer: D Individuals who develop mood disorders often have difficulty expressing feeling, especially feelings of anger toward significant others. Internalizing those feelings can contribute to loss of
For more nursing reviewers, contact ebookwholeseller@yahoo.com 09394837323

134

Generated by Foxit PDF Creator Foxit Software http://www.foxitsoftware.com For evaluation only.

self-esteem and guilt, and therefore negative cognitions and depression. Ignoring problems is not a helpful strategy. Recognizing problems and using problem-solving methods will contribute to mental health. Antidepressants are certainly necessary in the treatment of the mood disorder of depression; however, they are not used in primary prevention. Crisis intervention would be a strategy useful in the immediate treatment of a crisis of a mood disorder. It is not a tool of primary prevention. Source: Lippincotts Review Series, Mental Health and Psychiatric Nursing, by Ann Isaac, 3rd edition, page 117. 84. The husband of a client who has recently lost her job tells the clinic nurse that the clients moods are constantly changing from extremely crying. As past of an immediate assessment of the family situation, the nurse should question the husband and wife about which of the following? a. The clients academic and work history b. The specific history of psychopathology in clients family c. The clients specific symptoms, the duration of the symptoms, and the impact of the symptoms on the family d. The quality of couples marital relationship. Answer: C Assessment of the current family situation would include identifying the clients symptoms, duration of symptoms, and unique impact on this particular family The assessment data related to answer choices (option A) and (option B) would be important, but the immediate assessment would be more specific to the current family crisis. The quality of marital relationship would be one aspect of the entire family situation. Source: Lippincotts Review Series, Mental Health and Psychiatric Nursing, by Ann Isaac, 3rd edition, page 225. 85. During a daily community meeting, a client with bipolar disorder, manic type, begins pacing around the room and talking in aloud voice with a rapid speech. Which of the following is the most appropriate nursing intervention? a. b. c. d. Asking the client to accompany you and moving to a quieter room Allowing the community group to handle the clients behavior Ending the community meeting at this type Offering antianxiety medication to the client

Answer: A The most appropriate intervention when a client with mania begins to escalate is to remove the client from an over-stimulating environment. The community meeting is not an appropriate place for a client who is becoming agitated. The community group may be intimidated by client behavior and reluctant to intervene. The nurse is responsible for limit setting and intervention when client behavior is inappropriate. The community meeting is an important forum for client participation and should not be terminated because one client is upset. Removing the client from an overstimulating environment may be sufficient in helping a client regain self-control. The least restrictive means should be offered prior to use of chemical restraints. Source: Lippincotts Review Series, Mental Health and Psychiatric Nursing, by Ann Isaac, 3rd edition, page 117. 86. The school guidance counselor refers family with an 8-yaer old child to the mental health clinic because of the childs frequent fighting in school and truancy. Which of the following data would be a priority to the nurse doing the initial family assessment? a. b. c. d. The childs performance in school Family education and work history Family perception of current problem The teachers attempts to solve problem
09394837323

For more nursing reviewers, contact ebookwholeseller@yahoo.com

135

Generated by Foxit PDF Creator Foxit Software http://www.foxitsoftware.com For evaluation only.

Answer: C The familys perception of the problem is essential because change in any one part of a family system affects all other parts and the system as a whole. Each member of the family has been affected by the current problems related to the school system and the nurse would be interested in this here-and-now data. The childs performance in school and the teachers attempts to solve the problem are relevant and may be collected; however, priority would be given to the familys perception of the problem. The family education and work history may be relevant, but would not have priority. Source: Lippincotts Review Series, Mental Health and Psychiatric Nursing, by Ann Isaac, 3rd edition, page 246. 87. When interacting with a mother and father who are divorcing, the nurse notes that the major theme of parental disagreement is the behavior of their 13-year-old daughter is irresponsible and lacks respect for his authority, whereas the mother cites the belief that a strict, authoritarian father rules the daughter. Which of the following family systems concepts is this situation an example of? a. b. c. d. Differentiation of child Enmeshed relationship of parents Skewed relationship of parents Triangulation of child

Answer: D The concept of triangles in a family system refers to the emotional configuration involving three family members or two members and an issue. In this situation, the conflict between the spouses is handled by deflecting attention away from the spouses and onto the child. Differentiation is the process of developing autonomy within the family system. Enmeshed relationship between spouses refers to over-involvement with the expectation that everyone in the family think and act alike. A skewed relationship between spouses refers to one spouse who is dysfunctional and therefore roles are imbalanced. Source: Lippincotts Review Series, Mental Health and Psychiatric Nursing, by Ann Isaac, 3rd edition, page 246. 88. The parents of a client with schizophrenia express feelings of responsibility and blame for the clients problem. Which of the following would the nurse providing family education do? a. b. c. d. Acknowledge parents responsibility Explain the biologic nature of schizophrenia Provide referral to a support group Teach the parents various ways they must change.

Answer: B The parents are feeling responsible and this inappropriate self-blame can be limited by supplying them with the facts about the biologic basis of schizophrenia. Acknowledging the parents responsibility is neither accurate nor helpful to the parents to reinforce blame. Support groups are useful; however, the nurse needs to handle the parents self-blame directly instead of making a referral for this problem. Teaching the parents various ways they must change would reinforce the parental assumption of blame; although parents can learn about schizophrenia and what is helpful and not helpful, the approach suggested in this option implies the parents behavior is at fault. Source: Lippincotts Review Series, Mental Health and Psychiatric Nursing, by Ann Isaac, 3rd edition, page 246. 89. The school nurse is conducting a class on parent-child relationships to encourage functional family development. Which of the following things would the nurse teach the class about family resolution of conflict situations?
09394837323

For more nursing reviewers, contact ebookwholeseller@yahoo.com

136

Generated by Foxit PDF Creator Foxit Software http://www.foxitsoftware.com For evaluation only.

a. b. c. d.

Children need to be encouraged to accept parental advice Conflict generally does not arise in functional families Discussion of conflict in a clear, direct way is important Solutions to conflicts should be provided by a neutral party.

Answer: C In families, the ability to discuss difficult issues openly among members reflects healthy behavior. Communication needs to be reciprocal between parents and children. Healthy, functional families are defined not by the absence of conflict but by the manner in which it is handled. The family needs to work out solutions, not have solutions provided by another Source: Lippincotts Review Series, Mental Health and Psychiatric Nursing, by Ann Isaac, 3rd edition, page 247. 90. A 19-year-old client admitted to a psychiatric inpatient facility for treatment of major depression. The nurse learns that the clients father has been on total disability for 3 months since an accident and that the mother has recently experienced relapse of a chronic alcohol problem. The nursing diagnosis established is Family coping: ineffective- compromised related to situation stressors. Which of the following is the most appropriate goal (outcome criterion) for intervention? a. b. c. d. Establish independence of the client from the family system Ensure the mothers compliance with alcohol treatment Identify ownership of problem as belonging to parents. Use family and external resources to cope with problems.

Answer: D There are several problems currently facing this family, including the fathers disability, the mothers relapse, and the childs hospitalization. Mobilizing and using resources from both inside the family (strengths) and outside the family (support systems) will constitute the most appropriate outcome for the nursing diagnosis. Autonomy or differentiation of self takes place within the family system and does not mean that independence from the family system occurs. Ensuring the mothers compliance with alcohol treatment and identifying ownership of the problem as belonging to the parents are incorrect responses, because each member of the family is involved in the current problems. Source: Lippincotts Review Series, Mental Health and Psychiatric Nursing, by Ann Isaac, 3rd edition, page 246. 91. A client with benign essential hypertension has been referred for biofeedback training. Which of the following criteria would the nurse use to evaluate the clients success with this method? a. The client states that his stress level is under control. b. The clients blood pressure is normal while on a decreased dose of antihypertensive medication c. The client uses relaxation methods on a regular basis. d. The client follows recommended diet and medication plan without deviation. Answer: B Successful use of biofeedback enables the client to modify physiologic responses to stress, including blood pressure. A decreased need for an antihypertensive medication is an objective measurement of effectiveness. Although answer choices A and C are outcomes of stress management, they are not specific for biofeedback. Answer choice D would be a successful outcome of the medical treatment program. Source: Lippincotts Review Series, Mental Health and Psychiatric Nursing, by Ann Isaac, 3rd edition, page 77.

For more nursing reviewers, contact ebookwholeseller@yahoo.com

09394837323

137

Generated by Foxit PDF Creator Foxit Software http://www.foxitsoftware.com For evaluation only.

92.

A nurse is teaching a class on stress management. The nurse is questioned about the use of alternative treatments, such as herbal therapy and therapeutic touch. The nurse explains that the advantage of these methods would include all of the following except a. b. c. d. That they can be congruent with many cultural belief systems That they encourage the consumer to take an active role in health management That they promote interrelationships between mind-body-spirit That they usually work better than traditional medical practice

Answer: A Alternative treatment methods are often used as adjuncts to medical method, there is really no current scientific proof that these methods will work better than traditional medicine. This statement is quite global and therefore is not true. The other answer choice options are accurate regarding use of alternative treatment methods. Source: Lippincotts Review Series, Mental Health and Psychiatric Nursing, by Ann Isaac, 3rd edition, page 77. 93. A client hospitalized on an eating disorder unit is monitored by the nurse for one hour after eating. The rationale for this intervention is a. b. c. d. To develop trusting relationship To maintain focus on importance of nutrition To prevent purging behaviors To reinforce behavioral contact

Answer: C The client may experience increased anxiety during treatment and therefore may resume behaviors designed to prevent weight gain, such as vomiting or excessive exercise. Although the other answer choices are important areas for nursing intervention, they do not provide the rationale for remaining with a client for one hour after eating. Source: Lippincotts Review Series, Mental Health and Psychiatric Nursing, by Ann Isaac, 3rd edition, page 78. 94. The initial treatment priority for a client hospitalized for anorexia nervosa on a special eating disorder unit is a. b. c. d. To determine current body image To identify family interaction pattern To initiate refeeding program To promote client independence

Answer: C The physical need to reestablish near-normal weight takes priority because of the physiologic, life-threatening consequences of anorexia. The other answer choices are all important aspects of treatment, but they are not the highest priority in initial treatment. Source: Lippincotts Review Series, Mental Health and Psychiatric Nursing, by Ann Isaac, 3rd edition, page 78. 95. Which of the following attitudes from a nurse would hinder a discussion with an adolescent client about sexuality? a. b. c. d. Accepting Matter-of-fact Moralistic Nonjudgmental

Answer: C
For more nursing reviewers, contact ebookwholeseller@yahoo.com 09394837323

138

Generated by Foxit PDF Creator Foxit Software http://www.foxitsoftware.com For evaluation only.

Adolescents are not likely to feel free to ask questions and participate in a discussion if the nurse has a moralistic attitude toward sexual issues. Having an accepting, matter-of-fact, or nonjudgmental attitude will be helpful in allowing adolescents to feel comfortable discussing sexual issues. Source: Lippincotts Review Series, Mental Health and Psychiatric Nursing, by Ann Isaac, 3rd edition, page 79. 96. A young client, who is a mother for the first time is very anxious about her new parenting role. With the nurses encouragement, she has joined the new mothers support group at the local Y. This part of the plan is an example of: a. b. c. d. Tertiary Prevention Primary Prevention Secondary Prevention Therapeutic Prevention

ANSWER: B Primary Prevention is directed towards health promotion and prevention of problems. A Tertiary Prevention is focused on rehabilitation and the reduction of residual effects of illness. C Secondary Prevention is related to early detection and treatment of problems D There is no category of prevention called therapeutic prevention SOURCE: MOSBYs REVIEW QUESTIONS FOR THE NCLEX RN EXAMINATION BY SAXTON, TH PELLIKANT, NUGENT 5 EDITION # 14 p. 262 97. During an interview with the parents of an adolescent female, the nurse notices that her father continually defends and makes excuses for all his daughters actions whereas her mother seems to feel her daughter is just lazy and that there is nothing wrong with her that she couldnt change with some effort. The nurse recognizes that the dynamic used by the family is known as: a. b. c. d. Coalition Resignation Scapegoating Reaction Formation

ANSWER: A The father is siding with his daughter and supports her whereas the mother accuses her of negative behavior; this is an example of a coalition or alliance; both the mother and the father maybe in denial. B. Resignation is evident when someone gives up. C Scapegoating is when an individual is labeled or blamed by other family members as the cause of the familys problems D Reaction formation is a defense mechanism that causes individuals to overtly behave in a manner that is exactly opposite to what they really feel in an attempt to conceal unacceptable feelings. SOURCE: MOSBYs REVIEW QUESTIONS FOR THE NCLEX RN EXAMINATION BY SAXTON, PELLIKANT, NUGENT 5 TH EDITION # 15 p. 262 98. The nurse is aware that according to Erickson, a young childs increased vulnerability to anxiety in response to separations or pending separations from significant others results from failure to complete the developmental task called: a. b. c. d. Trust Identity Initiative Autonomy

ANSWER: A
For more nursing reviewers, contact ebookwholeseller@yahoo.com 09394837323

139

Generated by Foxit PDF Creator Foxit Software http://www.foxitsoftware.com For evaluation only.

Without the development of trust, the child has little confidence that the significant other will return; separation is considered abandonment by the child. B Without identity, The individual will have a problem forming a social role and a sense of self; this results in identity diffusion and confusion C Without initiative, the individual will experience the development of guilt when curiosity and fantasy about sexual roles occur. D Without autonomy, the individual has little self confidence, develops a deep sense of shame and doubt, and learns to expect defeat. SOURCE: MOSBYs REVIEW QUESTIONS FOR THE NCLEX RN EXAMINATION BY SAXTON, PELLIKANT, NUGENT 5 TH EDITION # 40 p. 265 99. The psychotherapeutic theory that uses hypnosis, dream interpretation, and free association as methods to release repressed feelings is the: a. b. c. d. Behaviorist Model Psychoanalytic Model Psychobiologic Model Social Interpersonal Model

ANSWER: B The psychoanalytic model studies the unconscious and uses the strategies of hypnosis, dream interpretation, and free association as a means of releasing repressed feelings. A the behaviorist model subscribes to the belief that the self and mental symptoms are viewed as learned behaviors that persists because they are consciously rewarding to the individual; this model deals with behaviors on a conscious level of awareness. C the psychobiologic model views emotional and behavioral disturbances as stemming from a physical disease; abnormal behavior is directly attributed to a disease process; this model deals with behaviors on a conscious level of awareness. D the social interpersonal model affirms that crucial social processes are involved in the development and resolution of disturbed behavior; this model deals with behavior on a conscious level of awareness. SOURCE: MOSBYs REVIEW QUESTIONS FOR THE NCLEX RN EXAMINATION BY SAXTON, TH PELLIKANT, NUGENT 5 EDITION # 46 p. 265 100. The best initial approach to take with a self- accusatory, guilt ridden client would be to: a. b. c. d. Contradict the clients persecutory delusions Accept the clients statements as the clients beliefs. Medicate the client when these thoughts are expressed. Redirect the client whenever a negative topic is mentioned

ANSWER: B The nurse must accept the clients statement and beliefs as real to the client to develop trust and move into a therapeutic relationship. A Clients cant be argued out of delusions C These feelings and thoughts are constant; this would result in an overdose. D Redirecting the clients conversation whenever negative topics are brought up adds to the clients feelings that negative thoughts are correct. SOURCE: MOSBYs REVIEW QUESTIONS FOR THE NCLEX RN EXAMINATION BY SAXTON, TH PELLIKANT, NUGENT 5 EDITION# 77 p. 268

For more nursing reviewers, contact ebookwholeseller@yahoo.com

09394837323

140

You might also like